You are on page 1of 228

9B3LP_00.

qxd 18/09/2003 15:17 Page i

YEAR 9 TEACHERS PACK 3


9B3LP_00.qxd 18/09/2003 15:17 Page ii

Contents

Introduction v 5 Handling Data 1


CD-ROM Manual x
5.1 Statistical investigations 56
5.2 Scatter graphs and correlation 58
1 Algebra 1 & 2 5.3 Scatter graphs and lines of
best fit 60
1.1 Sequences 2 5.4 Time series graphs 62
1.2 Pattern spotting 4 5.5 Two-way tables 64
1.3 Functions 6 5.6 Cumulative frequency diagrams 66
1.4 Graphs 8 5.7 Estimation of a mean from
1.5 Limits of sequences 10 grouped data 68
2 Number 1 6 Shape, Space and Measures 2
2.1 The four rules governing fractions 12 6.1 Similar triangles 70
2.2 Percentages and compound 6.2 Metric units for area and volume 72
interest 14 6.3 Length of an arc and area
2.3 Reverse percentages and of a sector 74
percentage change 16 6.4 Volume of a cylinder 76
2.4 Direct and inverse proportion 18 6.5 Rates of change 78
2.5 Ratio in area and volume 20
2.6 Numbers between 0 and 1 22 7 Number 2
2.7 Reciprocal of a number 24
2.8 Rounding and estimation 26 7.1 Standard form 80
7.2 Multiplying with numbers in
3 Algebra 3 standard form 82
7.3 Dividing with numbers in
3.1 Simultaneous equations 28 standard form 84
3.2 Solving by substitution 30 7.4 Upper and lower bounds 1 86
3.3 Find the nth term for a quadratic 7.5 Upper and lower bounds 2 88
sequence 32 7.6 Recurring decimals 90
3.4 Equations involving fractions 34 7.7 Efficient use of a calculator 92
3.5 Inequalities 36
3.6 Graphs showing direct 8 Algebra 4
proportion 38
3.7 Solving simultaneous equations 8.1 Index notation with algebra 94
by graphs 40 8.2 Square roots, cube roots and
other fractional powers 96
4 Shape, Space and Measures 1 8.3 Quadratic graphs 98
8.4 Cubic graphs 100
4.1 Pythagoras theorem 42
4.2 Solving problems using 9 Handling Data 2
Pythagoras theorem 44
4.3 Loci 46 9.1 Probability statements 102
4.4 Congruent triangles 48 9.2 Mutually exclusive events and
4.5 Circle theorems 50 exhaustive events 104
4.6 Tessellations and regular 9.3 Combining probabilities and
polygons 52 tree diagrams 106
4.7 Practical Pythagoras 54 9.4 Estimates of probability 108

ii HarperCollinsPublishers Ltd 2003


9B3LP_00.qxd 18/09/2003 15:17 Page iii

10 Shape, Space and Measures 3 13 Handling Data 3


10.1 Fractional enlargements 110 13.1 Revision of statistical
10.2 Trigonometry: The tangent of techniques 160
an angle 112 13.2 A handling data project 162
10.3 Trigonometry: The sine and
cosine of an angle 114 14 Shape, Space and Measures 4
10.4 Solving problems using
trigonometry 116 14.1 Shape and space revision 164
14.2, 14.3 Shape and space investigation 166
11 Algebra 5 14.4 Symmetry revision 168
14.5, 14.6 Symmetry investigation 170
11.1 Expansion 118
11.2 Factorisation 120 15 Handling Data 4
11.3 Quadratic expansion 122
11.4 Quadratic factorisation 124 15.1 Revision of probability 172
11.5 Change of subject 126 15.2 A probability investigation 174

12 Solving Problems and Revision 16 GCSE Preparation


12.1 Fractions, percentages and 16.1 Reinforcement of Number 176
decimals 128 16.2 Reinforcement of Number 178
12.2 The four rules; ratio; 16.3 Reinforcement of Number 180
standard form 130 16.4 Reinforcement of Number 182
12.3 Rules of algebra and linear 16.5 Reinforcement of Number 184
equations 132 16.6 Reinforcement of Number 186
12.4 Graphs 134 16.7 Reinforcement of Number 188
12.5 Shape, Space and Measures 136
12.6 Handling Data 138
SATs Paper 140

HarperCollinsPublishers Ltd 2003 iii


9B3LP_00.qxd 18/09/2003 15:17 Page iv
9B3LP_00.qxd 18/09/2003 15:17 Page v

Introduction

This is the higher-level teaching text for Year 9. It accompanies Maths Frameworking Year 9 Pupil Book 3 and
caters for students working at Levels 68. Students who are working at Levels 35 are catered for by Pupil Book 1
and Teachers Pack 1. Those working at Levels 57 are catered for by Pupil Book 2 and Teachers Pack 2.
Maths Frameworking has been based totally on the finalised National Numeracy Strategy document. The detailed
lesson plans deliver core material from the Frameworks medium-term plans. We have reduced the teaching time
from the 105 hours recommended in the NNS to around 85 one-hour lessons. This should enable teachers some
flexibility to include tests, extended activities and revision classes in their teaching programme, and allows for the
normal events that disrupt teaching time.
The lesson plans have the following features:
Framework objectives to identify the key learning outcomes from the Framework
Engaging Oral and mental starter activities to involve the whole class
Main lesson activities to help you lead students into exercise questions
Plenary guidance to round off the NNS three-part lesson
Key words which highlight when to introduce Framework Vocabulary terms
Extra Homework questions to consolidate and extend learning
Answers for all pupil book exercises, homework and SAT-style questions

PLUS
The free CD-ROM that comes with each teacher pack allows you to extract text and graphics from the
lesson plans, to help produce customised lessons for individualised teaching programmes. Diagrams can
also be reproduced for use on overhead projectors or electronic whiteboards. Full details of how to use
this resource are given on pages xxii.

Chapter numbers and titles in Maths Frameworking follow the NNS medium-term plans. Due to the break caused
by KS3 National Tests, the following specific approach has been taken to tackling the framework objectives in
later chapters:
Chapter 12 is devoted to revision and is followed by a mock SATs paper, for practice prior to KS3 National Tests.
Full answers and a tutorial section for the test, suitable for individual student use, are provided on the CD-ROM.
Chapters 13 to 15 allow students to consolidate and extend knowledge of Handling Data and Shape, Space
and Measures through a range of investigation tasks.
Chapter 16 consolidates Number and Algebra and prepares students for KS4 work.
The Oral and mental starters are designed to work with minimal specialised equipment a blackboard and a
piece of chalk would suffice but resources such as OHPs, A3-sized target boards, counting sticks, number
squares, student white boards and number fans make the activities easier to present and more accessible to
students. A selection of inexpensive or free numeracy resources are available from Collins. See the accompanying
website www.mathsframeworking.com for details.
The authors recognise that ICT provision in schools is varied and we have tried not to commit teachers to an
activity that they could not carry out. However, suggestions for activities using ICT are included throughout the
lesson plans and Pupil Book exercises. Some lessons also address cross-curricular issues such as Literacy and
Citizenship.
For further information and resources for ICT integration, visit www.mathsframeworking.com
The NNS is intended to improve standards. This can only be done by the good work of teachers in the classroom.
The authors appreciate the good work teachers do and hope that Maths Frameworking proves a suitable resource
to help them.
Kevin Evans, Keith Gordon, Trevor Senior and Brian Speed

HarperCollinsPublishers Ltd 2003 v


9B3LP_00.qxd 18/09/2003 15:17 Page vi

Framework Objectives
Matching Chart
This chart matches the National Strategy Framework Objectives (Year 9, core and extension) to specific lesson plans in
Maths Frameworking Year 9 Teachers Pack 3.

Objectives Chapter title Lesson number and title

Using and applying mathematics to solving problems Chapter title Lesson number and title
Solve increasingly demanding problems and evaluate solutions; explore Solving Problems and Revision 12.1 Fractions, percentages and
connections in mathematics across a range of contexts: number, algebra, decimals
shape, space, and measures, and handling data. 12.2 Long multiplication and
division; ratio; directed numbers
Represent problems and synthesise information in algebraic, geometric or Solving Problems and Revision 12.3 Rules of algebra and linear
graphical form; move from one form to another to gain a different equations
perspective on the problem. 12.4 Graphs
12.5 Shape, Space and Measures
Solve substantial problems by breaking them into simpler tasks, using a Handling Data 3 13.2 A handling data project
range of efficient techniques, methods and resources, including ICT.
Present a concise, reasoned argument, using symbols, diagrams, graphs Shape, Space and Measures 4 14.2, 14.3 Shape and space
and related explanatory text; give solutions to problems an appropriate investigation
degree of accuracy. 14.5, 14.6 Symmetry investigation
Suggest extensions to problems, conjecture and generalise; identify Shape, Space and Measures 4 14.2, 14.3 Shape and space
exceptional cases or counter-examples, explaining why. investigation
14.5, 14.6 Symmetry investigation

Numbers and the number system


Place value, ordering and rounding
Use rounding to make estimates. Number 1 2.8 Rounding and estimation
Write numbers in standard form. Number 2 7.1 Standard form
Understand upper and lower bounds; round numbers to three decimal Number 2 7.4 Upper and lower bounds 1
places and a given number of significant figures. 7.5 Upper and lower bounds 2

Fractions, decimals, percentages, ratio and proportion


Use efficient methods to add, subtract, multiply and divide fractions, Number 1 2.1 The four rules governing
interpreting division as a multiplicative inverse; cancel common factors fractions
before multiplying or dividing. Handling Data 2 9.2 Mutually exclusive events and
exhaustive events
9.3 Combining probabilities and
tree diagrams
Recognise when fractions or percentages are needed to compare Number 1 2.2 Percentages and compound
proportions; solve problems involving percentage changes. interest
Solving Problems and Revision 12.1 Fractions, percentages and
decimals
Use proportional reasoning to solve a problem, choosing the correct Number 1 2.3 Reverse percentages and
numbers to take as 100%, or as a whole; compare two ratios; interpret percentage change
and use ratio in a range of contexts, including solving word problems. 2.4 Direct and inverse proportion
Solving Problems and Revision 12.1 Fractions, percentages and
decimals
12.2 Long multiplication and
division; ratio; directed
numbers
Understand the implications of enlargement for area and volume. Number 1 2.5 Ratio in area and volume
Recognise and use reciprocals. Number 1 2.7 Reciprocal of a number
Know that a recurring decimal is an exact fraction. Number 2 7.6 Recurring decimals
Use algebraic methods to convert a recurring decimal to a fraction in Number 2 7.6 Recurring decimals
simple cases.

vi HarperCollinsPublishers Ltd 2003


9B3LP_00.qxd 18/09/2003 15:17 Page vii

Objectives Chapter title Lesson number and title

Calculations
Understand the effects of multiplying and dividing by numbers between 0 Number 1 2.6 Numbers between 0 and 1
and 1; use the laws of arithmetic and inverse operations.
Estimate calculations by rounding numbers to one significant figure and Number 1 2.8 Rounding and approximation
multiplying or dividing mentally.
Use known facts to derive unknown facts; extend mental methods of Oral and mental starter activities
calculation, working with decimals, fractions, percentages, factors, throughout
powers and roots; solve word problems mentally.
Check results using appropriate methods. Throughout

Calculator methods
Use the reciprocal key of a calculator. Number 2 2.7 Reciprocal of a number
Enter numbers in standard form into a calculator and interpret the display. Number 2 7.2 Multiplying with numbers in
standard form
7.3 Dividing with numbers in
standard form
Use a calculator efficiently and appropriately to perform complex Number 2 7.7 Efficient use of a calculator
calculations with numbers of any size, knowing not to round during
intermediate steps of a calculation; use the constant, ( and sign change
keys, function keys for powers, roots and fractions, brackets and the
memory.
Enter numbers and interpret the display in context (negative numbers, Solving Problems and Revision 12.1 Fractions, percentages and
fractions, decimals, percentages, money, metric measures, time). decimals
12.2 The four rules, ratios and
directed numbers

Algebra
Equations, formulae and identities
Solve a pair of simultaneous linear equations by eliminating one Algebra 3 3.1 Simultaneous equations
variable. 3.2 Solving by substitution
Construct and solve linear equations with integer coefficients (with and Algebra 3 3.1 Simultaneous equations
without brackets, negative signs anywhere in the equation, positive or 3.2 Solving by substitution
negative solution), using an appropriate method.
Solve linear inequalities in one variable, and represent the solution set on Algebra 3 3.5 Inequalities
a number line; begin to solve inequalities in two variables.
Solve problems involving direct proportion using algebraic methods, Algebra 3 3.6 Graphs showing direct
relating algebraic solutions to graphical representations of the equations; proportion
use ICT as appropriate.
Link a graphical representation of an equation or a pair of equations to Algebra 3 3.7 Solving simultaneous equations
the algebraic solution. by graphs
Simplify or transform algebraic expressions by taking out single-term Algebra 5 11.1 Expansion
common factors; add simple algebraic fractions. 11.2 Factorisation
Square a linear expression, expand the product of two linear expressions Algebra 5 11.2 Factorisation
of the form x n and simplify the corresponding quadratic expression; 11.3 Quadratic factorisation
establish identities such as a2 b2 = (a + b)(a b).
Derive and use more complex formulae, and change the subject of a Algebra 5 11.5 Change of subject
formula.

Sequences, functions and graphs


Generate terms of a sequence using term-to-term and position-to-term Algebra 1 & 2 1.1 Sequences
definitions of the sequence, on paper and using ICT. 1.2 Pattern spotting
1.5 Limits of sequences
Generate sequences from practical contexts and write an expression to Algebra 1 & 2 1.2 Pattern spotting
describe the nth term of an arithmetic sequence.
Find the inverse of a linear function. Algebra 1 & 2 1.3 Functions
Construct functions arising from real-life problems and plot their Algebra 1 & 2 1.4 Graphs
corresponding graphs.
Find the next term and the nth term of quadratic sequences and functions Algebra 3 3.3 Find the nth term for a
and explore their properties. quadratic sequence
Generate points and plot graphs of linear functions ( y given implicitly in Algebra 3 3.6 Graphs showing direct
terms of x), e.g. ay + bx = 0, y + bx + c = 0, on paper and using ICT; given Algebra 5 proportion
values for m and c, find the gradient of lines given by equations of the Solving Problems and Revision 3.7 Solving simultaneous equations
form y = mx + c. by graphs
Plot graphs of simple quadratic and cubic functions, e.g. y = x2, Algebra 4 8.3 Quadratic graphs
y = 3x2 + 4, y = x3. 8.4 Cubic graphs

HarperCollinsPublishers Ltd 2003 vii


9B3LP_00.qxd 18/09/2003 15:17 Page viii

Objectives Chapter title Lesson number and title

Integers, powers and roots


Know and use the index laws (including in generalised form) for Algebra 4 8.1 Index notation with algebra:
multiplication and division of positive integer powers; begin to extend negative powers
understanding of index notation to negative and fractional powers, 8.2 Square roots, cubes roots and
recognising that the index laws can be applied to these as well. other fractional powers
Use ICT to estimate square roots and cube roots. Algebra 4 8.3 Square roots, cube roots and
other fractional powers

Shape, space and measures


Geometrical reasoning: lines, angles and shapes
Understand and apply Pythagoras theorem. Shape, Space and Measures 1 4.1 Pythagoras theorem
4.2 Solving problems using
Pythagoras theorem
4.7 Practical Pythagoras
Understand congruence. Shape, Space and Measures 1 4.4 Congruent triangles
Apply the conditions SSS, SAS, ASA or RHS to establish the congruence Shape, Space and Measures 1 4.4 Congruent triangles
of triangles.
Know that the tangent at any point on a circle is perpendicular to the Shape, Space and Measures 1 4.5 Circle theorems
radius at that point; explain why the perpendicular for the centre to the
chord bisects the chord.
Distinguish between practical demonstration and proof. Shape, Space and Measures 1 4.5 Circle theorems
4.7 Practical Pythagoras
Explain how to find, calculate and use the interior and exterior angles of Shape, Space and Measures 1 4.6 Tessellations and regular
regular polygons. polygons
Know that if two 2-D shapes are similar, corresponding angles are equal Shape, Space and Measures 3 10.1 Fractional enlargements
and corresponding sides are in the same ratio.
Solve problems using properties of angles, of parallel and intersecting Solving Problems and Revision 12.5 Shape, Space and Measures
lines, and of triangles and other polygons, justifying inferences and
explaining reasoning with diagrams and text.
Visualise and use 2-D representations of 3-D objects. Shape, Space and Measures 4 14.4 Symmetry revision

Transformations
Enlarge 2-D shapes given a fractional scale factor; recognise the similarity Shape, Space and Measures 3 10.1 Fractional enlargements
of the resulting shapes; understand the implications of enlargement for
area and volume.
Transform 2-D shapes by combinations of translations, rotations and Shape, Space and Measures 4 14.4 Symmetry revision
reflections, on paper and using ICT; know that translations, rotations and
reflections preserve length and angle and map objects on to congruent
images; identify reflection symmetry in 3-D shapes.

Construction and loci


Find the locus of a point that moves according to a more complex rule, Shape, Space and Measures 1 4.3 Loci
involving loci and simple constructions.
Know from experience of constructing them that triangles given SSS, Shape, Space and Measures 1 4.4 Congruent triangles
SAS, ASA or RHS are unique, but that triangles given SSA or AAA are
not.

Coordinates
Find points that divide a line in a given ratio, using the properties of Shape, Space and Measures 2 6.1 Similar triangles
similar triangles.

Measures and mensuration


Use units of measurement to calculate, estimate, measure and solve Shape, Space and Measures 2 6.2 Metric units for area and
problems in a variety of contexts; convert between area measures volume
(mm2 to cm2, cm2 to m2, and vice versa) and between volume measures Shape, Space and Measures 4 14.1 Shape and space revision
(mm3 to cm3, cm3 to m3, and vice versa).
Know and use the formulae for length of arcs and area of sectors of Shape, Space and Measures 2 6.3 Length of an arc and area of a
circles. sector
Calculate lengths, areas and volumes in right prisms, including Shape, Space and Measures 2 6.4 Volume of a cylinder
cylinders.
Understand and use measures of speed (and other compound measures Shape, Space and Measures 2 6.5 Rates of change
such as density or pressure) to solve problems; solve problems involving
constant or average rates of change.

viii HarperCollinsPublishers Ltd 2003


9B3LP_00.qxd 18/09/2003 15:17 Page ix

Objectives Chapter title Lesson number and title

Begin to use sine, cosine and tangent in right-angled triangles to solve Shape, Space and Measures 3 10.2 Trigonometry: The tangent of an
problems in two dimensions. angle
10.3 Trigonometry: The sine and
cosine of an angle
10.4 Solving problems using
trigonometry
Know and use the formulae for the circumference and area of a Shape, Space and Measures 4 14.1 Shape and space revision
circle.
Calculate the surface area and volume of right prisms. Shape, Space and Measures 4 14.1 Shape and space revision

Handling data
Specifying a problem, planning and collecting data
Suggest a problem to explore using statistical methods, frame questions Handling Data 1 5.1 Statistical investigations
and raise conjectures.
Design a survey or experiment to capture the necessary data from one Handling Data 1 5.1 Statistical investigations
or more sources; determine the sample size and degree of accuracy
needed; design, trial and if necessary refine data collection sheets.
Identify possible sources of bias and plan how to minimise it. Handling Data 1 5.1 Statistical investigations
Handling Data 3 13.2 A handling data project
Discuss how data relate to a problem; identify possible sources, including Handling Data 1 5.1 Statistical investigations
primary and secondary sources. Handling Data 3 13.1 Revision of statistical
techniques
Design and use two-way tables. Handling Data 1 5.5 Two-way tables

Processing and representing data, using ICT as appropriate


Select, construct and modify, on paper and using ICT, suitable graphical Handling Data 1 5.2 Scatter graphs and correlation
representation to progress an enquiry, including: 5.3 Scatter graphs and lines of
line graphs for time series; best fit
scatter graphs to develop further understanding of correlation; 5.4 Time series graphs
identify key features present in the data. Handling Data 3 13.2 A handling data project
Select, construct and modify, on paper and using ICT, suitable graphical Handling Data 1 5.3 Scatter graphs and lines of best
representation to progress including lines of best fit by eye, understanding fit
what they represent.
Find the median and quartiles for large data sets; estimate the mean, Handling Data 1 5.6 Cumulative frequency diagrams
median and interquartile range of a large set of grouped data. 5.7 Estimation of mean from
grouped data
Find summary values that represent the raw data, and select the statistics Handling Data 3 13.1 Revision of statistical
most appropriate to the problem. techniques

Interpreting and discussing results


Analyse data to find patterns and exceptions, look for cause and effect Solving Problems and Revision 12.6 Handling data
and try to explain anomalies. Handling Data 3 13.2 A handling data project
Interpret graphs and diagrams and draw inferences to support or cast Solving Problems and Revision 13.2 A handling data project
doubt on initial conjectures; have a basic understanding of correlation. Handling Data 3
Examine critically the results of a statistical enquiry, and justify choice of Handling Data 3 13.2 A handling data project
statistical representation in written presentations, recognising the
limitations of any assumptions and their effect on conclusions drawn.
Communicate interpretations and results of a statistical enquiry using Handling Data 3 12.6 Handling data
selected tables, graphs and diagrams in support, using ICT as appropriate. 13.2 A handling data project
Identify what extra information may be required to pursue a further line of Handling Data 3 13.2 A handling data project
enquiry.

Probability
Use the vocabulary of probability in interpreting results involving Handling Data 2 9.1 Probability statements
uncertainty and prediction. Handling Data 4 15.1 Revision of probability
Identify all the mutually exclusive outcomes of an experiment; know that Handling Data 2 9.2 Mutually exclusive events and
the sum of probabilities of all mutually exclusive outcomes is 1 and use exhaustive events
this when solving problems. 9.3 Combining probabilities and
tree diagrams
Handling Data 4 15.1 Revision of probability
Understand relative frequency as an estimate of probability and use this Handling Data 2 9.4 Estimates of probability
to compare outcomes of experiments. Handling Data 4 15.1 Revision or probability
15.2 A probability investigation
Estimate probabilities from experimental data. Handling Data 2 9.4 Estimates of probability
Handling Data 4 15.1 Revision of probability
Compare experimental and theoretical probabilities in a range of Handling Data 4 15.1 Revision of probability
contexts; appreciate the difference between mathematical explanation 15.2 A probability investigation
and experimental evidence.

HarperCollinsPublishers Ltd 2003 ix


9B3LP_00.qxd 18/09/2003 15:17 Page x

Maths Frameworking
Year 9 Teacher Pack 3 CD-ROM

This free Maths Frameworking Teacher Pack CD- Navigating the CD-ROM
ROM provides all the pages of this pack in PDF
format. These can be read by Adobe Acrobat Use the black, triangular direction buttons at the
Reader. If your computer does not already have top of the screen to move forwards or backwards
the Acrobat Reader software it can be installed between pages of text.
directly from the CD-ROM (please refer to the You can also navigate your way around by
installation instructions below). clicking on the bookmarks to each lesson, that
If your computer already has Acrobat Reader appear on the left hand side of the screen. If a plus
installed, follow these steps to view the Maths or minus sign appears to the left of a bookmark
Frameworking Teacher Pack CD-ROM: then you can click on this to show or hide
subordinate bookmarks.
Macintosh
Insert the Maths Frameworking Teacher Pack
CD-ROM into your CD-ROM drive. Printing the PDF pages
Double-click the Maths Frameworking icon. Select the print options you want by using Page
Setup in the File menu. When you are ready to
PC print, select Print from the File menu and
For Windows: specify the pages that you wish to print.
Click the Start button and select Run.
Type D:\MF.pdf. If you are not using the D
Adapting the text
drive as your CD-ROM drive, replace D with
the appropriate letter. You can select text or a graphic from the lesson
Click OK.
plans and copy it to the Clipboard. Once the
selected text or graphic is on the Clipboard you
can switch to another application, such as a word
processor or graphics package, and paste it into a
Using the Maths new or existing document. (Note: If a font copied
Frameworking Teacher from a PDF document is not available on the
Pack CD-ROM system displaying the copied text, the font cannot
be preserved. Helvetica is substituted.)
These pages contains brief guidance to help you
to move around the CD, to enlarge and print
pages and to adapt any of the activities to suit To select text and copy it to the Clipboard:
your own requirements.
For further, extensive help in using Acrobat Reader 1 From the Tool Bar choose the Text Select icon.
with the CD-ROM, select Reader Online Guide
from the Help menu within Acrobat Reader.

To select a line (or lines) of text, select the first


letter of the sentence or phrase and drag to the
last letter. To select a vertical section of text
without selecting text on either side, hold down
the Ctrl (Windows and UNIX) or Option (Mac

x HarperCollinsPublishers Ltd 2003


9B3LP_00.qxd 18/09/2003 15:17 Page xi

OS) key as you drag across the document. Adobe Acrobat Reader 5
To select all the text on the page, choose Edit > software
Select All.
2 From the Edit menu select Copy to copy the
Installation
selected text to the Clipboard. You can then
Macintosh
view what you have selected by choosing
Show Clipboard from the Window menu. Insert the CD-ROM into your CD-ROM drive.
(Note: In Windows 95, the Clipboard viewer is Double-click the Install Acrobat Reader 5 icon
not installed by default; therefore, you cannot that appears in the window.
use the Show Clipboard command until you
install it. Install the Clipboard viewer by PC
choosing Start > Settings > Control Panel >
For Windows:
Add/ Remove Programs and clicking the
Click the Start button and select Run.
Windows Setup tab. Double-click Accessories,
Type D:\acroread\rp500enu.exe. If you are
check Clipboard viewer, and click OK.)
not using the D drive as your CD-ROM drive,
replace D with the appropriate letter.
To select and copy graphics to the Click OK.
Clipboard: Note that this is Adobe Acrobat 5 installer, but
all files on this CD-ROM are compatible with
1 Choose the Graphics Select tool by holding
Adobe Acrobat 4. Adobe Acrobat 5 will not run
down the mouse button on the Text Select icon
on Windows 3.1 or non-Power Macintoshes.
and dragging to the Graphics Select tool. Or
press Shift-V as necessary to cycle through the
group of tools. The cursor changes to a cross- System Requirements
hair icon.
Macintosh
PowerPC processor
Mac OS software version 8.6, 9.0.4, 9.1, or
Mac OS X
32 MB of RAM (with virtual memory on)
(64 MB recommended)
150 MB of available hard-disk space
2 Drag a rectangle around the graphic you want
to copy. To deselect the graphic and start over, PC
click anywhere outside the selected graphic.
3 From the Edit menu select Copy to copy the Intel Pentium processor
selected graphic to the Clipboard. The graphic Microsoft Windows 95 OSR 2.0, Windows 98,
is copied in a WMF format for Windows, and a Windows Millennium, Windows NT 4.0 with
PICT for Macintosh. Service Pack 5 or 6, Windows 2000, or
(Notes: Copied graphics may include text, but Windows XP
text copied with this tool will not be editable by 32 MB of RAM (64 MB recommended)
a word processor. Use the Text Select tool for 150 MB of available hard-disk space
any text you wish to be able to edit.
In case of copied graphics being of low Help
resolution, try zooming in, using the magnifying
glass tool, to make the desired section appear as When using Acrobat Reader, select Reader
large as possible before selecting and copying it.) Online Guide from the Help menu.

HarperCollinsPublishers Ltd 2003 xi


9B3LP_00.qxd 18/09/2003 15:17 Page xii

Attributions Restrictions on use


Adobe and Acrobat are trademarks of Adobe All rights reserved. Maths Frameworking Year 9
Systems Incorporated. Teacher Pack 3 CD-ROM must not be sold,
rented, leased, sub-licensed, lent, assigned or
Windows is a registered trademark and Windows
transferred, in whole or in part, to third parties. No
NT is a trademark of Microsoft in the U.S. and
part of this CD-ROM may be reformatted,
other countries. Pentium is a registered trademark
adapted, varied or modified by the user other than
of Intel Corporation.
specifically for teaching purposes where
Macintosh and Power Macintosh are registered enlargements and/or minor adaptations may be
trademarks of Apple Computer, Inc. necessary. This CD-ROM may not be reproduced
or transmitted in any form or by any means,
without the permission of the publishers other
than the form of printed copies for single use only.
Published by HarperCollinsPublishers Ltd
7785 Fulham Palace Road, London W6 8JB
HarperCollinsPublishers Ltd 2003
First published 2003

xii HarperCollinsPublishers Ltd 2003


9B3LP_00.qxd 18/09/2003 15:17 Page 1

Maths Frameworking Year 9


Lesson Plans

For use with Maths Frameworking Year 9 Pupil Book 3

HarperCollinsPublishers Ltd 2003 1


9B3LP_01.qxd 18/09/2003 15:17 Page 2

CHAPTER
1 Algebra 1 & 2
LESSON Framework objectives Sequences
Generate terms of a sequence using position-to-term definitions of the sequence,
1.1 on paper and using ICT.

Oral and mental starter


Put on the board 102 and ask: What does this represent?
You want the response Ten squared, as well as A hundred and Ten times ten.
Then put on the board 103 and ask: What does this represent?
You want the response Ten cubed, as well as A thousand and Ten times ten
times ten. You may even get One hundred times ten.
Now put on the board 104 and ask: What does this represent?
The response is now Ten to the power four and Ten thousand.
Now jump to 106 and ask: What does this represent?
You are looking for Ten to the power six and One million.
Next, put on the board 109 and ask: What does this represent? One billion is
the response that you want.
Then put on the board 1012: What does this represent? One trillion is the
response that you want.
Talk about the use made of powers to represent large numbers.
Then ask them about 10100: What does this represent? This is a googol, but you
are unlikely to get a response.

Main lesson activity


Put on the board T(n) = 2n + 7 and explain that this is a rule describing a
sequence. It gives the nth term in the sequence, from which every term can be
found by substituting the integers 1, 2, 3, 4, .
Get the class to generate this sequence: 9, 11, 13, 15, .
Notice how the sequence goes up in 2s and that the first term is 2 + 7.
Go through with the class the building up of the sequence with T(n) = 3n + 2.
This generates 5, 8, 11, 14, .
Notice that the sequence goes up in multiples of 3, the number in front of the n,
plus 2.
Show the class that this rule will also work for negative numbers.
Go through the sequence with T(n) = 4n 3. This sequence is 1, 5, 9, 13, .
Now put on the board 9, 13, 17, 21, and ask the class what the nth term is for
this sequence.
Ask the students for the reasons why they have suggested various rules. Lead
them to the fact that, since 4 is added on every time to obtain the next term, the
nth term will start with 4n. Then see what has to be added to 4 to get the first
term of 9. This will be 5. So the nth term will be 4n + 5.
Show the class the sequence 3, 8, 13, 18, and ask what the nth term is for this
sequence.
They should be able to identify the first part as 5n since 5 is added on each time.
Ask what must be added to 5 to get the first term of 3. The answer is 2. So the
nth term is given by 5n 2.
Put on the board the sequence 1, 4, 9, 16, 25, 36, . Then ask the class what
they notice about the differences between the consecutive terms.

2 HarperCollinsPublishers Ltd 2003


9B3LP_01.qxd 18/09/2003 15:17 Page 3

After some discussion, put on the board the first and second differences:
1 4 9 16 25 36
First differences 3 5 7 9 11
Second differences 2 2 2 2
Explain that when a sequence has the same second differences, it is a quadratic
sequence. That is, a sequence whose nth term contains n2. In the example
above, T(n) = n2.
Finally, run through the sequence T(n) = n(n + 2), and show that the second
differences are constant.
The class can now do Exercise 1A from Pupil Book 3.

Exercise 1A Answers

1 a 17, 21, 25 b 23, 28, 33 c 30, 37, 44 d 34, 44, 55 e 50, 66, 84
f 29, 40, 53
2 a 5, 7, 9, 11 b 1, 4, 7, 10 c 9, 13, 17, 21 d 2, 7, 12, 17 e 1, 4, 9, 16
f 2, 5, 10, 17 g 2, 6, 12, 20 h 8, 14, 22, 32
3 a 4n + 2 b 7n + 1 c 3 4n d 3n 18 e 0.2n + 2.2 f 0.3n + 1.4
n (2n + 1)
g h
(3n 1) (5n 1)
4 a 2n b 100
5 a 6, 12, 20, 30, 42 b 0, 0, 2, 6, 12 c 1, 2, 5, 10, 17 d 2, 4, 6, 14, 34
e 3, 6, 9, 12, 39
6 b Both are 2 c 2
7 b Both are 4 c 6 d 2A

Extension Answers
Key Words
There are many different solutions to each part. The following includes one example of
each:
a 1, 1.5, 2, 2.5, 3 b 0.25, 0.5, 0.75, 1, 1.25 c 2, 4, 6, 8, 10 sequence
d 5, 10, 15, 20, 25 e 2.5, 5, 7.5, 10, 12.5
nth term
quadratic
sequence
first difference
Plenary second
Discuss with the class the results of the investigation in Exercise 1A, especially if difference
any student has completed Question 7.
Homework

1 Write down the first four terms of each sequence whose nth term is given below.
a 3n + 1 b 4n 2 c n2 + 7 d n(n + 3) e (n + 3)(n 1)
2 Find the nth term of each of the following sequences.
a 5, 7, 9, 11, b 2, 5, 8, 11, c 1, 4, 9, 16, d 3, 6, 11, 18,
3 Find the nth term of each of the following sequences of fractions.
a 12 , 23 , 34 , 45, b 13, 25, 37, 49,
4 Find the nth term of each of the following sequences.
a 3.5, 5, 6.5, 8, 9.5, b 5.1, 7.2, 9.3, 11.4, c 3.6, 6.1, 8.6, 11.1,

Answers
1 a 4, 7, 10, 13 b 2, 6, 10, 14 c 8, 11, 16, 23 d 4, 10, 18, 28 e 0, 5, 12, 21
2 a 2n + 3 b 3n 1 c n2 d n2 + 2
n n
3 a b
(n + 1) (2n + 1)
4 a 1.5n + 2 b 2.1n + 3 c 2.5n + 1.1

HarperCollinsPublishers Ltd 2003 3


9B3LP_01.qxd 18/09/2003 15:17 Page 4

LESSON Framework objectives Pattern spotting


Generate terms of a sequence using term-to-term and position-to-term definitions of
1.2 the sequence, on paper and using ICT.
Generate sequences from practical contexts and write an expression to describe the
nth term of an arithmetic sequence.

Oral and mental starter


Draw on the board three measuring jugs labelled 1 gallon, 3 pints and 5 pints.
Tell the class that the gallon jug is full.
Ask if anyone knows how many pints there are in a gallon. There are 8 pints.
Working in pairs, ask which pair can work out, using just the three jugs, how to
divide the liquid into two equal measures of half a gallon (4 pints).
The solution is as follows:
From the full gallon jug, fill the 5 pint jug, leaving a measure of 3 pints in the
gallon jug.
From the 5 pint jug, fill the 3 pint jug leaving 3 pints, 3 pints and 2 pints.
Pour all of the 3 pint jug into the gallon jug, giving 6 pints and 2 pints.
Pour the 2 pints from the 5 pint jug into the 3 pint jug, leaving 6 pints and
2 pints.
From the gallon jug, fill the 5 pint jug, leaving 1 pint, 2 pints and 5 pints.
From the 5 pint jug, fill up the 3 pint jug, leaving 1 pint, 3 pints and 4 pints.
Now just pour the 3 pint jug into the gallon jug to leave 4 pints in the gallon
jug and 4 pints in the 5 pint jug.

Main lesson activity


Draw on the board a circle with a chord (any straight line from one part of the
circumference to another).
Ask how many regions there are in the circle. (There are two.)
Now draw another chord in the circle, intersecting the first one, and ask: How
many regions are in the circle now? (There are four.)
Build up a table of results while doing this, showing the number of lines and the
number of regions.
Ask the class if they can tell you the possible maximum number of regions when
another chord is drawn.
Many might suggest six due to the sequence starting 2, 4, . Put the suggestions
on the board.
Now draw in the chord to intersect both chords already in the circle. Count with
the class the number of regions. There are seven regions. Put this in the table.
Now ask again: What is the maximum number of regions if I draw another
chord in the circle?
Some may spot the pattern, which gives 11. If so, then get them to explain the
pattern to the rest of the class. Show that this is true. The explanation is that 2 is
added, then 3, then 4 and so on.
Now ask if anyone can describe the term-to-term rule. Encourage class
discussion here, and clarity of explanation. There may be a few good
suggestions, all different from each other but still correct.
The rule is:
T(n) = Term + n
where the build-up is from the term immediately before.
Explain this terminology: T(n) is the nth term and Term is the term immediately
before it.
The class can now do Exercise 1B from Pupil Book 3.

4 HarperCollinsPublishers Ltd 2003


9B3LP_01.qxd 18/09/2003 15:17 Page 5

Exercise 1B Answers

1 a 10 c 15, 21 e T(n) = Term + (n 1)


2 a You probably predicted 10 b Probably not, for there are only nine lines
c 12, 15 e T(1) = 0 , T(2) = 1, T(3) = 3, after this T(n) = Term + 3
3 a 20 c 27, 35 e T(n) = Term + n
4 Two-hundred-and-ten pin bowling

Extension Answers

21(n2 + 3n + 6)

Plenary Key Words


Discuss with the class how good their predictions have been and whether they
got better as the lesson went on. prediction
Explain that it is good to try to make a prediction, as this means they are actually
looking at the pattern, which should lead to a refinement in the rule being
looked for.
To get results from complicated situations, it is very often useful to create simple
diagrams in order to look at the pattern.
Homework

Look at the following diagrams.

1 2 3

Diagram 1 2 3 4 5 6
Crosses 1 5 13
a Before drawing a diagram, can you predict, from the table, the number of crosses which are in
Diagram 4?
b Draw Diagram 4, and count the number of crosses there are. Were you right?
c Now predict the number of crosses for Diagrams 5 and 6.
d Check your results for part c by Drawing diagrams 5 and 6.
e Write down the term-to-term rule for the sequence of crosses. (Hint 4 = 22, 8 = 23)

Answers
a You will get the following results.

Diagram 1 2 3 4 5 6
Crosses 1 5 13 29 61 125
e T(n) = Term + 2n

HarperCollinsPublishers Ltd 2003 5


9B3LP_01.qxd 18/09/2003 15:17 Page 6

LESSON Framework objectives Functions


Find the inverse of a linear function.
1.3
Oral and mental starter
Ask the question: Is 1234 5678 = 7 006 652?
Is there a quick way of checking without doing the whole multiplication?
One check is to look at the product of the last digit of each number: 4 8 = 32.
The end digit of this number must be the end digit of the original sum. Here they
are both 2, so it might be correct.
Another check is to do the digit sum scan. That is, add the digits of 1234, 5678
and the predicted answer, 7 006 652, which gives 10, 26 and 26.
Add the digits of any of these three numbers greater than 9, here giving 1, 8
and 8.
Now check that the product of the first two values is the same as the third value.
Here they are the same both 8 so the answer might be correct.
Try this out: Is 314 783 = 245 762?
The digit sums come to 8, 9 and 8.
The product, 8 9 = 72, has digit total 9. This is different from the last digit sum,
so the prediction is not correct.
Let the class try this out with a calculator to convince them that this procedure
always works.

Main lesson activity


Put on the board x 4x and ask the class whether they remember what this is.
They should tell you that it is the function x maps to 4x. Ensure that the class use
the correct terminology.
Put in a column underneath the numbers x 1, 2, 3, 4 and ask if anyone can
tell you what each number will map to. You should get the response:
1 4, 2 8, 3 12, 4 16
Now continue the diagram by making each term map back to itself. That is:
1 4 1, 2 8 2, 3 12 3, 4 16 4
Ask the class if they can tell you what function will map each value back to
where it started. You may need to hide the original column of numbers in order
to focus only on the other two columns.
You want to get the response x 41 x.
Tell them that this is the inverse function of x 4x. Explain that the term
inverse means here doing the opposite process to return to the original values.
Ask the class: What is the inverse function of x 5x? You may need to go
through this in the same way which you did for x 4x. Ask them to work out
that the inverse function of x 5x is x 15 x.
Discuss what is happening with the above inversion. That is, division is the
inverse of multiplication and vice versa. Ask: When we see a pair of inverse
functions, are they both inverses of each other? You may need to refer to both
x 4x and x 5x in order to show that this is true.
Now put on the board the function x x + 3, and ask if anyone can propose
what its inverse will be. Discuss each proposal and ask for a reason why the
proposal was made.
You need to lead the students to the inverse being x x 3.
Discuss with them the inverse of x x + 7, which is x x 7.
Now put on the board the function x 4x + 3 and ask for its inverse.
This is not easy to see, so you will need to break down the function into its two
steps of 4 and + 3. Show this in flow diagram notation as:
x 4 +3 4x + 3

6 HarperCollinsPublishers Ltd 2003


9B3LP_01.qxd 18/09/2003 15:17 Page 7

Given that the inverse is the opposite process which returns the mapped values to their original values, the flow
diagram needs to be viewed in reverse. Reverse the arrows and start with x at the right-hand side, to give:
( x 3)
4 3 x
4
Remind the class that the inverse of is , and of + is . This gives the inverse function:
( x 3)
x
4
Show that this is the inverse function of x 4x + 3 by choosing a starting set of numbers, say, 1, 2, 3 and 4.
The class can now do Exercise 1C from Pupil Book 3.

Exercise 1C Answers

1 a x 12 x b x 15 x c x x 6 d x x 1 e x x + 3 f x 5x
( x 3) ( x 1) ( x + 3) ( x + 2) ( x 7)
2 a x b x c x d x e x
2 3 4 5 4
(x + 5)
f x
6
3 Two different types of example are:
12
i x 10 x 1 9 1 ii x 1 12 1
x
282 262
373 343
( x 6) ( x + 12)
4 a x b x c x 4x 3 d x 5x + 2 e x 2( x 3)
2 3
f x 2( x + 7)
5 a {2, 4, 6, 8, 10} b 2, 4, 6, 8, 10 c Yes
9 The lines are symmetrical about the line y = x

Plenary Key Words


Ask: What is the inverse of multiplication? (Division.)
inverse
Ask: What is the inverse of addition? (Subtraction.)
Ask: What is the inverse of squaring?(Taking the square root.)
Discuss the problems that this last inverse has. For example, the square of a
negative number is the same as the square of its positive value. So what about
the inverse?
Homework

1 Write down the inverse of each of the following functions.


x
a x 3x b xx+8 c x6+x d x e x 2x + 1 f x 4x + 3 g x 3x 5
2
2 Write down two different types of inverse function and show that they are self inverse functions.
3 Write down the inverse of each of the following functions.
(6 + x)
a x 3(x + 5) b x 12 ( x + 5) c x
4
4 a On a pair of axes, draw the graph of the function x 2x + 3.
b On the same pair of axes, draw the graph of the inverse of x 2x + 3.
c Comment on the symmetries of the graphs.

Answers
( x 1) ( x 3) ( x + 5)
1 a x 13 x b xx8 c xx6 d x 2x e x f x g
2 4 3
2 There will be a variety of different correct answers
3 a x 13 x 5 b x 2x 5 c x 4x 6
4 c The graphs are reflections of each other in the line y = x

HarperCollinsPublishers Ltd 2003 7


9B3LP_01.qxd 18/09/2003 15:17 Page 8

LESSON Framework objectives Graphs


Construct functions arising from real-life problems and plot their corresponding
1.4 graphs.

Oral and mental starter


Ask: Who knows how many miles are equivalent to 8 km? (5 miles)
Use this fact to ask quick-fire questions about equivalences of the following:
Kilometres 16 24 32 40 64 80 96
Miles 10 15 20 25 40 50 60
Discuss how they need to think: How many 8s? Then multiply that by 5.
Now ask for approximations, such as 10 kilometres and 60 kilometres. Again, look for approximations of
eights. So, 10 km is just over eight, which will make it just over 5 miles. For an approximation, call it
6 miles. For 60 kilometres, divide by 8, giving 712, which gives 5 712 = 35 + 212 = 3712. As this is an
approximation, round to 38 miles.
Finish off with the class trying mentally to convert approximately each of the following:
Kilometres 20 35 50 70 90 100 200
Miles 13 22 31 44 56 63 125
As the concern is to find approximations, any answer which is close to the correct mileage will be acceptable.
The main intention is to practise mental division by 8 and multiplication by 5.

Main lesson activity


Draw on the board a pair of axes with the horizontal axis labelled Time and the vertical labelled Distance.
Ask: What might the graph look like if it were representing a car being driven at a steady speed?(Straight line.)
You may want to discuss that a steeper line represents a faster speed but you will need to create values for the
graph in order to show this.
Draw another pair of axes on the board and ask: What shape would the graph have if the car were slowing
down?
It would be a curve whose steepness becomes less and less until the curve is horizontal.
Now draw a third pair of axes on the board and ask what shape the graph would be for a car accelerating from
standstill to a steady speed.
This will show a different type of curve with a gradually increasing gradient which eventually becomes a
straight line (but not horizontal).
Discuss with the class the fact that graphs illustrate typical speeds. In real life, however, there would be many
changes in the speed of a car, resulting in a range of different graphs from the ones drawn.
Tell the class how useful graphs can be and that they can hold a lot of information.
Look at the graph in Pupil Book 3, page 10, which represents a race between three boys. Ask the class to tell
you the story behind the graph.
The class can now do Exercise 1D from Pupil Book 3.

Extension Answers

a 70 b 59 million
Population (millions)

60
50
40
30
20
10
0
01

21

41

61

81

01

21

41

61

81

01
18

18

18

18

18

19

19

19

19

19

20

Year

8 HarperCollinsPublishers Ltd 2003


9B3LP_01.qxd 18/09/2003 15:17 Page 9

Exercise 1D Answers

1 a 300 m b 75 m c 42 seconds d D: 40 seconds, E: 60 seconds


e Rocket D travelled a greater distance vertically. The graph is not calibrated for horizontal distance
2 a The cold tap was turned on, increasing the rate of flow into the bath b After 4 minutes
c The more water in the bath, the quicker it will flow out, gradually slowing down d 3 minutes
3 a b c d

4 a C b A c B
5 a C b A c B
6 a b c d e

Plenary Key Words


Draw on the board a pair of axes labelled Time on the horizontal axis and
axes
Distance on the vertical axis.
Ask a student to draw on the axes a graph representing his/her journey to school gradient
that day. acceleration
Discuss this graph and whether it actually shows the variations in speed, the
stationary times, etc.
If time permits, ask a student to sketch a graph of an aircraft journey from
London to Amsterdam.
Homework

1 Sketch graphs to show how the depth of water varies with time when water drips steadily into the
following containers.

a b c

2 Sketch distancetime graphs to illustrate each of the following situations.


a A car accelerating away from traffic lights.
b A train slowing down to a standstill in a railway station.
c A car travelling at a steady speed and then having to accelerate to overtake another vehicle
before slowing down to travel at the same steady speed again.
3 Sketch a graph to show the depth of water in a bath where it is filled initially with just hot water,
then the cold water is also turned on. After 2 minutes, a child gets into the bath, splashes about for
5 minutes before getting out, and pulling out the plug. It takes 6 minutes for the water to drain
away.

Answers
1 a b c 2
a b c
d d d

t t t

3 The graph may look something like this:


d

HarperCollinsPublishers Ltd 2003 9


9B3LP_01.qxd 18/09/2003 15:17 Page 10

LESSON Framework objectives Limits of sequences


Generate terms of a sequence using term-to-term and position-to-term definitions of
1.5 the sequence.

Oral and mental starter


Ask: Who can multiply 15 by 13 mentally? (195)
Should anyone be able to do this, ask them to explain how they arrived at the
answer. If a different method from what will be given in the lesson is used, take
it as an alternative method.
If no one is able to do this mentally, or as an alternative from the explanation
offered, explain that you can split the sum into two parts.
As 5 3 = 15, a multiplication can be split when multiplying by 15.
For example, to find 15 13, multiply 13 by 3 to get 39 and then multiply 39
by 5.
There are various ways to multiply 39 by 5. One way is to multiply 39 by 10 and
then halve the result. The number needed is half of 390. Halve this in two parts:
half of 300 + half of 90 = 150 + 45 = 195.
Ask the class to practise multiplying 16, 21, 34, 42 and 57 by 15. The table
below can be used to check the results.
Number 16 21 34 42 57
Number 15 240 315 510 630 855

Main lesson activity


Put on the board: Divide by 5 and add 4.
Tell the class that this is a rule for creating a sequence and that this is the term-
to-term rule.
Ask someone to give you a number between 0 and 100. Use this to start the
sequence. If you wish to have more control over the numbers, then choose your
own starting number, say 1.
Using the term-to-term rule, this will generate:
1, 4.2, 4.84, 4.968, 4.9936, 4.99 872, 4.999 744, 4.999 949
Ask the class: Do you notice anything about the numbers? They should spot
that the terms are getting closer and closer to 5.
The class will need calculators to do this, or they could use a spreadsheet if
available. If using a spreadsheet, they will need to know how to set up a formula
and be able to copy it from one cell to another.
The class can now do Exercise 1E from Pupil Book 3.

Exercise 1E Answers

1 a 1, 3.5, 4.75, 5.375, 5.6875, 5.843 75, 5.921 875, 5.960 937 5, 5.980 468 75,
5.990 234 375, 5.995 117 188, 5.997 558 594
b 6 c Sequence always gets closer and closer to 6
2 a 1, 4.5, 6.25, 7.125, 7.5625, 7.78 125,
b 8 c Sequence always gets closer and closer to 8
3 a 1, 5.5, 7.75, 8.875, 9.4375, 9.71 875, 9.859 375,
b 10 c Sequence always gets closer and closer to 10
4 a 12
5 Gets closer to 4.5
6 Gets closer to 6
7 Gets closer to 7.5
8 Gets closer to 9

10 HarperCollinsPublishers Ltd 2003


9B3LP_01.qxd 18/09/2003 15:17 Page 11

Extension Answers

AB
Sequence always gets closer to .
(A 1)

SATs Answers

1 a Pattern number Number of grey tiles Number of white tiles


5 6 10
16 17 32

b Pattern number Expression for Expression for


number of grey tiles number of white tiles
n n+1 2n
c 3n + 1 d 5n + 4
n
2 a b ii c 25, 10
3, 17
4 d i
(2n + 1)

Plenary Key Words


Put on the board the term-to-term rule Divide by 2 and add 10.

sequence
Ask: Can anyone suggest what value this sequence will get closer and closer to?
Why do you think that? term-to-term
Students should be using the results from Questions 1 to 4, Exercise 1E, to help
them to see a simple link, and that this sequence will get closer to 20.
Now change the term-to-term rule to Divide by 3 and add 10, and ask the
same question.
Using their results from Questions 5 to 8, the class may need help to see the link
to multiplying the two numbers and halving the result to get the answer. Here,
this gives the result of 15.
Homework

1 A sequence starting at 1 has the term-to-term rule Add 3 and divide by 2.


a Find the first 10 terms generated by this sequence.
b To what value does this sequence get closer and closer?
c Use the same term-to-term rule with different starting numbers. What do you notice?
2 Repeat Question 1, but change the term-to-term rule to Add 4 and divide by 2.
3 What would you expect the sequence to do if you used the term-to-term rule Add 7 and divide
by 2?
4 What will the sequence get closer to using the term-to-term rule Add A and divide by 2?
5 Investigate the term-to-term rule Add A and divide by 3.

Answers
1 b 3 c Always gets closer to 3
2 b 4 c Always gets closer to 4
3 Always gets closer to 7
4 A
A
5 The terms in the sequence will get closer to .
2

HarperCollinsPublishers Ltd 2003 11


9B3LP_02.qxd 18/09/2003 15:18 Page 12

CHAPTER
2 Number 1
LESSON Framework objectives The four rules governing fractions
Use efficient methods to add, subtract, multiply and divide fractions. Cancel
2.1 common factors before multiplying and dividing.

Oral and mental starter


Use a target board such as the one shown on the right.
Start by asking the students, as a group or individually, to give the first five 2 5 7 8
multiples of various numbers. 24 15 18 3
Once the idea of the multiple is well established, ask for the lowest common multiple
(LCM) of a pairs of numbers. (It may be necessary to remind students of the definition.) 6 9 27 14
Continue for as long as necessary. 12 20 10 25

Main lesson activity


This is essentially a lesson on the four rules governing fractions.
The students will have met already the addition and subtraction of fractions, so they just need reminding of the
basic rules.
Ask the class to work out 259 + 315 7 .
Outline the two methods for solving this addition. First, change to improper (top-heavy) fractions and proceed
as follows:
259 + 315 7 = 23 52
9 + 15

= 45 + 156
115
=
45 271
45 = 645 1
Second, separate the whole numbers from the fractions:
259 + 3157 = 2 + 3 + 59 + 15 7
25 21
= 5 + 45 + 45

= 5 + 46
45
= 5 + 145 1 = 645 1
Discuss the comparative advantages and disadvantages of each method. For example, the first method involves
larger numbers.
Now ask the students how to work out 256 125.
Show them both methods.
256 125 = 17 6 75
85 42 43
= 113
= 30 30
= 30
30
5 2 5 2
26 15 = 2 1 + 6 5

25
12
= 1 + 30
30
= 1 + 30 = 113
13

30
Ensure that the students understand the fact that the whole numbers are subtracted, the result of which is added
to the outcome of subtracting the fractions.
This is the first time that the students will have met multiplication and division of fractions.
One way to introduce this topic is to use calculators to investigate the answers to products such as 15 35, 57 14
and 43 38 . The students will see the rule very quickly.
Now repeat with 12 45 , 34 89 and 15 4 38 . Ask why the rule doesnt appear to work. Discuss cancelling in the initial
product and in the answer.
Explain that it is better to cancel the initial fractions, as this makes the calculations easier and means that the
answer does not need to be cancelled down. Demonstrate with:
41 25 5 31 5
3 7 2 =
9 28 10 42
Now ask for the answer to 215 178. Explain how this is done, namely:
11 15 3 33
215 178 = = = 418
5 1 8 8

12 HarperCollinsPublishers Ltd 2003


9B3LP_02.qxd 18/09/2003 15:18 Page 13

Make sure students know that the mixed numbers cannot be separated as in addition or subtraction. They must
be converted to improper (top-heavy) fractions.
Division is a little harder to see. If calculators are used to investigate problems such as 27 13 , 34 45 and 23 89 ,
some students may see the method.
Most are unlikely to see that it requires turning the dividing fraction upside down and multiplying by it.
Demonstrate this with the above examples.
This is an easy method to use but not an easy one to understand. It can best be explained by examples such as
how many halves in 7? The answer is, of course, the same as multiplying by 2.
Now ask for the answer to 223 195. Explain how this is done, namely:
8 14 84 93 12
223 159 = = 1 7 = = 175
3 9 3 14 7
Do more examples if necessary.
The class can now do Exercise 2A from Pupil Book 3.

Exercise 2A Answers

1 a 11
2 12 b 430 17
c 311 d 356 e 519
15 f 713 g 817
20 h 1124
18 1
2 a 1
212 b 330 7 c 115 1 d 216 e 120 13
f 323 g 417
18 h 223

24
3 a 11
128 b 6326
c 20 13
d 28 9 e 96 17
f 75 22

2
b 169
c 28 5 1
d 6 e 10 3 1
f 2 g 11 3
h 15 i 3 j 14
2 2
4 a 15
2 4 1 f 5 g 15 h 625 i 425 j 1110
1 4 4 7
5 a 35 b 85 c 4 d 24 e 121
6 a 4 b 119 c 114 d 18 7 e 223 f 23 g 16
49 h 1 1
12 i 5

6 j 2

3
7 a 2 b 216 3 c 2 d 124 1 e 115 f 12
19 g 712 h 516
19 1 i 22 7 j 423
2
9 cm2 9 2432 9 5
cm2 10 46 = 4 lengths 11 25 m 2
8

Key Words
Extension Answers

Magic number is 1
2
15 35 4
15 convert
7 13 15 equivalent
15
fraction
25 1 8
15 15
mixed number
top-heavy
fraction
improper
fraction
Plenary lowest common
Write the following problem on the board: 415 1012 38 . multiple
Work through it, cancelling whenever possible. The answer is 20 3 . cancelling
Repeat for 18 43 13
1 1 2 3 3 5 3 ) (answer: 112 ).
1
(answer: 4) and ( 8 + 6 ) ( 4 + 18
Homework

1 Convert each of the following pairs of fractions to equivalent fractions with a common denominator.
Then work out each answer, cancelling down and/or writing as a mixed number if appropriate.
a 225 + 214 b 223 + 118 c 258 112
5 5 134
d 312
2 Work out each of the following. Cancel before multiplying when possible.
a 16 38 b 23 34 c 29 16
3 d 415 137 e 238 135
3 Work out each of the following. Cancel at the multiplication stage when possible.
a 14 13 b 3
16 9
14 c 16 13 d 258 16
7 e 235 10
3

Answers
1 a 413 b 319
20 5 d 123
c 124
24
2 a 16 b 2 c 24 d 6 e 345
1 1
1

3 a 34 b 24
7 c 12 d 6 e 823

HarperCollinsPublishers Ltd 2003 13


9B3LP_02.qxd 18/09/2003 15:18 Page 14

LESSON Framework objectives Percentages and compound interest


Recognise when fractions or percentages are needed to compare proportions.
2.2 Solve problems involving percentage changes.

Oral and mental starter


Use a target board such as the one shown on the right.

5% 10% 22% 13%
Ask the students for the equivalent multiplier when there is a percentage
increase or decrease. For example, 1.05 for the multiplier for a 5% increase and 16% 25% 14% 35%
0.95 for a 5% decrease.

20% 17% 6% 15%
Go round the class picking students at random and asking for the appropriate
multipliers for an increase and/or a decrease for percentages on the board. 8% 12% 712% 2%

Main lesson activity


Start by asking the students what they know about interest and how it works.
Then move on to compound interest.
Emphasise the basis of compound interest, namely: an amount of money (the
principal) is invested at an annual percentage rate(R%), and over a period of
years the value increases by R% each year.
Make sure that the students understand that the interest for each year is added to
the principal of the previous year to give the new principal for the following
year.
Work through an example. First, use the method of working out the yearly
interest and adding it on for each year; then use a multiplier. For example,
calculate how much 300 will earn when invested for 3 years at 4% interest per
annum.
First method Increase and addition
After first year: 4% of 300 = 12. So, at end of first year, you have
300 + 12 = 312.
After second year: 4% of 312 = 12.48. So, at end of second year, you
have 312 + 12.48 = 324.48.
After third year: 4% of 324.48 = 12.98. So, at end of third year, you
have 324.48 + 12.98 = 337.46.
Second method Use a multiplier
After first year: 300 1.04 = 312
After second year: 312 1.04 = 324.48
After third year: 324.48 1.04 = 337.46
Point out that the last value is rounded to the nearest penny.
Most students are likely to prefer the second method. However, some will prefer
the structured nature of the first method.
Demonstrate the use of powers on a calculator. For example, the above result is
identical to 300 (1.04)3. This gives 337.4592, which has to be rounded to the
nearest penny.
Discuss the advantage (quick) and the disadvantage (any keying errors mean no
working from which to gain partial credit).
Repeat with an example that decreases each year. For example, an ant colony
has 30 000 ants. They start to die off at the rate of 22% per day. How many ants
will be left after 7 days?
The first method is too lengthy. The second method (using the constant
multiplier of 0.78) gives:
30 000, 23 400, 18 252, 14 236.56, 11 104.52, 8661.52, 6755.99,
5269.67
Hence, there are 5270 ants left at the end of 7 days.
The class can now do Exercise 2B From Pupil Book 3.

14 HarperCollinsPublishers Ltd 2003


9B3LP_02.qxd 18/09/2003 15:18 Page 15

Exercise 2B Answers

1 a 1.12 b 0.95 c 0.92 d 1.07 e 0.96 f 1.02 g 1.032 h 1.025


i 0.85 j 1.06 k 0.974 l 1.005 m 0.76 n 0.93 o 1.175
2 a 216.49 b 3740.06 c 214.90 d 19 348.42 e 80.77
3 a 2348.27 b 219.15 c 1334.49
4 a 33 662 b 18 837
5 6 days
6 12 days

Extension Answers

a 547.50 b 47.54 c 47.30


d 47.07, 46.83, 46.60, 46.37, 46.14, 45.91, 45.68, 45.45, 45.23, 45
e 555.12

Plenary Key Words


Discuss the advantages/disadvantages of using a multiplier and powers
compound
compared with other methods.
If the class is able enough, establish the formula for compound interest: interest
Percentage rate Time period multiplier
(
Total at end of period = Initial amount 1
100 )
So, take as an example 45 000 units decreasing at 6% each day for 3 days,
which gives:
6 3
( )
Total after 3 days = 45 000 1 = 37 376.28 units
100
Homework

1 How much would you have in the bank if you invest as follows?
a 450 at 3% interest per annum for 4 years.
b 6000 at 4.5% interest per annum for 7 years.
2 Stocks and shares can decrease in value as well as increase. How much would your stocks and
shares be worth if you had invested as follows?
a 1000, which lost 14% each year for 3 years.
b 750, which lost 5.2% each year for 5 years.

Answers
1 a 506.48 b 8165.17
2 a 636.06 b 574.25

HarperCollinsPublishers Ltd 2003 15


9B3LP_02.qxd 18/09/2003 15:18 Page 16

LESSON Framework objectives Reverse percentages and percentage


change
2.3 Use proportional reasoning to solve a problem, choosing the correct numbers to
take as 100%, or as a whole.

Oral and mental starter


Use a target board such as the one shown on the right.

1.05 0.90 1.22 0.87 0.6
Ask the students for the equivalent percentage increase or decrease for each
multiplier. For example, 1.05 represents a 5% increase. 1.16 0.75 1.14 1.35 1.3
Go round the class picking students at random and asking for the appropriate
0.8 0.81 1.06 0.65 0.02
percentage increase and/or decrease for the multipliers on the board.
0.92 1.12 1.175 0.98 1.88

Main lesson activity


Reverse percentages and how to choose the appropriate quantity to take as
100% are now going to be covered.
Ask the students for the original amount when the new value, after a 30%
increase, is 195.
Many may suggest that it is the same as a 30% decrease of 195, which is
136.50. However, when the latter amount is increased by 30%, the answer
is 177.45.
Explain that 195 represents not 100% but 130%.
Solve the problem using first the unitary method, and then a multiplier.

Method 1 Unitary method


This involves finding a single unit value, which in this case is the value
of 1%.
195 represents 130%
1.50 represents 1% (Dividing both sides by 130.)
150 represents 100% (Multiplying both sides by 100.)

Method 2 Use a multiplier


A 30% increase is represented by the multiplier 1.30.
Hence, divide 195 by 1.3 to find the original amount. This gives:
195 1.3 = 150

Discuss the disadvantages/advantages of each method. The students will


probably prefer to use the multiplier, as it is easier to work out and has fewer
steps.
Repeat with other examples if necessary.
Now ask the students: What is the percentage increase from 550 to 704?
Do the calculation on the board:
Actual increase = 704 550 = 154
154
Percentage increase = 100 = 28%
550
Emphasise that 550 is the original amount.
The class can now do Exercise 2C from Pupil Book 3.

16 HarperCollinsPublishers Ltd 2003


9B3LP_02.qxd 18/09/2003 15:18 Page 17

Exercise 2C Answers

1
1200 g
2
260
3
Camera 190, heater 60, printer 70, washer 250, sofa 450, computer 1800
4
128
5
50
6
41.4%
7
4.2%
8 946 154
9 15
10 South-east England 24%, Scotland 16%, Yorkshire 15%, East Anglia 36%

Extension Answers

a 19.6% b 11.4% c 12.7% d 79.6% e 2.2%

Plenary Key Words


Introduce the magic road sign, which is a mnemonic to help to recall the
different combinations used in the calculation of percentage change and reverse
reverse
percentages. percentage
unitary method
That is: multiplier
New value = Original value Multiplier power
Multiplier = New value Original value
Original value = New value Multiplier New
value

Original
Multiplier
value
Homework

1 A packet of biscuits claims to be 24% bigger! It now contains 26 biscuits. How many did it have
before the increase?
2 After a 10% price decrease, a hi-fi system now costs 288. How much was it before the decrease?
3 This table shows the cost of some items after 1712 % VAT has been added. Work out the cost of each
item before VAT.

Item Cost inc VAT Item Cost inc VAT


Radio 112.80 Cooker 329
Table 131.60 Bed 376

4 A pair of designer jeans is on sale at 96, which is 60% of its original price. What was the original
price?
5 A pair of boots, originally priced at 60, were reduced to 36 in a sale. What was the percentage
reduction in the price of the boots?

Answers
1 21 biscuits
2 320
3 Radio 96, table 112, cooker 280, bed 320
4 160
5 40%

HarperCollinsPublishers Ltd 2003 17


9B3LP_02.qxd 18/09/2003 15:18 Page 18

LESSON Framework objectives Direct and inverse proportion


Reduce a ratio to its simplest form. Use the unitary method to solve simple word
2.4 problems involving ratio, direct proportion and inverse proportion.

Oral and mental starter


Give the class a ratio, such as 2 : 5, and ask them to find the missing value in
the equivalent ratio : 15 (6 : 15). Repeat with, for example, 10 : (10 : 25),
: 35 (14 : 35).
Students could use number fans or mini white boards on which to write their
answers.
Repeat with other ratios.

Main lesson activity


Ask the class to solve the following problem:
If five exercise books have a total of 175 pages, how many pages will four
exercise books have?
Students may have an intuitive idea of the answer (140 pages) but outline the
unitary method. Five books have 175 pages so, one book has 35 pages.
Therefore, four books have 4 35 = 140 pages.
Alternatively this can be considered as a ratio, giving 5 : 175 = 1 : 35 = 4 : 140.
(Note: this relates to the starter activity.)
Repeat with the following examples.
Nine canteen tables can seat 72 people. How many people can sit at eight
canteen tables?
If nine tables seat 72 people, one table would seat eight people. Therefore,
eight tables will seat 8 8 = 64 people.
Alternatively, 9 : 72 =1 : 8 = 8 : 64.
A bus with 20 passengers on board takes 10 minutes to travel from Silkstone
to Barnsley. How long will the same bus with 10 people on board take to do
the same journey?
The number of passengers does not affect the speed of the bus. So, the time
taken will be exactly the same, 10 minutes.
The following examples use inverse proportion. Students find this concept quite
difficult.
Ask the class how to do this problem.
Five men build a wall in 9 days. How long would it take six men?
The wall would take one man 5 9 = 45 days. Therefore, six men take
45 6 = 7.5 days.
Repeat with the following examples.
Three taps can fill a sink in 15 minutes. How long would five taps take to
fill the sink?
One tap would fill the sink in 3 15 = 45 minutes. Now, 5 9 = 45, so it
would take five taps 9 minutes.
When I travel at 60 mph, my car does 10 miles to a litre of petrol. How
many miles will I get per litre if I travel at 30 mph?
This is impossible to answer as the petrol consumption of a car is not just
dependent on speed.
The class can now do Exercise 2D from Pupil Book 3.

18 HarperCollinsPublishers Ltd 2003


9B3LP_02.qxd 18/09/2003 15:18 Page 19

Exercise 2D Answers

1 39.20 2 1 h 40 min 3 2 days 4 3 h 20 min 5 2.86 6 14 min


7 6 min 8 3 days 9 240 miles 10 40 min 11 12 days
12 a 22.4 cm b 4.375 km 13 a 144 miles b 4.58 14 48 min 15 216 min
16 a 12 s b 6 s 17 360 min 18 9 days 19 5 min 20 15 min 21 21 tins
22 2 h 24 min 23 2.5 cm 24 20 miles 25 36 h 26 9 days 27 232 tubes
28 180 miles 29 7.5 h 30 253 exam papers

Extension Answers

a 2d b 3d c 21 d d 3d e 8d f 81 d

Plenary Key Words


Put the following problem on the board: xy = 24.

ratio
Ask the class what values x and y could take. These could be written on the
board. unit
Now ask what would happen to y if x doubles? ( y halves) direct proportion
Similarly, what happens to x if y trebles? (x assumes one third of its first value) inverse
Ask what happens when x is divided by 4? ( y is multiplied by 4) proportion
Repeat with xy = 36 and other fractions.
If time allows, discuss the connection with the inverse proportion questions.
Homework

1 In 4 hours a man earns 45. How much does he earn in 5 hours?


2 A man walking one dog takes 20 minutes to walk one mile. How long will it take him to cover one
mile if he walks two dogs?
3 In a week, grass grows 21 mm. How much does it grow in 4 days?
4 Fifty litres of petrol costs 35. How much will 20 litres of petrol cost?
5 Eight men dig a ditch in 9 days. How long would six men take?
6 A camping party of three has enough food to last them 4 days. If another person joins the party, how
long will the food last?
7 At 6 an hour, Jack takes 16 hours to earn enough for a guitar. If he had earned 8 an hour, how
long would it have taken him to earn the money?
8 Three bell ringers ring a tune on 6 bells in 5 minutes. How long would four bell ringers take to ring
the same tune?

Answers
1 56.25 2 20 min 3 12 mm 4 14 5 12 days 6 3 days 7 12 h 8 5 min

HarperCollinsPublishers Ltd 2003 19


9B3LP_02.qxd 18/09/2003 15:18 Page 20

LESSON Framework objectives Ratio in area and volume


Understand the implications of enlargement for area and volume.
2.5
Oral and mental starter
Give the class practice on squares and cubes. They should know the squares up
to 15 15 and the cubes up to 5 5 5.
Do this as a mental test or ask students individually to give you answers to
questions such as: four cubed; the cube root of eight; nine squared, and so on.

Main lesson activity


This section of Pupil Book 3 opens with an investigation which is reproduced
below.
Let students work through the investigation.
Use the plenary (see next page) to bring the students together to check their
answers.
Investigation

12 cm

4 cm 18 cm

2 cm 6 cm C
3 cm B
A

2 cm 4 cm 12 cm

These three blocks are similar. This means that the ratio height : length : width is
the same for all three blocks.
a Work out the area of the front face of each block.
b Work out the volume of each block.
Work out each of the following ratios and write it in the form 1 : n.
c i Length of block A to length of block B.
ii Area of the front face of block A to area of the front face of block B.
iii Volume of block A to volume of block B.
d i Length of block A to length of block C.
ii Area of the front face of block A to area of the front face of block C.
iii Volume of block A to volume of block C.
e i Length of block B to length of block C.
ii Area of the front face of block B to area of the front face of block C.
iii Volume of block B to volume of block C.
Look at your answers to parts c, d and e. What do you notice?
Explain the connection between the ratio of the lengths, areas and volumes of
similar shapes.
After the investigation, the class can now do Exercise 2E from Pupil Book 3.

20 HarperCollinsPublishers Ltd 2003


9B3LP_02.qxd 18/09/2003 15:18 Page 21

Exercise 2E Answers

1 b i 1 : 3 ii 1 : 9 iii 9 5 = 45 cm2 c i 1 : 5 ii 1 : 25
iii 25 5 = 125 cm2
2 b i 1 : 4 ii 1 : 64 iii 64 6 = 384 cm3 c i 1 : 5 ii 1 : 125
iii 125 6 = 750 cm3
3 90 cm2
4 4 cm2
5 64 cm3
6 2 cm3
7 a 41 b 81
8 a 1 : 100 b 1 : 10 000 c 1 : 1 000 000

Extension Answers

a Height : radius in same ratio b i 12.57 cm2, 28.27 cm2, 78.54 cm2
ii 50.27 cm3, 169.65 cm3, 785.40 cm3 c i 1 : 2.25 ii 1 : 6.25 iii 1 : 3.375
iv 1 : 15.625 d i 1 : 9 ii 1 : 27

Plenary Key Words


The plenary can be done at any appropriate time and is used to ensure that
linear scale
every student has grasped the concept of linear, area and volume scale factors
and the connection between them. factor
Stop students working and discuss the results of the investigation. area scale factor
Make sure they understand that a linear increase by a factor of 2, say, means an volume scale
area increase by a factor of 4 and a volume increase by a factor of 8. factor
Generalise this as linear 1 : a, area 1 : a2, volume 1 : a3.
Students have difficulty with the concept of a scale factor. Ensure they
understand that it is the original length, area or volume which is multiplied by
the scale factor to get the new length, area or volume.
Homework

1 Two similar, plane shapes, A and B, have lengths in the ratio 1 : 4. The area of shape A is 10 cm2.
What is the area of shape B?
2 Two similar, plane shapes, P and Q, have lengths in the ratio 1 : 2. The area of shape Q is 100 cm2.
What is the area of shape P?
3 Two similar solids, C and D, have lengths in the ratio 1 : 3. The volume of solid C is 15 cm3.
What is the volume of solid D?
4 Two similar solids, R and S, have lengths in the ratio 1 : 2. The volume of solid S is 72 cm3.
What is the volume of solid R?

Answers
1 160 cm2 2 25 cm2 3 405 cm3 4 9 cm3

HarperCollinsPublishers Ltd 2003 21


9B3LP_02.qxd 18/09/2003 15:18 Page 22

LESSON Framework objectives Numbers between 0 and 1


Use the laws of arithmetic and inverse operations.
2.6 Understand the effects of multiplying and dividing by numbers between 0 and 1.

Oral and mental starter


This brief starter could be missed out.
Quickly recall the rules for multiplying and dividing with directed numbers. Do
this with a few examples, such as:
2 +3 = 6 4 1 = +4 5 0.5 = 10

Main lesson activity


This lesson is essentially an investigation. The class can work in groups or
individually. Keep the introduction as brief as possible.
The introduction and Example 2.15 from Pupil Book 3 are reproduced here.

3 2 1 0 1 2 3
A B C D

1 0 1

The special numbers 1, 0 and 1 divide the number line into four sets of
numbers: A, B, C and D.
A contains all the numbers less than 1. B contains all the numbers between 1
and 0. C contains all the numbers between 0 and 1 and D contains all the
numbers greater than 1.

Example 2.15 a What happens when a number from set A is multiplied by


a number from set D?
b What happens when a number from set B is divided by 1?

a Choose any number from set A, say 2. Choose any


number from set D, say +3. Multiply them together:
2 +3 = 6.
The answer belongs to set A.
Try other combinations of numbers from set A and set D.
For example:
4 +4 = 16 1.5 5 = 7.5 5 1.5 = 7.5
They all belong to set A. So, a number from set A
multiplied by a number from set D always gives a number
in set A.
b Pick numbers from set B and divide each one by 1. For
example:
0.4 1 = 0.4 23 1 = 23 0.03 1 = 0.03
The answers are the same as the values from set B. So,
they all give numbers in set B.
This example, or similar examples, could be worked through to start the
investigation.
The class can now do Exercise 2F from Pupil Book 3.

22 HarperCollinsPublishers Ltd 2003


9B3LP_02.qxd 18/09/2003 15:18 Page 23

Exercise 2F Answers

1 2
A 1 B 0 C 1 D A 1 B 0 C 1 D
A D D C/D 0 A/B A A A C/D D D A A A/B
1 D 1 C 0 B 1 A 1 C 1 D A 1 B
B C/D C C 0 B B A/B B C C C/D A/B B B
0 0 0 0 0 0 0 0 0 0 0 0 0 0 0
C A/B B B 0 C C C/D C B B A/B C/D C C
1 A 1 B 0 C 1 D 1 A 1 B C 1 D
D A A A/B 0 C/D D D D A/B A A D D C/D

3 a ii b iii c iii d ii
4 For example: a 8 1 = 8 b 7 1.2 = 8.4 c 9 0.2 = 45
d 6 0.5 = +3

Extension Answers

No firm conclusions can be reached. The results depend on the numbers chosen.

Plenary Key Words


The answers to the two tables (Exercise 2F, Questions 1 and 2) should be
counter-example
discussed with the whole class. Highlight particularly the significance of
multiplying and dividing by numbers between 0 and 1. generalise
Familiarise them with the term counter-example. conclude
deduce
Homework

Say which of these statements is true. If it is not true, give a counter-example.


a The square of a number between 0 and 1 is also between 0 and 1.
b The square of a number between 0 and 1 is also between 0 and 1.
c Dividing any number by a number between 0 and 1 always gives a bigger answer.
d Dividing any positive number by a number between 0 and 1 always gives a bigger answer.

Answers
a True b False, for example: (0.4)2 = 0.16 c False, for example 7 0.5 = 14, which is smaller d True

HarperCollinsPublishers Ltd 2003 23


9B3LP_02.qxd 18/09/2003 15:18 Page 24

LESSON Framework objectives Reciprocal of a number


Recognise and use reciprocals.
2.7
2.2 Know how to use the reciprocal key on a calculator.

Oral and mental starter


As preparation for the mental test in the SATs papers, this starter concentrates on
SATs style questions using powers. Ask each question twice and allow about 10
seconds for the students to answer.
1 What is nine squared?
2 What is the square root of 144?
3 The diagram shows a cube made from smaller cubes. [Draw this on the
board.] What is the volume of the big cube?
4 The length of one side of a square is 4 cm. What is the area of the square?
5 The area of a square is 64 cm2. What is the perimeter of the square?
6 x squared is 4. Which two values can x have?
7 Look at the inequality. [Write x2 <17 on the board.] What is the largest
integer value that x could be?
8 What number is five cubed?
9 The nth term of a sequence is n plus one all squared. [Write (n + 1)2 on
board.] What is the fifth term of the sequence?
10 What is the square root of nine-sixteenths?
It would be worthwhile discussing the techniques involved when giving the
answers.
It is useful to repeat this test within a few days to see whether scores improve.

Answers 1 81 2 12 3 8 (cm3) 4 16 cm2 5 32 cm 6 2 and 2


7 4 8 125 9 36 10 Three-quarters

Main lesson activity


Every student will need a calculator.
First, define the reciprocal of x as 1 x.
Give some examples, such as the reciprocal of 2 is 1 2 = 0.5, the reciprocal of
10 is 1 10 = 0.1.
Now ask the class to find the reciprocal key on their calculators.
This will be marked in various ways. For example, it may look like one of the
following:

Do some examples and get the class to practise the use of this key.
Some students may find it just as easy to divide the number into 1, rather than
use the reciprocal key.
Next, ask them to find the reciprocal of each of these numbers: 40 (1 40 =
0.025) and 0.8 (1 0.8 = 1.25).
Do more examples, if necessary.
In Exercise 2G there are two questions (3 and 4) which are investigations. These
could be done as a class activity.
The class can now do Exercise 2G from Pupil Book 3.

24 HarperCollinsPublishers Ltd 2003


9B3LP_02.qxd 18/09/2003 15:18 Page 25

Exercise 2G Answers

1 a 1, 0.5, 0.33 , 0.25, 0.2, 0.166 , 0.142 857 14 , 0.125, 0.111 ,


0.1, 0.0909 , 0.0833 , 0.076 923 0769 , 0.0714 285 71 , 0.0666 ,
0.0625, 0.0588 , 0.055 , 0.0526 , 0.05
b Reciprocals of 1, 2, 4, 5, 8, 10, 16, 20
2 a 0.0333 b 200 c 0.0125 d 800 e 0.0005 f 500 g 0.01
h 0.000 001 (106)
a b
3 Reciprocal of is
b a
4 a They are perpendicular to each other
b A, 2, 12; B, 1, 1; C, 32, 23; D, 14, 4; E, 3, 13
c When a pair of lines is perpendicular, their gradients are the negative reciprocals
of each other
5 a 2 b 2 Always true
6 No, you are not allowed to divide by zero.

Extension Answers

For any power n, the reciprocal of 2n is 2n

Plenary Key Words


Invite the class to give you the value of each power from 103. Then write them
on the board in the form of a table.
reciprocal
divide into
103 102 101 100 101 102 103
0.001 0.01 0.1 1 10 100 1000

Now ask them to find the reciprocals of these numbers.


Establish the relationship between positive powers, negative powers and
reciprocals. For example:
104 = 1 104 107 = 1 107 10a = 1 10a
Homework

1 a Find, as decimals, the reciprocals of all the integers from 21 to 25.


b Which of the reciprocals are recurring decimals?
2 Find the reciprocals of each of the following numbers. Round your answers if necessary.
a 50 b 0.004 c 60 d 0.625

Answers
1 a 0.047 619 , 0.045 45 , 0.043 47 , 0.041 66 , 0.04
b All except the reciprocal of 25 which terminates, and the reciprocal of 23 which neither recurs or terminates
2 a 0.02 b 250 c 0.016 66 d 1.6

HarperCollinsPublishers Ltd 2003 25


9B3LP_02.qxd 18/09/2003 15:18 Page 26

LESSON Framework objectives Rounding and estimation


Estimate calculations by rounding numbers to one significant figure and multiplying
2.8
2.2 or dividing mentally.

Oral and mental starter


Have a set of Follow-me cards dealing with approximations. A set of 20 are
given below. These should be given to pairs of students to allow discussion. It is
suggested that each pair be given a few moments to come up with an estimate
(the teacher can write each one on the board) and that jottings are allowed.

1 START. You are 0.52 0.62 2 I am approx. 0.3. You are 892 0.48.
3 I am approx. 450. You are 0.29 0.31 4 I am approx. 0.09. You are 58 72.
5 I am approx. 4200. You are 0.32 61 6 I am approx. 18. You are 312 0.32.
7 I am approx. 90. You are 217 53 8 I am approx. 10 000. You are 0.092 0.12
9 I am approx. 0.009. You are 6.2 0.72 10 I am approx. 4.2. You are 187 0.68.
11 I am approx. 140. You are 3.2 0.33 12 I am approx. 0.9. You are 39 51.
13 I am approx. 2000. You are (72)2 14 I am approx. 4900. You are 0.32 0.11
15 I am approx. 0.03. You are 504 189 16 I am approx. 100 000. You are 96 0.11.
17 I am approx. 10. You are (0.68)2 18 I am approx. 0.49. You are 92 89.
19 I am approx. 8100. You are 0.092 321 20 I am approx. 27. END.

Main lesson activity


Ask the class to round the following numbers as indicated: 368 to the nearest
100, 23.9 to the nearest 10, 0.0713 to one decimal place (dp).
Ask them whether the answers have anything in common.
They may notice or may need to be prompted to state that the answers all
have one digit apart from zero.
These are examples of rounding to one significant figure (1 sf).
This is a concept that many students find hard to grasp. Emphasise that there will
be only one digit apart from zeros.
Do more examples of rounding to one significant figure, such as:
3789 4000 0.265 0.3 0.25 0.3 0.198 0.2
0.636 0.6 0.0621 0.06 0.0764 0.08
Now give some examples of how rounding to one significant figure can be used
to estimate answers to calculations. For example:
320 398 300 400 = 120 000
0.092 476 0.09 500
= 0.9 50
= 9 5 = 45
(29 + 88) (2.3 0.053) (30 + 90) (2 0.05)
= 120 0.1
= 1200 1 = 1200
17 0.42 20 0.4
= 200 4 = 50
Do more examples, if necessary.
The class can now do Exercise 2H from Pupil Book 3.

26 HarperCollinsPublishers Ltd 2003


9B3LP_02.qxd 18/09/2003 15:18 Page 27

Exercise 2H Answers

1 a 600 b 0.3 c 0.07 d 100 e 0.3 f 0.09 g 300 h 0.4 i 0.7


j 0.6 k 900 l 1
2 a 80 000 b 1 500 000 c 4200 d 160 000 e 8100 f 0.18 g 0.063
h 0.04 i 100 j 140 k 240 l 420
3 a 6 b 3000 c 5 d 400 e 40 f 50 g 500 h 600 i 400 j 50
k 400 l 0.3
4 a 0.7 600 = 420 b 300 0.5 = 600 c 3000 0.6 = 5000
d 0.06 0.2 = 0.012 e (20 0.05) 0.4 = 2.5
f (0.04 + 0.06) (0.07 + 0.08) = 0.015 g (200 0.1) (800 0.02) = 800 000
h 0.5 (30 0.4) = 40 i 52 8 0.22 = 25 8 0.04 = 200 0.04 = 5000
j (20 0.4) (2 + 0.5) = 3.2
5 a 4000 b 10 000 c 250 000 d 0.001 e 20 f 1900

Extension Answers

a 350 b 4220 c 4200 d 0.619 e 0.62 f 300 g 4700 h 4700


i 0.079 j 978.3 k 978 l 980

SATs Answers

1 a 14.5% b 17 255 c 39.3%


d Not enough information: total number of police officers not given for both years
15

2 a 48% b 1 : 5.8 c 34
3 a 70 1.09; 70 0.9 is 90% of 70, 70 1.9 is 70 increased by 90%, 70 0.09 is
9% of 70 b 0.86
1 1 2 2
4 For example: Let t = 1, w = 1. Then + = 1 + 1 = 2 but = = 1, and 2 1
1 1 1+ 1 2
5 a 13 b 28.2%
c In 1961, birthrate in Scotland was lower than birthrate in Northern Ireland
6 45 7 t = 4 r = 7.5
8 2 : 3 9 21%

Plenary Key Words


Put the following on the board:
most significant
(14.7 3.9) (0.96 + 0.59)
Ask a student to round to 1 sf and approximate the answer. That is: digit
(10 4) (1 + 0.6) = 40 1.6 least significant
At this stage, the problem becomes too difficult to do mentally. digit
Ask whether it is possible to approximate differently. For example: approximate
(15 4) (1 + 0.5) = 60 1.5 = 40 significant figure
Point out that approximations do not have to be made to 1 sf when a more
sensible approximation is possible.
Try this out with 23.6 7.8 49.2. Rounding to 1 sf gives 20 8 50, but
rounding to 25 8 50 = 4 is easier to do mentally.
Homework

1 By rounding each value to one significant figure, estimate the answer to each of the following.
a 0.83 793 b 618 0.32 c 812 0.38
d 0.78 0.049 e (38 3.2) 0.487 f (2.7 + 6.3) (0.52 0.17)

Homework answers
1 a 0.8 800 = 640 b 600 0.3 = 2000 c 800 0.4 = 2000 d 0.8 0.05 = 0.04
e (40 3) 0.5 = 240 f (3 + 6) (0.5 0.2) = 2.7

HarperCollinsPublishers Ltd 2003 27


9B3LP_03.qxd 18/09/2003 15:18 Page 28

CHAPTER
3 Algebra 3
LESSON Framework objectives Simultaneous equations
Solve a pair of simultaneous equations by eliminating one variable.
3.1

Oral and mental starter


Ask the class for the largest multiple of 9 less than 100. (It is 99.)
Repeat this for the largest multiples less than 100 of these numbers:
7 (98) 8 (96) 13 (91) 3 (99)
Discuss how students found the answers. They would probably have added on from known facts or used trial
and improvement. Working from known facts is a good way to find an answer.
Now ask for the largest multiple of 7 under 300. Demonstrate the following method for finding it.
Start from 300 7, which can be seen to be about 40.
Calculate 7 40 = 280, add on sevens to get 287, then 294, which is the largest multiple (as it is only
6 less than 300).
Now ask individual students to find each of the following using a similar method.
Largest multiple of 6 less than 500 (498).
Largest multiple of 7 less than 400 (399).
Largest multiple of 8 less than 300 (296).
Largest multiple of 11 less than 700 (693).
Largest multiple of 15 less than 800 (795).

Main lesson activity


Put on the board the equation x + y = 8. Ask the class to give you some solutions to the equation. Put them all
on the board. Examples are: x = 5, y = 3; x = 1, y = 7; x = 0, y = 8; x = 1, y = 9.
Now put on the board the equation 3x y = 12. Ask the class to give you some solutions of this equation.
Again, put them all on the board. Examples are: x = 4, y = 0; x = 6, y = 6; x = 2, y = 6; x = 5, y = 3.
Each equation has many solutions, but how many solutions will fit both equations simultaneously?
There is only one such solution to these two equations: x = 5, y = 3.
Explain to the class that solving simultaneous equations is an important part of mathematics and there are three
main ways of finding the unique solution.
Today, you are going to show them the method of elimination.
Put the two equations on the board again in the traditional simultaneous mode:
x+y=8 (i)
3x y = 12 (ii)
Add the two equations in columns, to get:
4x = 20 (Notice that, +y + y = 0.)
x=5
Now substitute x = 5 into equation (i) to give:
5+y=8
y=3
Show the class that this is the same solution which they had before.
Explain that, since there are two identical terms apart from their signs, the two equations are added together to
eliminate this variable.
Next, put on the board another pair of simultaneous equations:
5x + 2y = 20 (i)
3x + 2y = 16 (ii)
Since there are two identical terms (2y), 2y can be eliminated by subtracting one equation from the other. This
can be either top from bottom or bottom from top. It really depends on avoiding a negative quantity in the
resulting equation.

28 HarperCollinsPublishers Ltd 2003


9B3LP_03.qxd 18/09/2003 15:18 Page 29

Hence, subtract equation (ii) from equation (i). This gives:


2x = 4 (Notice that 2y 2y = 0.)
x=2
Substitute x = 2 into equation (ii). (Chosen as it has the smallest numbers.)
6 + 2y = 16
2y = 10
y=5
Show here, how to check the solution by substituting both x and y values into equation (i). That gives:
5 2 + 2 5 = 20, which is correct for equation (i).
If the class require further examples to be worked through, use those on page 42 in Pupil Book 9.3.
The class can now do Exercise 3A from Pupil Book 3.

Exercise 3A Answers

1 x = 4, y = 1 2 x = 1, y = 4 3 x = 3, y = 1 4 x = 5, y = 2 5 x = 7, y = 1
6 x = 5, y = 3 7 x = 4, y = 2 8 x = 2, y = 4 9 x = 3, y = 5 10 x = 2, y = 3
11 x = 4, y = 3 12 x = 5, y = 4

Extension Answers

1 a 29p b 19p
2 1.10

Plenary Key Words


Ask the class: When do we add and when do we subtract a pair of simultaneous
simultaneous
equations?
Use this as a discussion starter, but do ensure that every member of the class equation
understands that subtraction is undertaken when there are two identical terms elimination
with the same sign, and that addition is undertaken when terms are identical but substitution
the signs are not.
Homework

1 4x + y = 14 2 6x + 3y = 33
2x + y = 8 2x + 3y = 21
3 3x + y = 10 4 5x + 2y = 22
8x y = 1 7x 2y = 2
5 5x 4y = 36 6 5x + 3y = 50
2x 4y = 6 9x 3y = 48

Answers
1 x = 3, y = 2 2 x = 3, y = 5 3 x = 1, y = 7 4 x = 2, y = 6 5 x = 10, y = 3.5 6 x = 7, y = 5

HarperCollinsPublishers Ltd 2003 29


9B3LP_03.qxd 18/09/2003 15:18 Page 30

LESSON Framework objectives Solving by substitution


Solve a pair of simultaneous linear equations by eliminating one variable.
3.2

Oral and mental starter


Tell the class they are going to play a quick game of 24.
Give them four numbers: 1, 2, 4 and 6. Ask who can make a sum out of these
numbers whose answer is 24. Once one sum has been made, make another.
Two possible answers to this game are:
(4 6) (2 1) (4 1) (2 + 6)
Discuss the possible solutions and see who has made the most.
Next give them 1, 2, 4 and 9. Ask who can make a sum out of these numbers
whose answer is 24.
One solution is 42 + 9 1.
This game can be extended to more sets of four numbers, if you wish.

Main lesson activity


Remind the class that the last lesson was devoted to solving simultaneous
equations by adding or subtracting in order to eliminate one of the two
variables.
Now, they are going to solve equations where it is possible to eliminate one of
the variables by simple substitution.
Put on the board this pair of simultaneous equations:
x 2y = 1 (i)
2x + y = 12 (ii)
From equation (i) make x the subject:
x = 1 + 2y
Next, substitute this value for x in equation (ii):
2(1 + 2y) + y = 12
2 + 4y + y = 12
2 + 5y = 12
5y = 12 2 = 10
y=2 (Divide both sides by 5.)
Now substitute y = 2 in equation (i), which gives:
x = 1 + 2y
x=1+22=1+4
x=5
Go through another example, such as:
2x + y = 7 (i)
3x 2y = 7 (ii)
From equation (i), make y the subject:
y = 7 2x
Substitute this in equation (ii) to give:
3x 2(7 2x) = 7
3x 14 + 4x = 7
7x = 21
x=3 (Divide both sides by 7.)
The class can now do Exercise 3B from Pupil Book 3.

30 HarperCollinsPublishers Ltd 2003


9B3LP_03.qxd 18/09/2003 15:18 Page 31

Exercise 3B Answers

1 x = 4, y = 1 2 x = 2, y = 3 3 x = 4, y = 3 4 x = 7, y = 3 5 x = 2, y = 5
6 x = 4, y = 7 7 x = 2, y = 1 8 x = 3, y = 5 9 x = 6, y = 3 10 x = 8, y = 2
11 x = 1, y = 5 12 x = 4, y = 5

Extension Answers

1 2.25 2 1 h 20 min

Plenary Key Words


Tell the class that there will be times when they have to make a choice about
substitution
which method to use elimination or substitution. Take, for example:
3x + 4y = 31 variable
x 4y = 5
Work through the solution using both methods. The solution is x = 9 and y = 1.
You will find that most students see that the elimination method (adding the two
equations) will be the quicker method here.
Homework

1 3x + y = 8 2 6x + 4y = 36
2x + 5y = 27 2x + y = 11
3 5x + 2y = 47 4 3x + y = 24
3x y = 26 5x + 2y = 41
5 7x 4y = 16 6 8x 4y = 36
xy=1 x + 3y = 8

Answers
1 x = 1, y = 5 2 x = 4, y = 3 3 x = 9, y = 1 4 x = 7, y = 3 5 x = 4, y = 3 6 x = 5, y = 1

HarperCollinsPublishers Ltd 2003 31


9B3LP_03.qxd 18/09/2003 15:18 Page 32

LESSON Framework objectives Find the n th term for a quadratic


sequence
3.3 Find the next term and the nth term of quadratic sequences.

Oral and mental starter


Tell the class that last night in a pub quiz, you got correct answers to 28 questions out of 37. Ask them:
Approximately what percentage is this?
Jot down the answers on the board. (76% is the correct estimate.)
Ask the students closest to explain how they estimated the answer no calculators allowed.
One way is to try to get to a fraction out of 100. This can be done by multiplying both numbers by 3. This
would give 84 out of 117. Take from the 84 a third of the extra 17 from the denominator, which is about 6. This
gives 78% not a bad estimate.
Do encourage class discussion and a sharing of techniques here, as some students will have other quite
interesting ways to explore.
Once various strategies have been considered, ask for a few more to see whether they apply.
43 out of 77 62 out of 81 34 out of 92 51 out of 123
Find a third (to get to Find a quarter (to get to Find a tenth (92 needs about a Find a fifth (to get to
25) and multiply by 4 20) and multiply by 5 tenth added on to get to 100) 25) and multiply by 4
of 34 and add it on
56% 75% 37% 40%

Main lesson activity


Ask the class what the nth term is of the sequence 3, 5, 7, 9, .
You want the answer T(n) = 2n + 1.
Remind the class that any linear sequence can be expressed by an nth term such as T(n) = An + B.
Today, however, you are going to be looking at quadratic sequences. Remind the class that a quadratic
sequence is one whose second differences are the same throughout the sequence.
The nth term of any quadratic sequence is given by T(n) = An2 + Bn + C.
You may want to show that this is true by substituting some values for A, B and C, generating the sequence, and
seeing what the second differences are.
Having established that this is the way to express a quadratic nth term, put on the board the sequence 10, 24,
44, 70, 102.
Then show the differences:
10 24 44 70 102
First differences 14 20 26 32
Second differences 6 6 6
In a previous investigation it was established that half the second difference is A. Hence, A = 3, which gives the
nth term as:
T(n) = 3n2 + Bn + C
When n = 1, T(1) = 10. Hence, 10 = 3 + B + C B + C = 7
When n = 2, T(2) = 24. Hence, 24 = 12 + 2B + C 2B + C = 12
This gives a pair of simultaneous equations to solve:
B+C=7 (i)
2B + C = 12 (ii)
Subtract equation (i) from equation (ii), to obtain B = 5.
Substitute B = 5 into equation (i) to give C = 2.
Hence, the nth term is T(n) = 3n2 + 5n + 2.
You should check this with the class on the next few terms.
Explain that you now have a routine for finding the nth term of a quadratic sequence T(n) = An2 + Bn + C.
Step 1 Find the second difference. Halve it to give A.
Step 2 Find T(1) and T(2), or any other pair, to give a pair of simultaneous equations to solve for B and C.
Step 3 Solve the simultaneous equations in B and C, given by Step 2.
You may need to go through another example with the class. Use one of the examples in Pupil Book 3.
The class can now do Exercise 3C from Book 3.

32 HarperCollinsPublishers Ltd 2003


9B3LP_03.qxd 18/09/2003 15:18 Page 33

Exercise 3C Answers

1 a No, second differences are 0 b Yes, second differences are 2


c Yes, second differences are 1 d Yes, second differences are 2
e Yes, second differences are 6 f No, second differences are all different
2 a 14 b 24 c 29
3 a 9, 19, 33, 51, 73 b 5, 18, 37, 62, 93
n (n2 + 4)
4 a n2 + 7n b 3n2 + n + 5 c 3n2 + 2n 3 d e
2
(n + 3) 2
(n + 4n + 4)

Extension Answers

12 (n + 3)(n + 4) n

Plenary Key Words


Put on the board the triangular numbers together with their associated triangles
of circles. Ask if anyone can tell you what the nth triangular number is?
quadratic
(Answer: 12 n(n + 1)) sequence
Work through this with the class as a means of bringing together the strategy for second
finding the nth term of a quadratic sequence. difference
Homework

1 Find the first five terms of each of the following sequences given by:
a T(n) = n2 + 7n 3 b T(n) = 5n2 + 3n + 1 c T(n) = 6n2 5n
2 Find the nth term for each of the following quadratic sequences.
a 13, 25, 41, 61, 85 b 12, 18, 26, 36, 48 c 7, 14, 27, 46, 71
d 13, 72, 13
3 , 21
4 , 31
5 e 19, 18
4 , 31
9 , 16
, 25
48
69 f 12
25
44
, , ,
21 46
69 100
132
83 ,
193

Answers
1 a 5, 15, 27, 41, 57 b 9, 27, 55, 93, 141 c 1, 14, 39, 76, 125
2 a 2n2 + 6n + 5 b n2 + 3n + 8 c 3n2 2n + 6 d n/(n2 + n + 1) e n2/(2n2 + 3n + 4)
f (3n2 + 4n + 5)/( 6n2 + 7n + 8)

HarperCollinsPublishers Ltd 2003 33


9B3LP_03.qxd 18/09/2003 15:18 Page 34

LESSON Framework objectives Equations involving fractions


Solve linear equations involving fractions.
3.4
Oral and mental starter
Ask for some equivalent fractions to 12 . After a few correct suggestions, such as
42 , 10
5 ,
, ask for an equivalent fraction to 12 that uses 34.
The two possible answers are 34 17
68 and .
34
Now ask for two fractions equivalent to 13 that use the number 12 ( 12 4 ).
and 12
36
Repeat this with the following examples:
Equivalent to 15 using the number 45. 45
(225
9)
and 45
2 18 12
Equivalent to 3 using the number 18.
(27 and 18 )

Equivalent to 14 using the number 28. 28
(112 7)
and 28
3 36 27
Equivalent to 4 using the number 36.
(48 and 36)

Main lesson activity


x
Put on the board the equation = 5 and ask the class how to solve it.
3
You should be given the response multiply both sides by 3, which gives x = 15.
(4x + 5)
Then, put on the board the equation = 7 and ask How do we solve this?
3
Again it is a matter of simplifying, step-by-step, the side which has the variable, x.
Start by multiplying both sides by 3 to give 4x + 5 = 21. Then subtract 5 from
both sides to give 4x = 16. Finally, divide both sides by 4 to give x = 4.
Next, write the following example on the board:
x 1 2x + 8
=
2 6
Explain that when there is a fraction on both sides, the first step is to find the
product of the denominators and multiply both sides by it. Here, this gives:
2 6 = 12,
12 ( x 1) 12 (2x + 8)
=
2 6
Now remind the students about cancelling fractions. Hence, the denominator on
each can be eliminated to give:
6( x 1) = 2(2x + 8)
Expand each side to give 6x 6 = 4x + 16. Show how to add 6 to each side,
then subtract 4x from each side to give 2x = 22 and x = 11.
When the class understand this process, you may want to work through a more
difficult problem, such as:
4(2x + 1) 2(2x 4)
=
5 3
Again, multiply both sides by the product of the denominators and then cancel
down, to obtain:
15 4(2x + 1) 15 2(2x 4)
=
5 3
which cancels down to 12(2x + 1) = 10(2x 4).
Expanding and simplifying gives the solution x = 13.
The class can now do Exercise 3D from Pupil Book 3.

34 HarperCollinsPublishers Ltd 2003


9B3LP_03.qxd 18/09/2003 15:18 Page 35

Exercise 3D Answers

1 a x = 15 b t = 15 c m = 24 d x = 12 e w = 8
2 a 22 b 2.17 c 2.75
3 a 0.8 b 3.2 c 16 d 2
4 a 3 b 17 c 19 d 21.5 e 4 f 5.5

Extension Answers

1
1 a 11 7 c 13 d 32
b 13 47
e 17
5 f 26
5
5
2 a 14 b 83 c 15 d 70
73
e 0.57 f 0.6

Plenary Key Words


12
Put on the board the equation = 2 and ask the class if anyone can solve it. denominator
x
Some students should see that the solution is x = 6, but discuss with the class the
cancelling
methods that they might have used in order to get that solution.
245
Now give them = 14 and ask how this could be solved.
x
Multiply both sides by x to give 245 = 14x. Then divide both sides by 14 to
obtain x = 17.5.
Homework

1 Solve each of the following equations.


3x 3t 6m 2x 2w
a = 12 b = 6 c = 18 d = 8 e = 6
5 5 8 5 7
2 Solve each of the following equations.
x+1 x+5 2x + 4 3x + 1
a = 5 b = 8 c = 6 d = 2
3 4 5 8
3 Solve each of the following equations.
x1 x+1 2x + 3 x 2 3x 2 x + 4
a = b = c =
3 4 3 2 5 2
4 Solve each of the following equations.
5 3 4 5 7 5
a = b = c =
x1 x+1 3x 2 2x + 1 5x 2 3x + 5

Answers
1 a x = 20 b t = 10 c m = 24 d x = 20 e w = 21
2 a x = 14 b x = 27 c x = 13 d x = 5
3 a x = 7 b x = 12 c x = 24
4 a x = 4 b x = 2 c x = 11.25

HarperCollinsPublishers Ltd 2003 35


9B3LP_03.qxd 18/09/2003 15:18 Page 36

LESSON Framework objectives Inequalities


Solve linear inequalities in one variable and represent the solution set on a number
3.5 line; begin to solve inequalities in two variables.

Oral and mental starter


Ask the class if anyone can tell you the solution to x2 = 45.
It should quickly become clear that since 45 is not a square number, only an approximate answer can be given.
We are looking for the two whole numbers between which the solution lies. These will be 6 and 7, as 62 = 36
and 72 = 49. The solution looks closer to 7 than 6, so we might estimate this as 6.7 (and 6.7).
Repeat this process for the following numbers:
x2 = 58, between 7 and 8, estimate 7.6
x2 = 158, between 12 and 13, estimate 12.6
x2 = 179, between 13 and 14, estimate 13.4

Main lesson activity


Tell the class that if your salary was tripled and then they took 100 away you would still earn less than two
thousand pounds a week. Then ask them how much you might be earning? (You could use a similar problem,
with the intention being to get them thinking about a solution that involves an inequality.)
If you were to set the problem out as an equation it would look like
3x 100 < 2000
Go through the solution of this inequality, explaining that the rules are exactly the same as for normal
equations.
This will lead to 3x < 2100 x < 700
So, you are saying that you earn less than 700 per week.
Show how this can be illustrated on a number line as:

0 700
Explain the use of an empty circle to show a strict inequality and a solid circle to show the added equality used.
Now put up on the board the inequation 4x + 5 19. Show how this reduces to x 3.5 and show this on a
number line.

1 0 1 2 3 4 5 6

The class can now do Exercise 3E from Pupil Book 3.

Exercise 3E Answers

1 a b c d e

0 1 2 8 9 10 5 6 7 1 2 3 2 3 4
f g h i j

0 1 2 2 1 0 3 2 1 3 4 5 1 0 1
k l m n o

3 2 1 5 6 7 11 12 13 3 4 5 2 3 4
2 a 1, 2, 3, 4, 5, 6, 7, 8 b 1, 3, 5 c 2, 4, 6, 8 d 2, 3, 5 e 1, 4, 9, 16 f 2, 3, 5, 7, 11 g 3, 6, 9
3 a x 5 b x > 16 c x < 3 d x 10
4 a b

2 3 4 5 6 7 3 2 1 0 1 2 3 4
c d e f

4 5 6 7 8 0 1 1 2 3 4 5 1 2 3 4

36 HarperCollinsPublishers Ltd 2003


9B3LP_03.qxd 18/09/2003 15:18 Page 37

Extension Answers

1 a y b y c y
y6 x4 y 3x + 2
5
6 2
x x
4 x
1

2 x 4, y 1, y x

Plenary Key Words


Put on the board a sketch of the graph y = x and discuss with the class where the
inequality
region y > x actually is. Discuss the line itself and the fact that as this is y = x the
region does not include the line. inequation
Draw on the board a sketch of the graph y = x2 and repeat the discussion for the
region y > x2.
Homework

1 Solve the following inequalities and illustrate their solutions on number lines.
a 5x + 7 22 b 2x 3 10 c 4x + 3 < 11
d 2(x + 4) > 20 e 4(3t + 7) 16 f 2(5x 4) 17
2 Write down the values of x that satisfy the conditions given.
a 2(4x + 3) < 50, where x is a positive, prime number.
b 2(3x 1) 60, where x is a positive, square number.
c 4(5x 3) 100, where x is positive but not a prime number.
3 Solve the following inequalities and illustrate their solutions on number lines.
a 5x 4 < 11 b 3(2x + 5) 9
x > 1 x > 4

Answers
1 a b c

2 3 4 6 6.5 7 1 2 3
d e f

5 6 7 2 1 0 2 3
2 a 2, 3, 5 b 1, 4, 9 c 1, 4
3 a b

2 1 0 1 2 3 4 5 4 3 2 1 0

HarperCollinsPublishers Ltd 2003 37


9B3LP_03.qxd 18/09/2003 15:18 Page 38

LESSON Framework objectives Graphs showing direct proportion


Solve problems involving direct proportion, relating algebraic solutions to graphical
3.6 representation of the equations.

Oral and mental starter


Tell the class that you were looking at a DVD yesterday and noted that the
running time was 135 minutes. Ask the class how many hours and minutes this
is (2 hours 15 minutes).
Discuss with the class how to calculate this, using multiples of 60 to find the
number of hours with the remainder being minutes.
Now ask: What fraction of an hour is 15 minutes? Ask for both a fraction and a
decimal answer ( 14 and 0.25).
Next ask: What fraction of an hour is 10 minutes? This is 16 as a fraction, but the
decimal is more awkward. Can the class determine what the decimal answer
would be without using a calculator?
Starting from known facts, 13 = 0.333 3333 and 16 is half of this. Hence,
16 = 0.166 6667.
Alternatively, 36 = 12 = 0.5, and 46 = 23 = 0.666 6667, so 16 = 46 36 = 0.666 6667 0.5
= 0.166 6667.
Then ask: What fraction of an hour is 5 minutes?. This is 12 1, which is half of 16 or
0.083 333.
Talk about the potential confusion of using decimal notation in time. That is,
1.50 could mean 112 hours or 1 hour 50 minutes. The students should be sure
they know which unit is being used.

Main lesson activity


Tell the class that you took a taxi journey late at night and watched the taxi fare
change on the display. The table shows the fares at various times after getting
into the taxi.
Time 00:15 00:20 00:30 00:35
Fare () 1.30 3.00 6.40 8.10
Draw a pair of axes with time on the x-axis and the fare on the y-axis. Label
them. Mark the x-axis with one hour as the principal unit, subdivided into
5-minute sections. Mark the y-axis in pounds up to 10. Ensure there is enough
room to extend the y-axis down to at least 4. Now plot the points and draw a
suitable straight line through them.
Explain to the class that as the line is straight, the two variables have a linear
relationship. Furthermore, the fare increase in a time interval is directly
proportional to the length of time. This means that the fare always increases by
the same amount for a given increase in time. Show that this works out as 1.70
every 5 minutes.
Ask the students what the equation of the line will be in terms of time, t, in
hours, and fare, f, in pounds. Remind them of the general equation of a straight
line: y = mx + c, where m is the gradient and c is the yaxis intercept.
In one hour, the fare would have increased by 12 1.70, which is 20.40.
Hence, the gradient is 20.4. Extend the vertical axis down to 4. Then
demonstrate that the line intercepts it at f = 3.8. Therefore, the equation of the
line is f = 20.4t 3.8.
Explain that the extra point used to find the equation of the line is impossible in
real-life situations, since a negative fare is meaningless.
Ask what the fare would be at 00:50 if the same rate continued.
The graph can be used, or the equation. But if the equation is used, 50 minutes
must be changed into hours ( 60 50
= 0.833 hours), giving 13.20.

The class can now do Exercise 3F from Pupil Book 3.

38 HarperCollinsPublishers Ltd 2003


9B3LP_03.qxd 18/09/2003 15:18 Page 39

Exercise 3F Answers

1 b Yes c C = 1.5t 8 d 16C


2 b Yes c f = 48 000t 58 000 d Between 12 and 13 minutes past one
w
3 b Yes c L = + 10 d 1700 g
100
4 b Yes c S = 52t 156 d 338
5 b Approximately, yes c B = Approximately 0.66H d Approximately 760 cm

Extension Answers

a Approximately 10.40 AM b 5 times

Plenary Key Words


Ask the class what is meant by directly proportional.
directly
They need to understand that this means that one variable always increases by
the same amount for a given increase in the other variable. The graph of two proportional
variables which are directly proportional will always be linear (that is, a straight linear
line). relationship
Discuss the fact that some students did not find it easy to obtain the equations of
the lines in Pupil Book 3, Exercise 3F. Using the idea of y = mx + c will always
help them in this respect.
Homework

A baby squid is weighed from birth at midday for its first 5 days. The results are shown in the table
below.
Day 1 2 3 4 5
Weight (kg) 1.7 3.1 4.5 5.9 7.3

a Plot the points on a graph and join them with a suitable line.
b Is the increase in weight during a time interval directly proportional to the length of the interval?
c Write down the equation of the line showing the relationship between the weight (W ) and the age
(D) of the squid.
d If the relationship held, at what age would the squid first weigh over 15 kg?

Answers
b Yes c W = 1.4D + 0.3 d Day 11

HarperCollinsPublishers Ltd 2003 39


9B3LP_03.qxd 18/09/2003 15:18 Page 40

LESSON Framework objectives Solving simultaneous equations by


graphs
3.7 Link a graphical representation of a pair of equations to the algebraic solution.

Oral and mental starter


Ask whether any student can estimate the answer to 37 8.
You are hoping for About 300.
Discuss with the class the various methods they used, such as rounding 37 up to
40, then 8 40 = 320. But since 37 was rounded up, round down the answer to
300.
Ask for the approximation of 28 43.
You are hoping for About 1200 maybe from 30 40.
Discuss the different strategies the class have adopted.
Ask the following, ensuring that everybody is taking part and knows what they
are doing.
33 58 67 72 38 59 19 98 25 36
30 60 = 1800 70 70 = 4900 40 60 = 2400 20 100 = 2000 100 36 4 = 900
You know it will be You know it will be Did anyone spot this
less. less. way?

Should the class find these too simple, challenge them with the three- and two-
digit product estimations:
237 76 319 88 423 579 792 617
200 80 = 16 000 300 90 = 27 000 400 600 = 240 000 800 600 = 480 000

Main lesson activity


Put a pair of axes on the board and draw on the line with the equation
y = 2x + 1 from (1, 1) to (3, 7).
Tell the class this is the graph of y = 2x + 1. Ask them what the graph actually
represents.
What you want as a response is: Its a collection of solutions to the equation.
Every point on the graph represents a different possible solution. For example,
(0, 1) represents x = 0, y =1, (1, 3) represents x = 1, y =3, and so on.
Ask the class for some solutions to the equation and write them down, as
coordinates.
Now, using the same axes, put on the board the graph with the equation
y = 4x 2, from (1, 6) to (3, 10).
Again, ask for some solutions for this equation. For example, (0, 2) representing
x = 0, y = 2 and (2, 6) representing x = 2, y = 6.
Ask the question: What is so special about the point where the two lines cross?
One of the students should comment that the coordinates are on each graph. So,
at that point the solution of each equation is the same. In other words, this is the
solution of the two simultaneous equations.
Show that the point where these two lines cross is (112 , 4) and that this point does
indeed satisfy both equations.
Explain to the class that an alternative way to find the solution of simultaneous
equations is to draw their graphs.
The class can now do Exercise 3G from Pupil Book 3.

40 HarperCollinsPublishers Ltd 2003


9B3LP_03.qxd 18/09/2003 15:18 Page 41

Exercise 3G Answers

1 x = 1.5, y = 3.5 2 x = 2.5, y = 1.5 3 x = 4.5, y = 3.5 4 x = 1.5, y = 3.5


5 x = 1.3, y = 2.7 6 x = 2.2, y = 1.6 7 x = 1.2, y = 3.3 8 x = 2.1, y = 1.9

Extension Answers

1 Two solutions: x = 1.9, y = 2.6 and x = 3.2, y = 8.9


2 Two solutions: x = 0.9, y = 1.7 and x = 4.5, y = 15.8

SATs Answers

1 y = 7.3
7
2 a 2 b 1 c d 2
3
3 a (1 1) + (2 n) + (3 5) + (4 6) + (5 3) = 55 + 2n b 15 + n c 10
4 x = 1, y = 1

Plenary Key Words


Draw on the board a pair of axes. Then draw on the lines whose equations are intersection
x + y = 4 and x + y = 7.
Ask whether these two simultaneous equations have a solution.
This should prompt some useful class discussion about solutions and
intersections. The fact that these two lines are parallel to each other indicates
that there can be no solution to them as a pair.
Homework

1 a On the same pair of axes, draw the graphs of the equations y = 2x + 1 and y = 2x + 3.
b Explain why there is no solution to this pair of simultaneous equations.
2 a Does every pair of linear simultaneous equations have a solution?
b Explain your answer to part a.
3 a Does every pair of simultaneous equations which do have a solution, have a unique solution?
b Explain your answer to part a.
4 Sketch a pair of graphs, one quadratic and one linear, which represent a pair of simultaneous
equations that will have only one solution.

Answers
1 b Because the two lines are parallel, they do not intersect
2 a No b When the pair represent a pair of parallel lines
3 a No
b When one equation can be simplified to the other equation, there are an infinite number of solutions
4 U-shaped curve and a straight line, which is a tangent to the curve, will have only one point in common

HarperCollinsPublishers Ltd 2003 41


9B3LP_04.qxd 18/09/2003 15:18 Page 42

CHAPTER
4 Shape, Space and Measures 1
LESSON Framework objectives Pythagoras theorem
Understand and apply Pythagoras theorem.
4.1
Oral and mental starter
Ask the class to find the square and the square root keys on their calculators.
Ask individual students to explain how to use the two keys by giving examples.
Next, ask the class to copy the table below and then complete it as quickly as possible. They must only use
their calculators for the square numbers that they do not know.
x 1 2 3 4 5 6 7 8 9 10 11 12 13 14 15 16 17 18 19 20
x2
Now repeat the activity for square roots. Ask the class to give their answers to one decimal place.
x 1 2 3 4 5 6 7 8 9 10 11 12 13 14 15 16 17 18 19 20

x
Give the class the answers.
x 1 2 3 4 5 6 7 8 9 10 11 12 13 14 15 16 17 18 19 20
x 2 1 4 9 16 25 36 49 64 81 100 121 144 169 196 225 256 289 324 361 400

x 1 1.4 1.7 2 2.2 2.4 2.6 2.8 3 3.2 3.3 3.5 3.6 3.7 3.9 4 4.1 4.2 4.4 4.5

Main lesson activity


Tell the class that they are going to use Pythagoras theorem, which was discovered over two thousand years
ago. Explain that it is used to calculate the length of sides in right-angled triangles.
Draw the diagram on the right on the board or OHP. Remind the class that in a right-
angled triangle, the longest side, which is always opposite the right angle, is called the Hypotenuse
hypotenuse.
Ask the class to do the activity in Pupil Book 3, page 59.
Pythagoras theorem
Pythagoras was a Greek philosopher and mathematician who was born in about 581 BC on the island of
Samos, just off the coast of Turkey.
The following famous theorem about right-angled triangles is attributed to him.
In any right-angled triangle, the square of the hypotenuse is equal to the sum of
c
the squares of the other two sides.
a
Pythagoras theorem is usually written as:
c2 = a2 + b2
b
Show the class how to find the length of a hypotenuse by doing the following example.
Calculate the length x in the triangle shown on the right.
Using Pythagoras theorem:
x2 = 82 + 72 8 cm
x
= 64 + 49
= 113

So, x = 113 = 10.6 cm (1 dp). 7 cm
Then show them how to work this out on a scientific calculator.
8 x2 + 7 x2 = x =

This may not work on some makes of calculator, and you may need to show some students how to do it on
their calculator.

42 HarperCollinsPublishers Ltd 2003


9B3LP_04.qxd 18/09/2003 15:18 Page 43

Next, show the class how to find the length of a shorter side by doing the following example.
Calculate the length x of the triangle on the right.
Using Pythagoras theorem:
x2 + 72 = 102 7 cm 10 cm
x2 = 102 72
= 100 49
= 51 x

So, x = 51 = 7.1 cm (1 dp).
Then, show them how to work this out on a scientific calculator.
10 x2 7 x2 = x =

The class can now do Exercise 4A from Pupil Book 3.

Exercise 4A Answers

1 a 5.8 cm b 9.2 cm c 12.0 cm d 13.4 cm e 5.7 cm f 3.6 m g 4.1 m h 10.9 m


2 a 6.2 cm b 10.2 cm c 6.6 cm d 8.7 cm e 4.5 m f 4.0 m g 4.4 m h 4.4 m
3 x = 1.4 cm, y = 1.7 cm, z = 2.0 cm 4 10.8 cm 5 7.1 cm

Extension Answers

When a2 + b2 = c2, C is a right angle


a2 b2 c2 a2 + b2 Is a2 + b2 = c2? Is C right-angled, When a2 + b2 < c2, C is an acute angle
Is a2 + b2 > c2? acute or obtuse? When a2 + b2 > c2, C is an obtuse angle
Is a2 + b2 < c2?
Write =, > or <
9 16 25 25 = Right-angled
16 25 49 41 < Acute
25 36 49 61 > Obtuse
25 144 169 169 = Right-angled Key Words
16 64 100 80 < Acute
49 64 81 113 > Obtuse
hypotenuse
Pythagoras
theorem
Plenary square
Ask the class to explain Pythagoras theorem. square root
Tell them that in the next lesson they will apply Pythagoras to solve problems.
Homework

1 Calculate the length of the hypotenuse in each of the following right-angled triangles.
Give your answers to one decimal place.
a b c 16 cm
a b
2 cm 6 cm
3 cm 10 cm
c
9 cm

2 Calculate the length of the unknown side in each of the following right-angled triangles. Give your
answers to one decimal place.
a b c c
14 cm
7 cm b
5 cm 7.2 cm
9.8 cm
12 cm
a 25 cm
x
3 a Calculate x in the right-angled triangle shown on the right.
b Calculate the area of the triangle. 24 cm

Answers
1 a 3.6 cm b 10.8 cm c 18.9 cm 2 a 4.9 cm b 7.2 cm c 6.6 cm 3 a 7 cm b 84 cm2

HarperCollinsPublishers Ltd 2003 43


9B3LP_04.qxd 18/09/2003 15:18 Page 44

LESSON Framework objectives Solving problems using Pythagoras


theorem
4.2 Understand and apply Pythagoras theorem.

Oral and mental starter


The class will need calculators for this starter.
Ask the class to copy and complete the following table.
x 15 25 35 45 55 65 75
x2
Ask the class whether they can see the rule for squaring a number that ends in 5.
Answer:
x 15 25 35 45 55 65 75
x2 225 625 1225 2025 3025 4225 5625
The last two digits of x2 are 25 and the digit(s) before the 25 is (are) the product
of the number in the tens column of x and this number plus 1.

Main lesson activity


Explain to the class that Pythagoras theorem can be used to solve various
practical problems. When solving a problem, it is helpful to proceed as follows.
Draw a diagram for the problem, clearly showing the right angle.
Decide whether the hypotenuse or one of the shorter sides needs to be found.
Label the unknown side x.
Use Pythagoras theorem to calculate x.
Round your answer to a suitable degree of accuracy.
Show the class how to do the following problem.
A ship sails 25 km due east. Then it sails for a further 45 km due south. Calculate
the distance the ship would have travelled if it had sailed the direct route.
First, draw a diagram to show the distances sailed by the ship. Then label the
direct distance x. 25
Now use Pythagoras theorem:
x2 = 252 + 452
= 625 + 2025 45
= 2650 x

So, x = 2650 = 51.5 km (1 dp).
The class can now do Exercise 4B from Pupil Book 3.

44 HarperCollinsPublishers Ltd 2003


9B3LP_04.qxd 18/09/2003 15:18 Page 45

Exercise 4B Answers

1 108 km 2 10.8 m 3 7.7 m 4 2.56 m 5 1.78 m


6 a 36.1 cm b 22.4 cm
7 a 7.4 cm b 22.2 cm2
8 4.5 km
9 a 4 b 5
10 a 4.5 b 4.8 c 4.2 d 9.8

Extension Answers

a Some further Pythagorean triples with a odd are given below.


a b c
3 4 5
5 12 13
7 24 25
9 40 41
11 60 61
13 84 85
15 112 113
b For a odd and greater than 1, b = 12 (a2 1) and c = 12 (a2 + 1)
c Yes

Plenary Key Words


Ask the class to give a summary of how to go about solving a practical problem
using Pythagoras theorem.
Pythagoras
theorem
Homework

1 A plane flies due east for 120 km from airport A to airport B. It then flies due north for 280 km to
airport C. Finally, it flies directly back to airport A. Calculate the direct distance from airport C to
airport A. Give your answer to the nearest kilometre.
2 The length of a football pitch is 100 m and the width of the pitch is 80 m. Calculate the length of a
diagonal of the pitch. Give your answer to the nearest metre.
3 The regulations for the safe use of ladders states: For a 6 m ladder, the foot of the ladder must be
placed between 1.5 m and 2.2 m from the building.
a What is the minimum height the ladder can safely reach up the side of a building?
b What is the maximum height the ladder can safely reach up the side of a building?
4 Calculate the area of an equilateral triangle whose side length is 10 cm. Give your answer to one
decimal place.

Answers
1 305 km
2 128 m
3 a 5.6 m b 5.8 m
4 43.3 cm2

HarperCollinsPublishers Ltd 2003 45


9B3LP_04.qxd 18/09/2003 15:18 Page 46

LESSON Framework objectives Loci


Find the locus of a point that moves according to a more complex rule, involving
4.3 loci and simple constructions.

Oral and mental starter


Ask the class to imagine a stick standing upright in the ground. Then tell them to
imagine a fly moving around the stick, so that it is always exactly 5 cm from it.
Next, invite the class to describe the shape which shows all the different
positions at which the fly could be. Allow them to work in pairs or groups, and
give them about 5 minutes to discuss their answers. (Answer: surface of a
cylinder with a hemisphere on top.)

Main lesson activity


Tell the class that they are going to learn how to find loci for more complicated
situations than those which they met in Year 8.
Remind them that a locus is the movement of a point according to a given set of
conditions or a rule.
Go over the two important constructions of Example 4.4 in Pupil Book 3, which
can now be stated to be loci.
The locus of a point which is always The locus of a point which is
equidistant from each of two fixed equidistant from two fixed lines
points, A and B, is the perpendicular AB and BC, which meet at B, is
bisector of the line joining the two the bisector of the angle ABC.
points.
A

A B

B
C

Explain that a locus can sometimes be a region, as shown in the three examples
to the right.
A point which moves so that it is always 5 cm from a fixed point X has a locus
X
which is a circle of radius 5 cm, with its centre at X.

The locus of a set of points which are 5 cm or less from a fixed point X is the
region inside a circle of radius 5 cm, with its centre at X.
X
Note that the region usually is shaded.

The locus of a set of points that are less than 5 cm from a fixed point is the
region inside a circle of radius 5 cm, with its centre at X.

Note that the boundary usually is drawn as a dashed line to show that the X
points which are exactly 5 cm from X are not to be included.

The class can now do Exercise 4C from Pupil Book 3.

46 HarperCollinsPublishers Ltd 2003


9B3LP_04.qxd 18/09/2003 15:18 Page 47

Exercise 4C Answers

3 a b c

X X X

4 5 6 P X

B
A B C X

D Q R

7 8

X Y A B

9 All points on the surface of a sphere with radius 10 cm


10 All points on the surface of a half cylinder with half of a hemisphere at each end, both with a radius of 5 cm

Extension Answers
Key Words
1 Locus is a cycloid (see diagram)
angle bisector
2 Locus is an arc of the circle whose radius is 3 m
4 Point is where perpendicular bisector of AB meets road
perpendicular
bisector
equidistant
Plenary locus
loci
Ask the class to give the definition of a locus. region
Then ask them to make up some examples of their own.
Homework

1 Using a ruler and compasses, construct the locus which is A B


equidistant from the points A and B. 5 cm

2 Using a ruler and compasses, construct the locus which is equidistant A


from the perpendicular lines AB and BC.
3 Draw a diagram to show the locus of a set of points which are 4 cm or 5 cm
less from a fixed point X.
4 Two alarm sensors, 6 m apart, are fitted to the side of a house, as shown C
below. The sensors can detect movement to a maximum distance of 5 m. B 5 cm
Draw a scale drawing to show the region that can
6m
be detected by both sensors. Use a scale of 1 cm
to 1 m.

Answers
1 Perpendicular bisector of AB 4
2 Angle bisector of angle ABC
3 Shaded region inside the circle of radius 4 cm

HarperCollinsPublishers Ltd 2003 47


9B3LP_04.qxd 18/09/2003 15:18 Page 48

LESSON Framework objectives Congruent triangles


Know from experience of constructing them that triangles given SSS, SAS, ASA or
4.4 RHS are unique, but that triangles given SSA or AAA are not.
Apply the conditions SSS, SAS, ASA or RHS to establish the congruence of triangles.
Explain how to find, calculate and use the interior and exterior angles of regular
polygons.
Understand congruence.

Oral and mental starter


Ask the class how to recognise congruent shapes. Their answer should be: Two shapes are congruent when
they are exactly the same shape and size.
Draw on the board the following triangles, or use a prepared OHT.
8 cm 7 cm
7 cm 8 cm
8 cm
A B C 9 cm D 6 cm
6 cm 6 cm
8 cm 8 cm
9 cm

6 cm
8 cm 8 cm 8 cm 10 cm
6 cm 8 cm
E F 8 cm G H

10 cm 6 cm
6 cm 6 cm
Ask the class which pairs of triangles are congruent, inviting individual students to explain their answers.
The following questions could also be asked:
Do you need dimensions to show that the triangles are congruent?
Do you have to draw the triangles exactly to prove that they are congruent?
Do you need to be given any angles to show that the triangles are congruent?
Ask them whether they can draw other pairs of triangles which are congruent but include angles.

Main lesson activity


Tell the class that they will now learn how to prove that two triangles are congruent when information is given
on both triangles.
Remind the class that they already know how to construct triangles from given information. Then summarise
the following on the board or using an OHT.

Side Side Hypotenuse


Side

Angle Angle Angle


Side Side Side Side

Three sides (SSS) Two sides and the Two angles and the Right angle, hypotenuse
included angle (SAS) included side (ASA) and side (RHS)
Now apply these conditions to show that the two triangles given on the A Z X
right are congruent.
70
Invite individual students to state which angles and which sides are
equal. Write their responses on the board, which should be as follows:
B = X C = Y BC = XY 70 5 cm
Then ask the class whether this proves that ABC is congruent to B 56
XYZ, to which they should respond affirmatively. Get them to state 56
the condition of congruence (ASA). 5 cm
Explain to the class that it is a convention to show congruence by using the symbol . C Y
Hence, the congruence of these two triangles may be written as:
ABC XYZ
The class can now do Exercise 4D from Pupil Book 3.
48 HarperCollinsPublishers Ltd 2003
9B3LP_04.qxd 18/09/2003 15:18 Page 49

Exercise 4D Answers

1 a C = D, BC = DE, AC = DF (SAS) b GH = KL, GI = JL, HI = JK (SSS)


c N = R, O = P, NO = PR (ASA) d T = V = 90, SU = WX (hypotenuse), TU = VX (RHS)
2 X = 40 (angles in a triangle). So, B = X, C = Y, BC = XY (ASA)
3 a Angles are the same but no sides are given. So, the two triangles could be drawn with different side lengths. AAA is
not a condition for congruence
4 ADB = ADC = 90, AB = AC (hypotenuse), AD is a common side. So, ABD ACD (RHS)
5 ACD ABC ABD BCD, AXB CXD, AXD BXC

Extension Answers

1 Draw, in turn, each diagonal and show that the two triangles formed are congruent (SSS). Hence, opposite angles are
equal
2 a AB = AD, BC = CD, AC a common side. So, ABC ADC (SSS). b ABE ADE, DCE CE
3 Four different triangles are possible, ignoring reflections or rotations
4 Case 1 Case 2 Case 3

Side 2 Side 3 Side 3 Side 3


Side 2
Side 2 Side 2
Angle Angle Angle
Side 1 Side 1 Side 1
Arc for the radius of side 2 cuts Arc for the radius of side 2 touches Arc for the radius of side 2
side 3 in two places. So, two triangles side 3, forming a right angle. does not meet side 3. So, it is not
can be drawn. This is condition RHS. possible to draw the triangle.

Plenary Key Words


Ask the class to write in their books the four conditions which show that two
congruent
triangles are congruent.
congruence
Homework

1 Show that each of the following pairs of triangles are congruent. Give reasons for your answers and
state which condition of congruence you are using. 7 cm
G J K
a A F b
5 cm 5 cm
9 cm 8 cm
7 cm 9 cm
40 40
B C D E
6 cm 6 cm
H 8 cm I L
M Q
c d S
10 cm
75

55 R V W

15 cm
15 cm 9 cm
55 75
N 10 cm O P
T U X
9 cm
2 ABCD is a rectangle and E is the mid-point Answers
of AB. A E B 1 a C = D, BC = DE, AC = DF (SAS)
b GH = JK, GI = JL, HI = KL (SSS)
c N = R, O = Q, NO = QR (ASA)
d T = W = 90, SU = VX (hypotenuse),
TU = WX (RHS)
2 AE = EB, AD = BC, A = B (SAS)
D C
Explain why AED is congruent to BEC.

HarperCollinsPublishers Ltd 2003 49


9B3LP_04.qxd 18/09/2003 15:18 Page 50

LESSON Framework objectives Circle theorems


Distinguish between practical demonstration and proof.
4.5 Know that the tangent at any point on a circle is perpendicular to the radius at that
point. Explain why the perpendicular from the centre to the chord bisects the chord.

Oral and mental starter


Draw a circle and a vertical line on the board or OHP, as in the diagram.
Tell the class to imagine the line getting closer to the circle, passing through the
circle and then moving to the other side of the circle.
Ask the class to draw on their white boards or on the board, the different
situations which can occur.
Answer:

Touches at Cuts the circle Passes through Touches at Does not touch
a point twice the centre a point again again

Main lesson activity


Remind the class of the following terms for parts of a circle that they met in Year 8.
A circle is a set of points equidistant from a centre, O.
Circumference, C
Circumference The distance around a circle.
O

Diameter, d
Radius The distance from the centre of a circle to its
circumference.
d
Radius, r
Diameter The distance from one side of a circle to the other, O r
passing through the centre.

Chord A line which cuts a circle into two parts.


Chord
Tangent A line that touches a circle at a single point on the
circumference.

Tangent

Alert the class to the fact that they are going to meet two important circle theorems in Exercise 4E:

O O

The radius at the point of contact of a tangent The perpendicular bisector of a chord
to a circle is perpendicular to the tangent. passes through the centre of a circle.

The class can now do Exercise 4E from Pupil Book 3.

50 HarperCollinsPublishers Ltd 2003


9B3LP_04.qxd 18/09/2003 15:18 Page 51

Exercise 4E Answers

3 a 28 b 65 c 50 d 120 e 51 f 29 g 123 h 48 i 50
4 a 8.5 cm b 22.9 cm c 6.4 cm d 10.9 cm
5 Centre of the circle is the intersection of the perpendicular bisector of EF and the perpendicular drawn from either E or F
6 Centre of the circle is the intersection of the perpendicular bisector of XY and the perpendicular bisector of XZ

Extension Answers

1 a 33 b 42 c 70 d 52 e e = f = 67
2 In ABO, ABO = 90 (angle in a semicircle). OB is a radius of the small circle and since ABO = 90, AB is a
tangent (radius is perpendicular to tangent)
3 Join OA, OB and OP. In AOP and BOP, OA = OB (radii), OAP = OBP = 90 (radii are perpendicular to tangents),
OP is a common side. So, AOP BOP (RHS), hence AP = BP

Plenary Key Words


Invite individual students to explain on the board or OHP the two circle
theorems they have met in the lesson.
chord
radius
tangent
Pythagoras
theorem
Homework

1 Calculate the size of the lettered angle in each of the following diagrams.

a b 38 c

O b O
110
a O
43 c

d e f
d

O 61
122 O
e f
O 56

2 Use Pythagoras theorem to calculate the length x in each of the following diagrams. Give your
answers to one decimal place.
20 cm
a b c d
O 10 cm
14 cm
O 8.5 cm
x
x x 3 cm O
7 cm x O
3 cm 3 cm
18 cm
3 A circle passes through the three points A, B and C. B
On a copy of the diagram, construct the circle,
using a ruler and compasses. A

Answers
1 a 47 b 52 c 55 d 58 e 34 f 61 2 a 19.3 cm b 7.1 cm c 4.2 cm d 10.5 cm
3 Centre of the circle is intersection of perpendicular bisector of AB and perpendicular bisector of BC (and
perpendicular bisector of AC)

HarperCollinsPublishers Ltd 2003 51


9B3LP_04.qxd 18/09/2003 15:18 Page 52

LESSON Framework objectives Tessellations and regular polygons


Explain how to find, calculate and use the interior and exterior angles of regular
4.6 polygons.

Oral and mental starter


Show the class two transparencies, each displaying a set of parallel lines 4 cm
apart, as below.

Ask them what shapes are formed when one transparency is placed on top of the
other and one of them is rotated.
Then show them that only squares and rhombuses can be formed.
Now ask the class what shapes can be formed when one of the transparencies
has parallel lines 2 cm apart. (Answer: rectangles and parallelograms)

Main lesson activity


Remind the class about tessellations, which they met in Year 7. A tessellation is a
pattern made on a plane (flat) surface with identical shapes which fit together
exactly, leaving no gaps. Explain that it is usual to draw up to about ten of the
shapes to show the tessellating pattern.
Show the class how equilateral triangles and squares tessellate.
Ask the class whether any other regular polygons will tessellate. Explain that
they will be doing a practical activity to discover which of the regular polygons
tessellate and the reason why.
Remind the class how to find the size of the interior angle of a regular polygon
by showing them an example for a regular pentagon.
x is an exterior angle of the regular pentagon.
The sum of the exterior angles for any polygon is 360. Since all the exterior
angles are equal, it follow that:
x = 360 5 = 72
y is an interior angle of the regular pentagon. Hence:
y = 180 72 = 108
For Exercise 4F, the class will require squared paper, isometric paper, card for y x
making regular polygon templates and scissors. Sets of commercially produced,
regular polygons can also be useful, so that the students can easily visualise the
shapes, particularly if they have difficulty in making their own templates.
The class can now do Exercise 4F from Pupil Book 3.

52 HarperCollinsPublishers Ltd 2003


9B3LP_04.qxd 18/09/2003 15:18 Page 53

Exercise 4F Answers

2 b There are gaps 3 b There are gaps (squares)


4 a Regular polygon Size of each interior Does polygon
angle tessellate?
Equilateral triangle 60 Yes
Square 90 Yes
Regular pentagon 108 No
Regular hexagon 120 Yes
Regular octagon 135 No
b Size of the interior angle divides exactly into 360
c No, each interior angle of a regular nonagon is 140, which does not divide
exactly into 360

Plenary Key Words


Invite a student to the board and give her/him a regular polygon, asking the
student to show whether it tessellates.
exterior angle
Continue to invite individual students to the board, giving each a different interior angle
polygon. regular polygon
tessellate
Homework

1 Work out, by making templates or by drawing diagrams, which of the following regular polygons
tessellate, and which do not. In each case, write down a reason for your answer.
a Equilateral triangle b Square c Regular pentagon d Regular hexagon e Regular octagon
2 Draw a diagram to show how squares and equilateral triangles together form a tessellating pattern.

Answers
1 a Yes, interior angle is 60, which divides exactly into 360
b Yes, interior angle is 90, which divides exactly into 360
c No, interior angle is 108, which does not divide exactly into 360
d Yes, interior angle is 120, which divides exactly into 360
e No, interior angle is 135, which does not divide exactly into 360
2 For example:

HarperCollinsPublishers Ltd 2003 53


9B3LP_04.qxd 18/09/2003 15:18 Page 54

LESSON Framework objectives Practical Pythagoras


Distinguish between a practical demonstration and a proof.
4.7 Understand and apply Pythagoras theorem.

Oral and mental starter


Tell the students to imagine a square.
Now tell them to imagine cutting the square along one of its diagonals.
Next, ask the students to describe the two shapes that are left. Answer: two isosceles right-angled triangles.
Then tell them to imagine cutting the square again, but this time along a line which is parallel to the diagonal.
Finally, get the students to describe the two shapes that are left. Answer: an isosceles right-angled triangle and a
pentagon.

Main lesson activity


Explain to the class that the aim of the lesson is to show the difference between a practical demonstration and a
proof. A practical demonstration shows how a rule or theorem works by using a specific example, whereas a
proof shows how the rule or theorem works for all cases. A theorem is usually proved by using algebra.
The activity in Pupil Book 3, page 75 gives a practical demonstration to show Pythagoras theorem. Each
student will need a sheet of thin card and a pair of scissors. For completeness, the activity is given below.
In your book, draw the right-angled triangle X, as below.

5 cm
3 cm
X

4 cm
On the card, draw eight more triangles identical to X. Cut them out and place them to one side.
On your original triangle, X, draw squares on each of the three sides of the triangle. Label them A, B and C,
as below.

5 cm B
X 3 cm
4 cm

On the card, draw another diagram identical to this. Cut out the squares A, B and C.
Arrange the cut-outs of the eight triangles and three squares as in the two diagrams below.
Diagram 1 Diagram 2

X X X B
X
C
X
A
X X X

What can you say about the total area of Diagram 1 and of Diagram 2?
Now remove the four triangles from each diagram.
What can you say about the areas of squares A, B and C?
Show how this demonstrates Pythagoras theorem.
Answer Diagram 1 and Diagram 2 have the same total area.
Area of A + Area of B = Area of C
So, 16 + 9 = 25 cm2, which is 42 + 32 = 52 cm2.
More able students can do the Extension Work, which gives a proof of Pythagoras c
b
theorem.
Proof of Pythagoras theorem
A right-angled triangle has sides a, b and c. a

54 HarperCollinsPublishers Ltd 2003


9B3LP_04.qxd 18/09/2003 15:18 Page 55

The diagram on the right can be drawn using four of these triangles. a b
Area of each triangle = 12 ab.
b c
So, area of the four triangles = 2ab.
c
Area of the large square = (a + b)2. a
Area of the large square can also be written as c2 + 2ab. Hence:
(a + b)2 = c2 + 2ab
a + 2ab + b2 = c2 + 2ab
2
a c
a2 + b2 = c2 b
c
which is Pythagoras theorem.
b a
SATs Answers

1 a Sum of interior angles of a triangle = 180, so 2 180 = 360 b 540 c 900


2 a 20.8 cm b 9.8 cm 3 A and C, three equal sides (SSS)
4

2m

5 a AD = 33.8 cm, AC = 31.2 cm and CD = 13 cm, so perimeter = 78 cm


b AC2 + CD2 = 1142.44 and AD2 = 1142.44, so by Pythagoras theorem ACD is right-angled
6 a ABO = x (ABO is isosceles, since radii are equal), CBO = y (CBO is isosceles, since radii are equal)
b x + x + y + y = 180, 2x + 2y = 180. So, x + y = 90 = ABC

Plenary Key Words


Ask the class to explain the difference between a practical demonstration and a
proof
proof.
prove
Pythagoras
theorem
Homework

Practical demonstration with a difference


Cut out an 8 cm by 8 cm square and then cut it up Now rearrange the four pieces to make a
into two right-angled triangles and two trapezia, rectangle, as in the diagram below.
as in the diagram below.

3 cm

3 cm 5 cm

5 cm

5 cm 3 cm
What is the area of the square and of the rectangle?
Can you explain why this practical demonstration does not work?
Answers
The area of the square is 64 cm2 and the area of the rectangle is 65 cm2. The apparent increase of 1 cm2 in the
area of the rectangle is due to the fact that the diagonal of the rectangle is not a straight line. This can be shown
by applying Pythagoras theorem.

The length of the diagonal in the rectangle should be 194 = 13.928 388 28. This diagonal is the length of the

hypotenuse of the triangle plus the length of the sloping side of the trapezium, which is 73 + 29 =
13.929 168 55.
This shows that the diagonal of the rectangle cannot be a straight line.

HarperCollinsPublishers Ltd 2003 55


9B3LP_05.qxd 18/09/2003 15:19 Page 56

CHAPTER
5 Handling Data 1
LESSON Framework objectives Statistical investigations
Suggest a problem to explore using statistical methods, frame questions and raise
5.1 conjectures.
Discuss how data relate to a problem. Identify possible sources, including primary
and secondary sources.
Design a survey or experiment to capture the necessary data from one or more
sources. Determine the sample size and degree of accuracy needed. Design, trial
and if necessary refine data collection sheets.
Identify possible sources of bias and plan how to minimise it.

Oral and mental starter


The students can work in small groups for this activity.
Write the following sources of information on a sheet: Questionnaire, Printed
tables in books, Internet, Computer database and Observation sheet.
Give the students cards, or a list, containing the following topics.
Primary Data Secondary Data
Number of left-handed students Long jump performances by
in the class international athletes
TV viewing habits of students Car engine sizes
Reaction times of students Populations of various countries
Whether students are better at Football results in Europe
catching with their left or Prices of different makes of
right hand second-hand cars
The amounts of pocket money
received by males and females
in school
They should discuss each topic and decide how best to investigate each one,
using the sources given above.

Main lesson activity


Explain to the class that the aim of the lesson is to look at how to plan a
statistical investigation.
Point out that sometimes the most difficult part is to decide a topic to investigate.
(Steps 1 and 2 in the table below.)
Explain that to help them they will be given a planning sheet so that they may
work systematically through their problem.
Take an example of your choice or use the example given below.

Step Example
1 Decide which general topic to The cost of housing in different parts of
study the UK
2 Specify in more detail Comparing the costs in Wales and
England
3 Consider questions which you Is the average price higher in Wales?
could investigate Is there a bigger difference in the prices
in England than in Wales?
4 State your hypotheses (Your The price is higher in Wales
guesses at what could happen) There is more variation in price in
England

56 HarperCollinsPublishers Ltd 2003


9B3LP_05.qxd 18/09/2003 15:19 Page 57

5 Sources of information required Internet. Estate agents. Building societies


and banks mortgage reports. Government
data: for example, Office for National
Statistics http://www.statistics.gov.uk/
6 Relevant data Average house prices in different
counties of Wales and England
7 Possible problems Counties of different sizes may affect the
average unfairly, causing your data to be
biased
8 Data collection Make sure that your sample size is big
enough to draw valid conclusions
9 Decide on level of accuracy If results are within, say, 1000 pounds
required of each other, you may decide that the
results do not support the hypothesis that
the prices are different
10 Determine sample size Make sure that you collect enough data
from both countries
11 Construct tables for large sets of
raw data in order to make work
manageable
12 Decide which statistics are most If there are a few extreme values, you
suitable may choose to ignore the mean, as this
will distort the results

You could now show the students the three examples in Pupil Book 3.
The class can now do Exercise 5A from Pupil Book 3.

Exercise 5A Answers

Answers will vary but should be similar in style to the examples.

Plenary Key Words


Use a groups planning sheet to discuss the points on it.

questionnaire
Ask other groups to contribute points that can be added to the planning sheet.
Explain that the homework is to produce an individual plan for a different topic. printed table
The students could use ideas already used by other groups. database
survey
statistic
bias
census
Homework

Take a different topic to those already studied and prepare a new planning sheet.

Answers
Answers will vary but should be similar in style to the examples.

HarperCollinsPublishers Ltd 2003 57


9B3LP_05.qxd 18/09/2003 15:19 Page 58

LESSON Framework objectives Scatter graphs and correlation


Select, construct and modify, on paper and using ICT, suitable graphical
5.2 representation to progress an enquiry, including scatter graphs to develop further
understanding of correlation. Identify key features in the data.

Oral and mental starter


Write a simple, open, multiplication table on the board or OHP, as shown. +3 +2 +1
Prompt the students to tell you how to complete it. +3
Now add on an extra column, with zero. +2
Complete the extra column, with the students help. +1
Prompt them to extend the table, into negative numbers.
Use patterns (for example, subtracting 3s in the first row) to
+3 +2 +1 0 1 2 3
complete the columns.
+3 9 6 3 0 3 6 9
Now prompt the students to fully extend the table downwards,
+2 6 4 2 0 2 4 6
as shown.
+1 3 2 1 0 1 2 3
Show the class that, using patterns, they
+ 0 0 0 0 0 0 0 0 0
have just proved the rules for
+ + 0 1 3 2 1 0 1 2 3
multiplying positive and negative
0 0 0 0 2 6 4 2 0 2 4 6
numbers: for example, + = .
0 + 3 9 6 3 0 3 6 9

Main lesson activity


Introduce the class to the table below, which gives the rules for combining two
scatter graphs, which have a common axis, to obtain the resulting correlation.
Positive Negative
correlation No correlation correlation
Positive correlation Positive No correlation Negative
No correlation No correlation Cannot tell No correlation
Negative correlation Negative No correlation Positive

As can be seen from the table, the resulting graph can have its axes in either
order, as this does not affect the correlation.
An easy way to remember these rules is by comparing them with the rules for
multiplying together positive and negative numbers, as shown below.
Multiply () + 0
+ + 0
0 0 The exception 0
0 +
Combining two graphs showing no correlation can be misleading, as the answer
could be a graph with either positive, negative or no correlation. Hence you
cannot tell just by using the rules.
Take, for example, the case of the fish caught off Rhyl.
Hours of sunshine

Hours of sunshine
Temperature (C)

Number of fish caught daily off Rhyl Number of fish caught daily off Rhyl Temperature (C)

In this example, the two left-hand graphs show no correlation but combining
hours of sunshine with temperature gives a positive correlation.

58 HarperCollinsPublishers Ltd 2003


9B3LP_05.qxd 18/09/2003 15:19 Page 59

Tell the class that 12 students marks were collected from their tests in
three different subjects, and two scatter graphs created. Then put these
graphs on the board.
Ask the class to tell you what the subject of each test could be.
Then tell the class that they are actually Mathematics (Test A), Science

Test B
(Test B) and Art (Test C).
Invite them to discuss the correlation between the Mathematics and
Science scores, and between the Mathematics and Art scores.
Now prompt the class to tell you the correlation between the Science and
the Art scores. In this case, there would be negative correlation. Test A
Now ask them to look at the rules for combining two correlation graphs
in Pupil Book 3. They could copy into their books the table for
correlations and the table for multiplying together positive and negative
numbers.
Point out how similar the rules are, but emphasise the exception to the

Test C
rule, namely, two graphs, each showing no correlation, do not necessarily
mean that the derived graph would have no correlation.
The class can now do Exercise 5B from Pupil Book 3.

Exercise 5B Answers Test A

1 Correlation between Q and R Key Words


a Negative d Cannot tell g Negative
b No correlation e Positive h No correlation
scatter graph
c Positive f No correlation i No correlation
correlation
2 a Positive correlation b No correlation c No correlation positive
3 a Negative correlation b Negative correlation c Positive correlation
correlation
negative
Plenary correlation
Finish the lesson with a short test of multiplications of positive and negative no correlation
whole numbers, to reinforce the rules for combining two scatter graphs.
1 +8 3 2 6 5 3 +11 0 4 13 13 5 +8 6
6 9 4 7 0 +14 8 +15 15 9 +14 +14 10 7 10
Answers
1 24 2 30 3 0 4 169 5 48
6 36 7 0 8 225 9 196 10 70
Homework

1 The test results of ten students are recorded for four different subjects. Here are the results.
Student French Spanish English Music
A 45 52 63 35
B 64 60 56 45
C 22 30 46 58
D 75 80 70 30
E 47 60 55 42
F 15 24 40 50
G 80 74 68 42
H 55 65 53 48
I 85 77 75 41
J 33 47 51 50
a Plot the data for French and Spanish on a scatter graph.
b Describe the relationship between French and Spanish.
c Plot the data for English and Music on a scatter graph.
d Describe the relationship between English and Music.
e Plot the data for Spanish and English on a scatter graph.
f Describe the relationship between Spanish and English.
g Use your answers to parts d and f to state the correlation between Music and Spanish.

Answers
b Positive correlation d Negative correlation f Positive correlation g Negative correlation

HarperCollinsPublishers Ltd 2003 59


9B3LP_05.qxd 18/09/2003 15:19 Page 60

LESSON Framework objectives Scatter graphs and lines of best fit


Select, construct and modify, on paper and using ICT, suitable representation to
5.3 progress an enquiry, including scatter graphs to develop further understanding of
correlation, lines of best fit by eye, understanding what they represent.

Oral and mental starter


Using students Show me white boards or sheets of paper, ask the class to draw
and show you a sketch illustrating positive correlation.
Ask a student to give an example of two variables having positive correlation.
Ask those who think that their sketches show strong positive correlation, to keep
showing their sketches.
Discuss different strengths of positive correlations.
Repeat for negative correlation.
Briefly talk about no correlation.

Main lesson activity


Continuing from the Oral and mental starter, ask a student to draw a strong
positive correlation on the board or OHP.
Introduce the idea that it could represent the test results for Paper 1 and Paper 2
of the SATs. Label the axes with graduation marks and Paper 1 and Paper 2.
Tell them that a student was absent for the second paper but, although they
could not have an official level, the school wanted to know how the student
might have performed.
Ask the class to describe the trend. Students who do well on Paper 1 usually do
well on Paper 2.
Explain that in order to make an accurate guess at the missing students Paper 2
score, you are going to put a trend line (called a line of best fit) on the scatter
graph. Discuss where the line should go. The Pupil Book suggests using a ruler
to draw a line between the plotted points, passing as close as possible to all of
them, but you could also show them the idea of covering the points with a
finger. In this case the line of best fit will run down the middle of the finger.
Draw in a line of best fit. It is important to stress the need to use a ruler and also
that different students would have slightly different lines of best fit.
Now ask a student to come out and read off the Paper 2 score.
The class can now do Exercise 5C from Pupil Book 3.

60 HarperCollinsPublishers Ltd 2003


9B3LP_05.qxd 18/09/2003 15:19 Page 61

Exercise 5C Answers

1 c Darren, as the point representing him is not close to the line of best fit.
2 c The older they are, the closer they live to the shops.
3 c Answers will vary according to line of best fit drawn but should be approximately
12 14.

Extension Answers

Explanations which imply that it is not sensible to extrapolate lines of best fit outside the
range of the original data.
In Question 2, the distance which a child lives from the shops will depend on the parent.
So, the age of the child is not a factor in the question as he/she is not the homeowner. In
Question 3, they could imply that a 20-year-old may be working and have a much
greater income.

Plenary Key Words


Look at an example and discuss the problem of trying to extrapolate from a line
of best fit. You could use the questions posed in the extension.
scatter graph
Stress that students will need to be able to explain, for specific examples, correlation
reasons why it might not be valid to extrapolate data outside the given range. line of best fit
positive
correlation
negative
correlation
interpret
Homework

1 The table shows the scores of some students in a music exam and in a maths exam.
Student A B C D E F G H I J
Music 35 48 72 23 76 51 45 60 88 17
Maths 42 57 80 32 65 69 50 71 94 25
a Plot the data on a scatter graph. Use the x-axis for the music exam scores, from 0 to 100, and the
y-axis for the maths exam scores, from 0 to 100.
b Draw a line of best fit.
c One person did not do quite as well as expected on the maths test. Who do you think it was?
Give a reason.
2 A survey is carried out to compare the ages of people with the reaction time in a test.
Age (years) 45 62 83 24 76 63 44 42 37 50
Reaction time (seconds) 0.15 0.31 0.58 0.20 0.62 0.43 0.21 0.25 0.18 0.49
a Plot the data on a scatter graph. Use the x-axis for the range of ages, from 0 to 90 years, and the
y-axis for reaction times, from 0 to 1 seconds.
b Draw a line of best fit.
c Use your line of best fit to estimate the reaction time of a 30-year-old.
d Explain why it would not be sensible to use the line of best fit to predict the reaction time of
someone aged 100.

Answers
1 c Student E, as their point is not as close to the best fit line as the other students points
2 c Answers will vary according to line of best fit drawn but should be approximately 0.2
d Line of best fit is for range 2483 years old. 100 years old is outside this range, so correlation might not
continue to be linear

HarperCollinsPublishers Ltd 2003 61


9B3LP_05.qxd 18/09/2003 15:19 Page 62

LESSON Framework objectives Time series graphs


Select, construct and modify, on paper and using ICT, suitable graphical
5.4 representation to progress an enquiry, including line graphs for time series. Identify
key features present in the data.

Oral and mental starter


Ask individual students to give different units of time. You may need to give an
example, such as months.
Write any correct answers on the board or OHP.
For each example, draw a horizontal scale and put a few labels on it, as shown
on the right. January February March
Other examples which they may offer could include year, season, hour and days
of the week.

Main lesson activity


Tell the class that the aim is to look at different types of graph involving time.
Explain that for all the types of graph which they are going to look at or produce,
the time axis is always the horizontal axis.
Invite the class to look at Graph 1 in Pupil Book 3 (mean temperature difference
from normal for the UK in 2002).
Ask them to give some facts from the graph. For example, in 10 out of 12
months, the mean temperature was above normal; February and March showed
the greatest differences. You could prompt them by asking what they can tell you
about February and March.
Then ask the students to look at the winter months and the summer months and
compare the weather. They should observe that there are bigger temperature
changes in the winter months.
Now tell the class that you want them to look at the other graphs in Pupil
Book 3. This activity could be done individually or in small groups. Suggest that
they write down any key features of the graphs.
The class can now do Exercise 5D from Pupil Book 3.

62 HarperCollinsPublishers Ltd 2003


9B3LP_05.qxd 18/09/2003 15:19 Page 63

Exercise 5D Answers

1 a Time becomes shorter b 34 c After third bounce


d In theory, the ball never comes to rest. In practice, it would eventually stop
bouncing
2 a Mean temperature was exceeded on 10 months out of 12. Extreme changes in
temperature were greater above normal (about 3.3C) than below normal (about
1.2C)
b As data is for only one year, this could be exceptional. Data would need to be
recorded over at least 10 years for valid conclusions to be drawn.
3 a June b September
c Different pattern for average rainfall each month and different pattern for number
of days of rain each month
d Perth (approx values) 3 + 3 + 4 + 7 + 13 + 17 + 18 + 16 + 14 + 9 + 7 + 4 = 115.
Brisbane (approx values) 13 + 14 + 14 + 11 + 10 + 7 + 7 + 6 + 7 + 10 + 10 + 11
= 120. So, Brisbane has 5 more days of rainfall.

Plenary Key Words


Choose a graph and ask a group to list some key features.

time series
Ask the other students to add to it.
Explain that when there are two similar graphs, such as two rainfall graphs, it is graphs
important to compare them, looking for both similarities and differences. raw data
key features
line graphs
Homework

1500
Visitors (1000)

1000

500

0
Nov Dec Jan Feb Mar Apr May Jun Jul Aug Sep Oct Nov
2001 2002

North American Western Europe

Write a brief report on the similarities and differences between the visits from the UK for North
America and Western Europe. Make at least three statements. Try to give reasons for your answers.

Answers
More UK people visit Western Europe than North America (three to four times more), probably because Western
Europe is nearer, so the costs are less. There is greater variation in the number of visitors for Western Europe from
month to month (April being the most popular month). July and August are the most popular months to visit North
America

HarperCollinsPublishers Ltd 2003 63


9B3LP_05.qxd 18/09/2003 15:19 Page 64

LESSON Framework objectives Two-way tables


Design and use two-way tables.
5.5
Oral and mental starter
Put the class into small groups of four or five students.
Give the groups a mental test of ten questions. Appoint one person from each
group as team captain to record a teams answer to each question.
After the answers have been given, ask the team captains to record their answers
in a two-way table on the board or OHP, using ticks for correct answers, as shown.
Group Q1 Q2 Q3 Q4 Q5 Q6 Q7 Q8 Q9 Q10
1
2
3

Test
1 4.99 4 2 25% of 60 3 12 of a 12
4 600 4000 5 72 0.2 6 What is the HCF of 36 and 48?
7 Write down one answer to x2 + x = 0 8 Increase 132 by 20%
9 What is the square root of 196? 10 Give both solutions to (5 + x)2 = 81
Answers
1 19.96 2 15 3 14 4 2 400 000
5 360 6 12 7 x = 0 or x = 1
8 158.40 9 14 or 14 10 x = 4 and x = 14

Main lesson activity


Keeping the students in their groups, explain that their task is to collect data from
the whole class and record it.
Use the data collection sheets as shown below, or the students can design their own.
One student from each group is the collector, the rest of the group are the
informers. Each collector goes from group to group collecting their data.
Favourite subject Favourite colour Favourite TV programme
Boys Girls Boys Girls Boys Girls
English Blue Soap
Science Red Drama
Art Yellow Cartoon
Maths Green News
Other

Favourite music Favourite food Favourite hobby


Boys Girls Boys Girls Boys Girls
Rock Chips Sport
Pop Salad Computer
Dance Pizza Music
RnB Burger

Now use other combinations to form different two-way tables. For example:
Favourite colour
Blue Red Yellow Green
Sport
Favourite Computer
hobby Music

64 HarperCollinsPublishers Ltd 2003


9B3LP_05.qxd 18/09/2003 15:19 Page 65

Having collected their data, the students can record it in their books. In each case, ask the students to pick out
a key feature. A key feature could be that the data appears random (no relationship between the two variables).
The class can now do Exercise 5E from Pupil Book 3.

Exercise 5E Answers

1 a b Boxed toys are worth more than unboxed toys


Condition Difference between boxed and not boxed but the percentage difference in value reduces
Excellent 100% 60% = 40% as the condition deteriorates.
Very good 80% 50% = 30%
Good 60% 40% = 20%
Average 40% 25% = 15%
Poor 20% 10% = 10%

2 a For the age range 10 to 12, a larger percentage of boys have mobile phones. b As the boys and girls get older, both
For the age range 13 to 15, a larger percentage of girls have them. percentages increase.
3 In June, July and August, 252 birthdays but in November, December and January, 228 birthdays. This would support
the claim. Answers may vary depending on how the data is analysed but the conclusion should be the same.
4 160 cm and above: 20 boys but only 16 girls. This would support the claim. Answers may vary depending on how the
data is analysed but the conclusion should be the same.

Extension Answers
Key Words
a 9
40 b 31

80 c 1
10 d 67

80 e 11

16
two-way table
relationship
Plenary data
Ask the class to select a table where they saw a relationship. tally
Look at, for example, boys favourite colour and boys favourite music. Are their frequency
responses different from girls?
Write any relationships on the board. Ask the class what they could do to test
whether the results were representative of the school.
Homework

1 Two fair spinners are spun and the scores are 2 A year group recorded the days of the week on
added together to get a total score. This is which they were born. Here are the results.
recorded in the two-way table, shown below.
Day Boys Girls
Monday 23 19
1 1 Tuesday 19 25
Wednesday 27 28
2
4

Thursday 31 26
3

3
Friday 35 41
Saturday 14 17
Second spinner Sunday 12 11
+ 1 2 3 Total 161 167
1 2 3 a Write a comment on the births of boys and
First
2 3 girls.
spinner
3 b Write a comment about the number of births
4 on different days of the week.
a Complete the table of total scores. Answers
b List all the total scores which are prime 1 a
Second spinner
numbers.
+ 1 2 3
c State the most likely total scores. 1 2 3 4
d Write down the probability of getting a total First
2 3 4 5
score of 7. Give your answer as a fraction in spinner
3 4 5 6
its simplest form. 4 5 6 7
e Write down the probability of getting a total 1 e 14
b 2, 3, 5 and 7 c 4 and 5 d 12
score of 5. Give your answer as a fraction in 2 a Each day, number of births of boys is close to that
its simplest form. of girls
b Fewer births on Saturdays and Sundays

HarperCollinsPublishers Ltd 2003 65


9B3LP_05.qxd 18/09/2003 15:19 Page 66

LESSON Framework objectives Cumulative frequency diagrams


Find the median and quartiles for large data sets. Estimate the median and
5.6 interquartile range of a large set of grouped data.

Oral and mental starter


Write a blank table on the board or OHP.
Ask the class to indicate if their birthday is in January. Record the number in the
table.
Repeat for February and so on for the whole year.
Now ask them to look at the table and tell you how many students have a
birthday in the first month of the year, the first two months of the year, the first
three months and so on. Add extra columns to your table and record the data as
shown.
Prompt the students to explain how they worked out each answer.
Complete the cumulative frequency column in the table. For example:
Month Number of students Birthdays Cumulative frequency
January 5 First month 5
February 7 First two months 12
March 2 First three months 14

Main lesson activity


Continuing from the Oral and mental starter, explain to the class they are about
to learn how to estimate medians and quartiles for large sets of grouped data.
Draw the axes for a cumulative frequency graph on the board. Write Month
along the horizontal axis and Cumulative frequency along the vertical axis.
Plot the origin and say this is the beginning of January so there are no birthdays.
Discuss with the class where they think the next point should be plotted. Use the
data obtained in the starter.
Stress that in order to include, for example, all five students, you must plot at the
end of January and so on the upper class boundaries.
Complete the graph and ask the class to copy the tables and the graph onto
graph paper, choosing suitable scales so that it uses most of the page.
Now ask the class whether they think it would be better to join the points using
straight lines or a curve. Explain that they can use either unless a question
specifies polygon or curve.
If some students are waiting for others, they can start to draw the graphs from
Exercise 5F.
n n 3n
Explain that because there is a large set of data, you can use , , , where n
2 4 4
is the total frequency, to read off the values of the median, lower quartile and
upper quartile. Show the class how to do this on the board and then ask them to
read off their values.
Write on the board Median = , Lower quartile = , Upper quartile =
and ask them to copy and fill in their answers.
Now ask the class to compare their answers. There may be slight differences.
Explain that the slight differences in the graphs and the fact that they are using
grouped data are why their results are estimates.
Finish off by giving the class the formula for finding the interquartile range:
Interquartile range = Upper quartile Lower quartile
Write Interquartile range = And ask them to work out their value and write it
down.
The class can now do Exercise 5F from Pupil Book 3.

66 HarperCollinsPublishers Ltd 2003


9B3LP_05.qxd 18/09/2003 15:19 Page 67

Exercise 5F Answers

1 a Temperature, T (C) Cumulative frequency


T 5 8
T 10 23
T 15 65
T 20 90
T 25 100
c Median = 13.2 C, IQR = 17 10.2 = 6.8 C
2 a Time, t (min) Cumulative frequency
t 5 5
t 10 17
t 15 26
t 20 33
t 25 37
t 30 40 Key Words
c Median = 11.7 min, IQR = 17.9 7.1 = 10.8 min
3 a Mass, M (kg) Cumulative frequency cumulative
M 5 0 frequency
M 10 15
M 15 46 upper class
M 20 68 boundary
M 25 80
quartile
c Median = 14 kg, IQR = 18.2 10.8 = 7.4 kg
lower quartile
upper quartile
interquartile
Plenary range
Look back at one of the questions for example, the birthday data and ask the polygon
students what the median tells them. Ask those with a birthday before the median curve
date to put up their hands. Hopefully, this will be approximately half the class.
Repeat this for the quartiles and for students with a birthday between the lower and upper quartiles.
Homework

For each table of data:


a Copy and complete the cumulative frequency table.
b Draw the cumulative frequency graph.
c Use your graph to estimate the median and the interquartile range.
1 The height of 100 plants.
Height, h (cm) Number of plants Height, h (cm) Cumulative frequency
0 < h 10 6 h 10
10 < h 20 24 h 20
20 < h 30 27 h 30
30 < h 40 30 h 40
40 < h 50 13 h 50
2 The time that the school bus is late on 40 days.
Time, t (min) Number of days Time, t (min) Cumulative frequency
0<t 5 12 t 5
5 < t 10 15 t 10
10 < t 15 6 t 15
15 < t 20 7 t 20

Answers
1 a Height, h (cm) Cumulative frequency 2 a Time, t (min) Cumulative frequency
h 10 6 t 5 12
h 20 30 t 10 27
h 30 57 t 15 33
h 40 87 t 20 40
h 50 100 c Median = 7.7 min, IQR = 12.5 4.2 = 8.3 min
c Median = 27.4 cm, IQR = 36 17.9 = 18.1 cm

HarperCollinsPublishers Ltd 2003 67


9B3LP_05.qxd 18/09/2003 15:19 Page 68

LESSON Framework objectives Estimation of a mean from grouped


data
5.7 Estimate the mean of a large set of grouped data.

Oral and mental starter


Introduce a counting stick, as shown below.

Tell the class that, for example, one end is 12 and the other end is 16. Then ask them the value of the mid-
point.
Repeat this, increasing the level of difficulty each time. For example, using fractions and decimals.
Now give the class the end values. Tell them that you want, for example, the product of eight and the mid-
values. So, using 12 and 16, the answer would be 8 14 = 112.
Repeat.

Main lesson activity


Tell the class that they are going to be estimating the means of large sets of grouped data.
Remind them of the key words which they have met before in this context, such as mid-value, frequency,
mean, estimate and so on. You could ask the students for definitions.
Put the numbers 10, 12, 12, 12 on the board and ask the class to tell you the mean. Hopefully, they will say
11.5.
Now write the same data in a grouped frequency table and ask the same question.
Frequency
10 1
12 3
Clearly, you should get the same answer, but students may suggest the answer 11 (as it is half way between
10 and 12).
Point out that every piece of data has to be counted, so there are three 12s (36) and a 10 (total 46). This is then
divided by 4 (total frequency) to give the answer 11.5.
Now refer them to the example in Pupil Book 3, page 96. Point out that, as in the last section on cumulative
frequency, any results will be estimates because you do not know the original (individual) data as we are
dealing with grouped data.
Explain that in order to estimate the mean, we have to use mid-values as these offer the best guess of the value
of the average of the data in a class interval.
Work through the example to show the class how to estimate the mean using a table with four columns, as
shown below.
Time, t (seconds) Frequency, f Mid value, x, of time fx
(seconds) (seconds)
13 < T 14 12 13.5 162
14 < T 15 21 14.5 304.5
15 < T 16 39 15.5 604.5
16 < T 17 20 16.5 330
17 < T 18 8 17.5 140
Total = 100 Total = 1541
Show the students how they can now calculate an estimate of the mean.
1541
The estimate of the mean time = = 15.41 seconds.
100
The class can now do Exercise 5G from Pupil Book 3.

Exercise 5G Answers

1 c i 4.5 kg ii 27.35 cm
2 19.8 C
3 8.4 min (8 min 24 s)

68 HarperCollinsPublishers Ltd 2003


9B3LP_05.qxd 18/09/2003 15:19 Page 69

Extension Answers

1 i 4<M6 ii 24 < L 28 2 20 < T 30 3 7<t9

SATs Answers

1 a False (1988 725, 1998 500) b Cannot be certain, trend may not continue
2 a 65 b 30 and 50 c Game A and Game B positive relationship, Game A and Game C no relationship
d Game B and Game C no relationship
3 a Positive correlation (wider the diameter, higher the tree)
b Point if plotted on scatter graph is not near to line of best fit
c Approximately 5.2 m d False, false, false, false
4 a 4.85 g b 0.1 g c Point at 12.30 pm. Bird is under the mass to survive, as furthest away from line of best fit.
5 a A b Approximately 350 h
6 a Old 7 x < 8, new 6 x < 7
b For example, the new version has shorter sentences: that is, there are more words per sentence in old version.
c 20% 30%
7 a 30.50 32 b 83% c 9.50 11.50 e A true, B and C false
8 a 28 years b 16 to 18 years c Younger people, on average, went to the theme park

Plenary Key Words


Discuss the effect of extreme values on a mean.
grouped data
Ask the class to give you a definition of an extreme value. Point out that
sometimes these are called rogue values and sometimes they are called outliers. estimate
Remind the class that when tables contain peoples ages, for example, 02 years, mean
would have a mid-value of 1.5 years. (They will need this information to enable median
them to do the homework.) mid-class value
Remind the class how to find which class interval the median lies in and refer frequency table
them to the Extension Work. They may wish to do this as an oral exercise.
Homework

Copy and complete each table of values given below.


a Complete each table including the totals.
b Calculate an estimate of each mean.
1 Age, A (years) Frequency, f Mid-value, x, of age (years) f x (years)
1112 5 12 60
1314 8
1516 12
1718 5
Total = Total =

2 Time, t, (hours) Frequency, f Mid-value, x, of time (hours) f x (hours)


0<t2 2 1 2
2<t4 7
4<t6 10
6<t8 5
Total = Total =

Answers
1 a Age, A, Frequency, f Mid-value, fx 2 a Time, t, Frequency, f Mid value, fx
(years) x, of age (years) (hours) x, of time (years)
(years) (hours)
1112 5 12 60 0<t2 2 1 2
1314 8 14 112 2<t4 7 3 21
1516 12 16 192 4<t6 10 5 50
1718 5 18 90 6<t8 5 7 35
Total = 30 Total = 454 Total = 24 Total = 108
b Estimate of mean = 15.13 years b Estimate of mean = 4.5 hours

HarperCollinsPublishers Ltd 2003 69


9B3LP_06.qxd 18/09/2003 15:19 Page 70

CHAPTER
6 Shape, Space and Measures 2
LESSON Framework objectives Similar triangles
Find points that divide a line in a given ratio, using the properties of similar
6.1 triangles.

Oral and mental starter


Invite the class to imagine two 10p coins, one of which is held still while the other coin is rolled around it so
that the coins are always in contact.
Now, ask the class how many revolutions the moving coin will make before it returns to its starting position.
(Answer: Two)

Main lesson activity


Remind the class about the properties of an enlargement by showing them how a triangle is enlarged by a scale
factor of 2.
A A

B C

B C

Triangle ABC has been mapped onto triangle ABC by an enlargement of scale factor 2.
Under an enlargement all of the angles are the same size and corresponding sides are in the same ratio.
So, AB : AB = AC : AC = BC : BC = 1 : 2.
AB AC BC
This can also be written as = = = 2.
AB AC BC
Explain to the class that the two triangles are said to be similar.
Two triangles are similar if their angles are the same size or their corresponding
sides are in the same ratio.
Explain that only one of these conditions is required to show that the triangles
are similar.
Show the class how to use similar triangles by completing the following three
examples. Some revision on parallel lines may be required.
X
Example 1
Show that the two triangles on the right are similar. A
In triangle ABC, C = 80 (the sum of the angles in a
48
triangle = 180) and in triangle XYZ, X = 48 (the sum of
the angles in a triangle = 180).
Since the angles in both triangles are the same, triangle 52 80 52
Y Z
ABC is similar to triangle XYZ. B C
Example 2
Triangle ABC is similar to triangle DEF. Calculate the length of the side DF.
Let the side DF = x.
Since the triangles are similar, corresponding sides are D
in the same ratio. A
DE EF DF
So, = = = 3. 6 cm 12 cm x
AB BC AC 4 cm
x 15
Therefore, = = 3. So, x = 18 cm.
6 5 B 5 cm C
E 15 cm F

70 HarperCollinsPublishers Ltd 2003


9B3LP_06.qxd 18/09/2003 15:19 Page 71

Example 3 A
In the triangle, EB is parallel to DC. Calculate the length of the side DC.
AEB = ADC (corresponding angles in parallel lines) and ABE = ACD 4 cm
3 cm
(corresponding angles in parallel lines). So, triangle AEB is similar to triangle ADC E B
(since A is common to both triangles). 4 cm
Let the side DC = x.
Since triangle AEB is similar to triangle ADC, the corresponding sides are in the same D C
x
ratio.
DC AC
So, = = 2.
EB AB
x 8
Therefore, = = 2. So, x = 6 cm.
3 4
The class can now do Exercise 6A from Pupil Book 3.

Exercise 6A Answers

1 a Yes b No c Yes d No 2 a 3 equal angles b 8 cm 3 a 3 equal angles b 12 cm


4 a PST = PQR (corresponding angles), PTS = PRQ (corresponding angles), P common b 6 cm
5 a BAE = AED (alternate angles), ABD = BDE (alternate angles), ACB = DCE (vertically opposite angles)
b 6 cm 6 12 cm 7 40 m 8 90 cm

Extension Answers

1 a Triangle ABC and triangle ADE, 9 cm b Triangle PQR and triangle STR, 6 cm
c Triangle XYZ and triangle VWY, 5 cm d Triangle JKL and triangle JMN, 5 cm

Plenary Key Words


Ask the class to explain the two conditions needed to show that two triangles are
similar
similar.
Check that the students understand the difference between similar triangles and similarity
congruent triangles.
Homework

1 State whether each of the pairs of triangles below are similar.


a b 30
87
42 51 46 30
42 114

c 10 cm d
9 cm
5 cm 6 cm 6 cm
2 cm 4 cm
4 cm
8 cm
4 cm 9 cm
2cm
2 a Explain why triangle ABC is similar to triangle PQR. C
12 cm
b Find the length of the side QR. R
A 57 Q
35 88
3 In the triangle below DE is parallel to BC. Find the 88 6 cm
length of BC. 9 cm P
B
A

4 cm
D E
6 cm Answers
6 cm 1 a Yes b No c Yes d No
2 a 3 equal angles b 8 cm
B C 3 15 cm

HarperCollinsPublishers Ltd 2003 71


9B3LP_06.qxd 18/09/2003 15:19 Page 72

LESSON Framework objectives Metric units for area and volume


Use units of measurement to calculate, estimate, measure and solve problems in a
6.2 variety of contexts; convert between area measures (mm2 to cm2, cm2 to m2, and
vice versa) and between volume measures (mm3 to cm3, cm3 to m3, and vice
versa).

Oral and mental starter


Multiplying and dividing by 10, 100, 1000 and 10 000 will be revised.
On the board, draw the grid on the right, or use a prepared OHT.
35 0.0035 3500
Tell the class that the starting number is 3.5.
Ask individual students to point to the number that is:
3.5 10 3.5 100 3.5 1000 3.5 10 000 0.000 35 3.5 0.35
Then ask the class to explain the rules for multiplying by powers of 10.
Next, ask individual students to point to the number that is:
350 0.035 35 000
3.5 10 3.5 100 3.5 1000 3.5 10 000
Finally, ask the class to explain the rules for dividing by powers of 10.

Main lesson activity


Tell the class that the lesson is about converting the metric units of area, volume and capacity.
Remind them that the metric units for area are: the square millimetre (mm2),
the square centimetre (cm2) and the square metre (m2).
Draw two squares on the board and explain why 1 cm 10 mm
1 cm2 = 100 mm2.
Draw another two squares on the board and explain to the
1 cm 1 cm2 10 mm 100 mm2
class why 1 m2 = 10 000 cm2.
The students can now copy the following into their books:
Metric units of area
100 mm2 = 1 cm2
1m 100 cm
10 000 cm2 = 1 m2
2
10 000 m = 1 hectare (ha)
1 hectare = 100 ares 1m 1 m2 100 cm 10 000 cm2
Note that, for measuring the area of fields, the m2 is too small,
while the km2 is too large. Hence, a more conveniently sized
unit is used the are, which is 100 m2. Land area is usually
given in units of 100 ares, where 100 ares = 1 hectare.
Remind the class that the metric units for volume are: the cubic millimetre (mm3),
the cubic centimetre (cm3) and the cubic metre (m3).
Draw two cubes on the board and explain why 1 cm 10 mm
1 cm3 = 1000 mm3. 1 cm 10 mm
Draw another two cubes on the board and explain why
1 m3 = 1 000 000 cm3.
The students can now copy the following into their books: 1 cm 1 cm3 10 mm 1000 mm3
Metric units of volume
1000 mm3 = 1 cm3
1 000 000 cm = 1 m3
3

Remind the class that the metric units for capacity 1m 100 cm
1m 100 cm
are: the litre (l), the centilitre (cl) and the
millilitre (ml). They can now copy the following into
their books:
Metric units of capacity 1m 1 m3 100 cm 1 000 000 cm3
1 m3 = 1000 litres
1000 cm3 = 1 litre
1 cm3 = 1 millilitre
Stress the following:
To convert from a large unit to a smaller unit, always multiply by the conversion factor.
To convert from a small unit to a larger unit, always divide by the conversion factor.

72 HarperCollinsPublishers Ltd 2003


9B3LP_06.qxd 18/09/2003 15:19 Page 73

Show the class the following examples of conversion.


35 000 cm2 to m2 0.35 cm3 to mm3 3500 cm3 to litres
2
35 000 cm = 35 000 10 000 = 3.5 m 2

0.35 cm3 = 0.35 1000 = 350 mm3


3500 cm3 = 3500 1000 = 3.5 litres

The class can now do Exercise 6B from Pupil Book 3.

Exercise 6B Answers

1 a 40 000 cm2 b 70 000 cm2 c 200 000 cm2 d 35 000 cm2 e 8000 cm2
2 a 200 mm2 b 500 mm2 c 850 mm2 d 3600 mm2 e 40 mm2
3 a 8 cm2 b 25 cm2 c 78.3 cm2 d 5.4 cm2 e 0.6 cm2
4 a 2 m2 b 8.5 m2 c 27 m2 d 1.86 m2 e 0.348 m2
5 a 3000 mm3 b 10 000 mm3 c 6800 mm3 d 300 mm3 e 480 mm3
6 a 5 m3 b 7.5 m3 c 12 m3 d 0.65 m3 e 0.002 m3
7 a 8 litres b 17 litres c 0.5 litre d 3000 litres e 7200 litres
8 a 8.5 cl b 120 cl c 84 ml d 4.5 litres e 2400 ml
9 160 10 a 10 800 m2 b 1.08 hectares 11 150 litres 12 6 days 13 500

Extension Answers
Key Words
1 250 2 a 1296 square inches b 46 656 cubic inches
3 4840 square yards 0.405 hectares square millimetre
square
centimetre
Plenary square metre
hectare
Write the following on the board and ask the students to fill in the blanks.
1 _____ mm2 = 1 cm2 4 _____ mm3 = 1 cm3 6 _____ litres = 1 m3 cubic millimetre
2
2 _____ cm = 1 m 2 3
5 _____ cm = 1 m 3 7 _____ cm3 = 1 litre cubic centimetre
2
3 _____ m = 1 hectare (ha) cubic metre
Answers 1 100 2 10 000 3 10 000 4 1000 5 1 000 000 6 1000 litre
7 1000
Homework

1 Express each of the following in mm2.


a 3 cm2 b 8 cm2 c 4.5 cm2 d 0.8 cm2
2 Express each of the following in m2.
a 40 000 cm2 b 70 000 cm2 c 32 000 cm2 d 5000 cm2
3 Express each of the following in cm3.
a 2 m3 b 9 m3 c 3.7 m3 d 0.3 m3
4 Express each of the following in litres.
a 8000 cm3 b 12 000 cm3 c 23 500 cm3 d 250 cm3
5 A rectangular park is 620 m long and 340 m wide. Find the area of the 25 cm
park in hectares.
6 Calculate the volume of the box on the right. Give your answer in litres. 10 cm
40 cm

Homework Answers
1 a 300 mm2 b 800 mm2 c 450 mm2 d 80 mm2 2 a 4 m2 b 7 m2 c 3.2 m2 d 0.5 m2
3 a 2 000 000 cm3 b 9 000 000 cm3 c 3 700 000 cm3 d 300 000 cm3
4 a 8 litres b 12 litres c 23.5 litres d 0.25 litre 5 21.08 hectares 6 10 litres

HarperCollinsPublishers Ltd 2003 73


9B3LP_06.qxd 18/09/2003 15:19 Page 74

LESSON Framework objectives Length of an arc and area of a sector


Know and use the formulae for length of arcs and area of sectors of circles.
6.3

Oral and mental starter


Revise the names and spelling of the various parts of a circle.
First, ask the students to sketch a circle on their whiteboards or in their books.
Then ask them to draw and label all the different parts of a circle which they can
remember.
Ask individual students to show the answers on their whiteboards, or to write
separate answers on the board. Check their spelling.
The following terms should be covered: centre, radius, diameter, arc,
circumference, chord, sector, segment, and tangent.

Main lesson activity


Remind the class how to calculate the circumference and the area of a circle
using the formulae:
C = d = 2r
A = r2
r
O

Remind them that = 3.142 or they can use the key on their calculator.
Draw the diagram below on the board or on a prepared OHT.
arc Explain that the arc, AB, is part of the circumference,
A B and that the sector, AOB, is a slice of the circle
sector
enclosed by the arc AB and the radii OA and OB.
AOB is the angle of the sector and is usually denoted
O by the Greek letter (pronounced theta).


Explain that the length of the arc AB as a fraction of the circumference is .
360

So, the length of the arc AB = d.
360

Similarly, the area of the sector AOB = r2
360
Complete the following example, making sure that the students obtain the
correct answers using their calculators.
Calculate: a the length of the arc AB. B
A
b the area of the sector AOB.
Give the answers to three significant figures. 30
8 cm
30
a Length of the arc AB = 16 = 4.19 cm (3 s.f.). O
360
30
b Area of the sector AOB = 82 = 16.8 cm2 (3 s.f.).
360
The class can now do Exercise 6C from Pupil Book 3.

74 HarperCollinsPublishers Ltd 2003


9B3LP_06.qxd 18/09/2003 15:19 Page 75

Exercise 6C Answers

1 i a 10.5 cm b 52.4 cm2 ii a 5.50 cm b 19.2 cm2 iii a 4.19 cm


b 25.1 cm2 iv a 5.24 m b 10.5 m2 v a 5.24 m b 6.54 m2
vi a 18.8 m b 67.9 m2
2 a 23.5 m b 34.4 m2
3 109 mm
4 29 cm2

Extension Answers

a i 2 cm ii 6 cm2 b i 4 cm ii 32 cm2 c i 2 cm ii 12 cm2


d i 6 cm ii 27 cm2 e i 4 cm ii 6 cm2

Plenary Key Words


Draw this diagram on the board. A B arc
Ask the class to write down the formulae for the length
of the arc AB and the area of the sector AOB. circumference
Check the students answers. radius
O
sector

key on your calculator.


Homework

In this exercise take = 3.142 or use the


1 Calculate (i) the length of the arc and (ii) the area of the sector for each of the following circles.
Give your answers correct to three significant figures.

a b c

8 cm
30 10 cm
5 cm 40

135

2 Calculate the total perimeter of the sector on the right.


Give your answer correct to three significant figures.

12 cm 12 cm

45

3 Calculate the area of the sector below. Give your answer correct to three significant figures.
3.5 cm 3.5 cm
150

Answers
1 a i 2.62 cm ii 6.55 cm2 b i 5.59 cm ii 22.3 cm2 c i 23.6 cm ii 118 cm2
2 33.4 cm 3 16.0 cm2

HarperCollinsPublishers Ltd 2003 75


9B3LP_06.qxd 18/09/2003 15:19 Page 76

LESSON Framework objectives Volume of a cylinder


Calculate lengths, areas and volumes in right prisms, including cylinders.
6.4

Oral and mental starter


Show the class various 3-D shapes such as: cube, cuboid, square-based
pyramid, tetrahedron, triangular prism, cylinder, sphere.
Ask them to identify each shape and the spelling of each name. Write all the
names on the board.

Main lesson activity


Remind the class how to calculate the volume of a prism. The volume, V, of a
prism is found by multiplying the area A of its cross-section by its length l:
V = Al
l

Explain to the class that the cross-section of a cylinder is a circle with radius r.
So, the area of the cross-section is A = r 2. r
If the height of the cylinder is h, then the volume, V, for the cylinder is given by
the formula:
h
V = r2 h = r2h

Carefully go through the following example on the board, making sure that the
4 cm
students obtain the correct answer using their calculators.
Calculate the volume of the cylinder, giving the answer correct to three
significant figures. 7 cm
V = 42 7 = 352 cm3 (3 s.f.)

The class can now do Exercise 6D from Pupil Book 3.

76 HarperCollinsPublishers Ltd 2003


9B3LP_06.qxd 18/09/2003 15:19 Page 77

Exercise 6D Answers

1 a 1130 cm3 b 226 cm3 c 75.4 m3 d 37.7 m3 e 9.82 cm3


2 1062 mm3
3 cylindrical tin (a 2352 cm3 b 2300 cm3 c 2356 cm3)
4 a 35.3 litres b 141
5 a 1.51 m3 b 1131 litres
6 a 10 000 cm3 b 250 cm2 c 8.9 cm

Extension Answers

1 a 207 cm2 b 37.7 m2


2 r = 5.4 cm, h = 10.8 cm

Plenary Key Words


Draw a cylinder on the board.

cross-section
Ask the class to explain how the formula for the volume of a cylinder is derived.
cylinder
radius
volume

In this exercise take = 3.142 or use the key on your calculator.


Homework

1 Calculate the volume of each of the following cylinders. Give your answers correct to three
significant figures.
a b c
3 cm 8m
5 cm
4m
12 cm 2 cm

2 The diagram below shows a metal pipe of length 1 m. It has an internal diameter of 2.8 cm, and an
external diameter of 3.2 cm. Calculate the volume of metal in the pipe. Give your answer correct to
the nearest cubic centimetre.
1m

3.2 cm

2.8 cm

3 A cylindrical can holds 2 litres of oil. If the height of the can is 25 cm, calculate the radius of the
base of the can. Give your answer correct to one decimal place.

Answers
1 a 339 cm3 b 157 cm3 c 101 m3
2 188 cm3 3 5.0 cm

HarperCollinsPublishers Ltd 2003 77


9B3LP_06.qxd 18/09/2003 15:19 Page 78

LESSON Framework objectives Rates of change


Understand and use measures of speed (and other compound measures such as
6.5 density or pressure) to solve problems; solve problems involving constant or
average rates of change.

Oral and mental starter


Explain to the class that the starter is about converting minutes into fractions and decimals of an hour.
Copy the table below on the board or use a prepared OHT.
The first example has already been completed.
Time in minutes 30 minutes 15 minutes 45 minutes 20 minutes 6 minutes 10 minutes
1
Time as a fraction 2 hour
of an hour
Time as a decimal 0.5 hour
of an hour
Ask the students to copy and complete the table in their books or on their whiteboards.
Answers (Explain that sometimes the answers may have to be given to three decimal places.)
Time in minutes 30 minutes 15 minutes 45 minutes 20 minutes 6 minutes 10 minutes
1 41 hour 43 hour 31 hour 1 61
Time as a fraction 2 hour
10 hour hour
of an hour
Time as a decimal 0.5 hour 0.25 hour 0.75 hour 0.333 hour 0.1 hour 0.167 hour
of an hour

Main lesson activity


Explain to the class that a rate of change is the way of comparing how one quantity changes with another. Rates
of change can always be recognised because their units contain per or p or / which means for every.
Write the following examples of compound measures on the board:
Speed with units miles per hour (mph), kilometres per hour (km/h) or metres per second (m/s).
Density with units grams per cubic centimetre (g/cm3).
Fuel consumption with units miles per gallon (mpg) or kilometres per litre (km/l).
Explain that speed is the distance travelled per unit of time and that the relationship between speed, distance
and time can be expressed by the following three formulae:
Distance Distance
Speed = Distance = Speed Time Time =
Time Speed
When we refer to speed, we usually mean average speed, as it is unusual to maintain the same, exact speed in
one journey.
Draw the diagram below and explain that the relationships between distance D, time T and speed S can be
remembered by using this triangle:
Covering up the quantity you want to find leads to the three formulae:

D D D
S= D = ST T=
T S
S T

Show the students how to use the triangle by doing the following examples.
Example 1
A train travels at an average speed of 80 mph. Find the distance travelled by the train in 212 hours.
Using the formula D = ST, the distance travelled = 80 212 = 200 miles.
Example 2
A car travels 120 km on a motorway at an average speed of 80 km/h. Find the time taken for the journey.
D 120
Using the formula T = , the time taken = = 1.5 hours = 112 hours or 1 hour 30 minutes.
S 80
Explain that density is the mass of a substance per unit of volume and that the formula for density is:
Mass
Density =
Volume

78 HarperCollinsPublishers Ltd 2003


9B3LP_06.qxd 18/09/2003 15:19 Page 79

Draw the diagram below and explain that the relationships between density D, mass M and volume V can be
remembered by using this triangle:
Covering up the quantity you want to find leads to the three formulae:
M M
M D= M = DV V=
V D
D V

Show the students how to use the triangle by doing the following examples.
Example 1
The volume of a wooden block is 20 cm3 and its mass is 18 g. Find the density of the wood.
M 18
Using the formula D = , the density of the wood is = 0.9 g/cm3.
V 20
Example 2
Find the mass of a plastic model, which has a volume of 30 cm3 and a density of 1.6 g/cm3.
Using the formula M = DV, the mass of the model = 1.6 30 = 48 g.
The class can now do Exercise 6E from Pupil Book 3.

Exercise 6E Answers

1 a 240 miles b 180 miles c 30 miles d 40 miles 2 a 52.5 mph b 42 mph c 60 mph d 63 mph
3 a 3 hours b 2 hours 30 minutes c 3 hours 20 minutes d 1 hour 45 minutes
4 a 75 mph b 120 km/h c 125 miles d 225 km e 12.5 seconds f 3 hours 20 minutes
5 216 metres 6 600 mph 7 a 10 m/s b 25 m/s c 3313 m/s
8 156.8 g 9 5 litres 10 1.6 g/cm3

Extension Answers

1 42 mph 2 48 mph 3 17.1 km/h 4 450 g 5 a 90 miles b 20 gallons

SATs Answers

1 a 721 hours b 465 mph c 60 miles 2 a 60 mph b 30 mph c 40 mph


3 a 15 cm b 12 cm c Yes, the angles are the same 4 a B b A c 2.83 cm 5 0.72 cm

Plenary Key Words


Summarise the lesson by asking the class to write down the relationships between
density
distance D, time T and speed S by using the triangle at the top of the page.
Repeat for density. pressure
speed
Homework

1 Find the distance travelled by a hiker who walks for 3 hours at an average speed of 2.5 mph.
2 Find the time taken to drive a car 125 km at an average speed of 75 km/h.
3 A runner runs a 1000 m race in 3 minutes 20 seconds. Find his average speed in m/s.
4 Find the density of a gold ingot that has a mass of 4825 g and a volume of 250 cm3.
5 The density of sea water is 1.05 g/cm3. If a bucket with a capacity of 5 litres is filled with seawater,
find the mass of the water in the bucket. Give your answer in kilograms.
6 The density of cork is 0.25 g/cm3. Find the volume of a block of cork that has a mass of 120 g.

Answers
1 7.5 miles 2 1 hour 40 minutes 3 5 m/s 4 19.3 g/cm3 5 5.25 kg 6 480 cm3

HarperCollinsPublishers Ltd 2003 79


9B3LP_07.qxd 18/09/2003 15:20 Page 80

CHAPTER
7 Number 2
LESSON Framework objectives Standard form
Write numbers in standard form.
7.1 Use algebraic methods to convert a recurring decimal to a fraction in simple
cases.

Oral and mental starter


Use a target board, such as the one shown on the right, to do this exercise in
x3 x5 x4 x2
multiplying and dividing powers.
Point to two powers and ask a student to multiply or divide them. x8 x7 x7 x6
Ensure that the class know which way round to do the division.
x3 x6 x8 x4
Recall the rules for multiplying and dividing powers.
x2 x10 x9 x5

Main lesson activity


Students may have met the idea of standard form through extension activities.
Explain that a way is needed to express and use very large and very small
numbers without writing out many zeros. For example 3 400 000 000 can be
written as 3.4 109, and 0.000 072 can be written as 7.2 105.
Ask students if they can see the connection with the original number and the
power of 10.
Make sure they know and understand the definition of a standard form number:
A 10n, where 1 A < 10 and n is an integer.
Demonstrate how to convert ordinary numbers to standard form. For example:
540 000 = 5.4 105 0.000 005 89 = 5.89 106
89 630 000 = 8.963 10 7

The class may realise that this is a matter of moving the digits and counting the
number of places that they move.
Repeat with examples such as
23 106 = 2.3 10 106 = 2.3 107
and
0.56 104 = 5.6 101 104 = 5.6 105
Repeat with more examples as necessary.
Now reverse the process and ask the class to convert numbers in standard form
to ordinary numbers. For example:
3.4 105 = 340 000 3.6 107 = 0.000 000 36
They should be able to relate this to work done previously with powers of 10.
Repeat with more examples as necessary.
The class can now do Exercise 7A from Pupil Book 3.

80 HarperCollinsPublishers Ltd 2003


9B3LP_07.qxd 18/09/2003 15:20 Page 81

Exercise 7A Answers

1 a 5.69 103 b 1.2 106 c 9.38 105 d 7.78 104 e 3.965 108
f 5.61 102 g 7.3 101 h 4.3 109
2 a 3.4 103 b 5.6 102 c 3.71 105 d 9.2 106 e 7.6 101
f 5 104 g 7.2 106 h 4 104
3 a 8.9 106 b 5.3 103 c 1.8 104 d 3.33 107 e 6.7 106
f 8.923 103 g 7.35 101 h 9 105
4 a 2 300 000 b 456 c 670 000 d 3590 e 9 000 000 f 2 010 000
g 34 780 h 87 300 000
5 a 0.000 067 b 0.0385 c 0.000 78 d 0.005 39 e 0.000 008 f 0.167
g 0.003 21 h 0.000 000 66
6 a 4 600 b 0.057 66 c 930 d 0.001 22 e 50 000 f 0.305 g 4 820 000
h 0.0543
7 a 4.3 106 b 5.68 103 c 7.8 103 d 5.8 102 e 9.4 104
f 2.01 104 g 8 104 h 8 102 I 2.5 103 j 5.6 103
k 6.7 104 l 3.59 104
8 a 880 b 53 200 c 0.003 14 d 0.903 e 0.001 82 f 7 950 000
g 504 000 000 h 0.000 684 2

Extension Answers

a 1.5 108 km b 2.5 103 cm c 2.21 109 s d 9 1028 gm


e 1.868 107 g

Plenary Key Words


Make sure that the class is aware of the difference between a calculator standard form
display showing standard form as 4.39 and the actual standard form number 4.3
109. This is a common error in SATs and GCSE exams.
integer
Put some calculator displays on the board. Then ask individual students to less than
convert them: first, to a standard form number and next to an ordinary number. less than or
equal to
Homework

1 Write each of the following numbers in standard form.


a 63 000 000 b 0.000 74 c 322 000 d 83 300
e 0.000 000 71 f 92 321 g 0.009 35 h 0.000 0005
2 Write each of the following standard form numbers as an ordinary number.
a 4.9 104 b 4.36 103 c 8.4 103 d 5.68 102
e 8 109 f 4.82 104 g 9.2 106 h 6.03 101
3 Write each of the following numbers in standard form
a 68 103 b 37.8 105 c 0.87 103 d 58 104

Answers
1 a 6.3 107 b 7.4 104 c 3.22 105 d 8.33 104 e 7.1 107 f 9.2321 104 g 9.35 103
h 5 107
2 a 49 000 b 0.004 36 c 8400 d 0.0568 e 8 000 000 000 f 0.000 482 g 9 200 000
h 0.603
3 a 6.8 104 b 3.78 104 c 8.7 104 d 5.8 103

HarperCollinsPublishers Ltd 2003 81


9B3LP_07.qxd 18/09/2003 15:20 Page 82

LESSON Framework objectives Multiplying with numbers in standard


form
7.2 Enter numbers in standard form into a calculator and interpret the display.

Oral and mental starter


Use a target board like the one shown on the right.
Ask individual students to give each number in standard form. 134 5477 0.023 0.25 2000
(The numbers are straightforward enough to allow this to be done 35 0.067 0.007 0.039 270
mentally.)
50 6000 0.134 0.03 0.001
0.256 256 0.002 1300 100

Main lesson activity


Tell the class that they are going to learn how to multiply together two numbers
in standard form.
Give them: (3 106) (3 102) and ask them what they think the answer is.
Some students may respond intuitively that it is 9 108.
Explain the process of separating the numbers and the powers:
3 3 106 102.
Now give the class: (6 106) (3 102). Ask them for the answer. Some will
probably respond 18 108.
Ask them what is wrong with this. Explain that it is not in standard form.
Demonstrate how to get it in standard form. That is, 1.8 10 108 = 1.8 109.
Repeat with: (2.5 102) (8 105). This gives 20 107 = 2 10 107 =
2 106.
Do other examples as necessary.
Next give the class: (4.56 103) (2.13 107). They will realise that this is
difficult without the use of a calculator.
Explain how to use a calculator to enter numbers in standard form.
4.56 103 is entered as:
4 . 5 6 EXP 3
2.13 107 is entered as:
2 . 1 3 EXP 7 +/
Note that on some makes of calculator, the appropriate key may be marked EE
or with some other notation, and that the sign change key may operate
differently.
Explain the calculator notation which relates to the previous plenary.
Do the above calculation. The display should say 0.00 097 128 or 9.712804.
Convert this to the standard form number 9.7128 104 or round to 9.71 104
(3 sf).
Make sure that no students enter 4.56 103 as:
4 . 5 6 1 0 EXP 3
This will give a value of 4.56 104.

The class can now do Exercise 7B from Pupil Book 3.

82 HarperCollinsPublishers Ltd 2003


9B3LP_07.qxd 18/09/2003 15:20 Page 83

Exercise 7B Answers

1 a 8 105 b 1.2 108 c 8 107 d 9 105 e 3.2 107 f 4.2 102


g 2.1 101 h 2.6 103 i 5.6 103 j 2.25 105 k 1.12 103
l 3.6 107
2 a 9.46 109 b 1.152 108 c 1.288 1010 d 5.51 105 e 4.672 105
f 1.674 101 g 2.99 104 h 1.311 103 i 4.6656 107
3 a 9.82 1010 b 7.28 107 c 7.27 109 d 2.35 106 e 4.05 103
f 5.84 102 g 2.95 106 h 1.56 106 i 6.13 1013 j 1.63 105
4 1.25 109 bytes
5 1.2 m

Extension Answers

The American mathematician Edward Kasner asked his nine-year-old nephew to


invent a name for the number, ten to the power of one hundred (10100). His nephew
called it a googol. Another mathematician came up with googolplex, and defined it
to be ten to the power of googol.
There is no limit to the largest number in practical use, but remember that the total
number of elementary particles in the known universe is only about 1080.

Plenary Key Words


Give the class the following problem: 0.012 0.006.
standard form
They should be able to do this as 12 6 = 72, with six decimal places in the
answer: 0.000 072. addition of
Ask them to translate the calculation to a standard form problem which would powers
be: (1.2 102) (6 103) = 7.2 105.
Repeat with:
0.085 0.04 = 0.0034
(8.5 102) (4 102) = 34 104 = 3.4 103
Discuss the similarities and the advantages of each method.
Homework

1 Do not use a calculator for this question. Work out each of the following and give your answer in
standard form.
a (4 102) (2 106) b (5 103) (4 102) c (6 103) (2 104)
d (9 102) (3 108) e (5 105) (8 103) f (7 103) (7 103)
2 You may use a calculator for this question. Work out each of the following and give your answer in
standard form. Do not round off your answers.
a (2.1 105) (3.4 103) b (3.2 103) (1.5 104) c (3.6 103) (2.8 108)
d (1.5 102) (2.5 104) e (3.8 104) (2.8 104) f (8.6 104) (1.5 107)

Answers
1 a 8 108 b 2 106 c 1.2 106 d 2.7 107 e 4 107 f 4.9 107
2 a 7.14 108 b 4.8 107 c 1.008 104 d 3.75 106 e 1.064 101 f 1.29 102

HarperCollinsPublishers Ltd 2003 83


9B3LP_07.qxd 18/09/2003 15:20 Page 84

LESSON Framework objectives Dividing with numbers in standard form


Divide numbers in standard form.
7.3 Enter numbers in standard form into a calculator and interpret the display.

Oral and mental starter


Use a target board such as the one shown on the right.
2 103 3.2 102 5.4 102 4.3 101
Ask individual students to give the ordinary numbers
shown by the numbers in standard form. (The numbers are 7 104 6.3 103 6.3 103 5.1 102
simple enough for the students to do this mentally.)
3.6 103 8.1 102 2.5 103 3 103
4.1 101 6.8 101 4 105 8 102
6 105 5.2 103 1.7 102 5.6 101

Main lesson activity


This lesson focuses on dividing one number in standard form by another number
in standard form.
Give the class: (9 106) (3 102) and ask them what they think the answer is.
Some students may intuitively respond that it is 3 104.
Explain the process of separating the numbers and the powers:
(9 3) (106 102). Make sure they are happy with the position of the
multiplication sign.
Now give the class: (4 106) (8 102). Ask them for the answer. Some will
probably respond 0.5 104.
Ask them what is wrong with this. Explain that it is not in standard form.
Demonstrate how to get it in standard form. That is, 5 101 104 = 5 103.
Repeat with: (2 102) (8 105). This gives 0.25 103 = 2.5 101 103 =
2.5 102.
Do with other examples as necessary.
Next give the class: (4.56 103) (2.13 107). They will realise that this is
extremely difficult without the use of a calculator.
Make sure that they can enter this into their calculators using the EXP (or
equivalent) key and the sign change key.
The display should say 2.140 845 0710
Round off and convert this to the standard form number 2.14 1010 (3 sf).
The class can now do Exercise 7C from Pupil Book 3.

84 HarperCollinsPublishers Ltd 2003


9B3LP_07.qxd 18/09/2003 15:20 Page 85

Exercise 7C Answers

1 a 3 104 b 7.5 102 c 2 101 d 3 105 e 5 105 f 1.2 109


g 9 107 h 4 108 i 1.6 105 j 5 102 k 7 107 l 3 103
2 a 4.5 104 b 4.05 102 c 2.8 105 d 3.4 102 e 8 109 f 6 109
g 7 104 h 8 105 i 2 102
3 a 5.61 104 b 1.37 103 c 1.32 104 d 2.92 102 e 5.98 1011
f 2.88 109 g 6.23 1012 h 4.48 104 i 2.5 103 j 1.2 103

4 6.366 103 km 5 33.3

Extension Answers

Milli = one thousandth (103); micro = one millionth (106); nano = one billionth
(109); pico = one trillionth (1012); femto = one quadrillionth (1015); atto = one
quintillionth (1018); zepto = one sextillionth (1021); yocto = one septiillionth (1024)

Plenary Key Words


Give the class this problem: 0.012 0.006.
They should be able to do this as:
standard form
0.012 0.006 = 0.12 0.06 division
= 1.2 0.6 = 12 6 = 2 subtraction of
Ask them to translate the calculation to a standard form problem, which would powers
be:
(1.2 102) (6 103)= 0.2 101 = 2
Repeat with:
0.055 0.05 = 1.1
(5.5 102) (5 102) = 1.1 100 = 1.1
Discuss the similarities and the advantages of each method.
Homework

1 Do not use a calculator for this question. Work out each of the following and give your answer in
standard form.
a (8 105) (2 103) b (4 105) (5 107) c (6 103) (2 104)
d (1.2 103) (3 102) e (6 106) (8 101) f (5 102) (8 103)
2 You may use a calculator for this question. Work out each of the following and give your answer in
standard form. Do not round off your answers.
a (6.15 105) (1.5 102) b (3.15 106) (1.4 101)
c (3.19 103) (1.45 102) d (2.32 103) (2.9 105)
e (5.85 103) (6.5 103) f (1.495 106) (4.6 102)

Answers
1 a 4 102 b 8 103 c 3 107 d 4 102 e 7.5 106 f 6.25 104
2 a 4.1 103 b 2.25 107 c 2.2 105 d 8 101 e 9 107 f 3.25 107

HarperCollinsPublishers Ltd 2003 85


9B3LP_07.qxd 18/09/2003 15:20 Page 86

LESSON Framework objectives Upper and lower bounds 1


Understand upper and lower bounds for discrete data and continuous data.
7.4

Oral and mental starter


Use a target board such as the one shown on the right.
100 marbles 25 cm 500 eggs 300 g
Ask individual students to give the limits of each
number chosen by you to a given accuracy. For 5 kg 25 m 400 people 200 ml
example, 100 marbles (to nearest 10), 95 to 104;
5 litres 80 cm 2000 400 sweets
80 cm (to nearest cm), 79.5 to 80.5.
The students may need to have some of the rules of 50 g 100 cars 150 mm 220 ducks
upper and lower bounds explained to them,
600 mph 600 people 40 km 120 bees
particularly the difference between discrete and
continuous data.

Main lesson activity


Ensure that the class can work out the limits, called the upper and lower
bounds, of numbers given to degrees of accuracy.
Make sure they are happy with the difference between limits of accuracy for
discrete quantities, such as a number of people, and continuous quantities, such
as the weight of people.
Some students may be happier to have a rule for continuous data:
Lower bound = Value Half the degree of accuracy
Upper bound = Value + Half the degree of accuracy
For example, 500 given to the nearest 10 has a lower bound of 500 half of
10 = 495 and an upper bound of 500 + half of 10 = 505
23 cm to the nearest centimetre has upper and lower bounds of 23 half a
centimetre: 22.5 23 < 23.5
Emphasise that the upper value is a limit and can be given as a value and need
not be truncated to 23.4 or 23.4999.
This is a concept students find hard to grasp. Justify it by using the notation:
22.5 Value < 23.5.
In other words, the value can equal the lower bound but cannot equal the upper
bound.
Give students the following example:
One piece of wood is 25 cm long and another is 30 cm long, both
measurements to the nearest centimetre. If the pieces are joined end to end,
what are the upper and lower bounds of the combined length?
Upper and lower bounds of first piece: 24.5 Length < 25.5
Upper and lower bounds of second piece: 29.5 Length < 30.5
Lowest possible combined length: 24.5 + 29.5 = 54 cm
Largest possible combined length: 25.5 + 30.5 = 56 cm
Upper and lower bounds of combined length: 54 Combined length < 56 cm
Next, do the following example:
One bag of marbles contains 200 marbles, measured to the nearest 10. Another
bag of marbles contains 120 marbles, measured to the nearest 10. If both bags
are combined, what are the upper and lower bounds for the number of marbles
in the bag.
Upper and lower bounds for first bag: 195 Marbles 204
Upper and lower bounds for second bag: 115 Marbles 124
Emphasise why these limits are precise.
Upper and lower bounds for combined bags: 310 Marbles 328
The class can now do Exercise 7D from Pupil Book 3.

86 HarperCollinsPublishers Ltd 2003


9B3LP_07.qxd 18/09/2003 15:20 Page 87

Exercise 7D Answers

1 a 25 Toffees 34 b 15 Rice < 25 g c 65 Speed < 75 mph


d 69.5 Speed < 70.5 mph e 15 String < 25 cm f 19.5 String < 20.5 cm
g 495 Cake < 505 g h 49.5 Donut < 50.5 g i 999.5 Jug < 1000.5 cm3
j 39.5 Hard drive < 40.5 Gb
2 a 395 Length < 405 cm b 295 Width < 305 cm
c 13.8 Perimeter < 14.2 m
3 a 9.5 9.5 cm b 10.5 10.5 cm c No, could be guaranteed to cover 95 cm
d No, if all tiles are 9.5 cm, he will need 32 22 = 704
4 a 6250 b 6745
5 a 195 Number 204 b 195 Mass < 205 g
c Number is discrete data, mass is continuous
6 a 23 500 b 244 990 245 000
7 a 225 b 234
8 No, if each slab is only 39.5 cm, he will only have enough for 19.75 m
9 Yes, 9 mugs would be 205 9 = 1845. Smallest jug is 1850 ml
10 No, Melanies lowest possible total is 43 + 48 + 62 + 38 + 52 = 243, which is not
enough

Extension Answers

a For largest length left, take the shortest length of cut-off piece from the longest length
of original: 120.5 84.5 = 36 cm. For shortest length left, take the longest length of
cut-off piece from the shortest length of original: 119.5 85.5 = 34 cm.
b For largest possible length, divide the smallest possible area of the rectangle by the
smallest possible width: 42.5 1.5 = 28.33 cm. For smallest possible length, divide
the largest possible area by the largest possible width: 47.5 2.5 = 19 cm.
Key Words
Plenary upper bound
Give the class the following problem.
lower bound
a = 5 to nearest unit, b = 16 to nearest unit. strict inequality
What are the upper and lower bounds of a + b (20, 22), a b (69.75, greater than/less
90.75), a b (10, 12), a b (0.2727 , 0.3548 )? than
Discuss the required combinations, particularly the subtraction and division.
Homework

Do not use a calculator for Questions 1 and 2.


1 Find the upper and lower bounds between which the following quantities lie.
a In a hive there are 2000 bees to the nearest 100.
b The amount of honey in a jar is 200 ml to the nearest 10 ml.
c The width of a field is 70 m to the nearest metre.
d The mass of a loaf is 0.6 kg to the nearest 100 grams.
2 A poster is 2.5 metres by 1.5 metres, each measurement accurate to the nearest 10 cm.
a What are the upper and lower bounds for the length of the poster?
b What are the upper and lower bounds for the width of the poster?
c What are the upper and lower bounds for the perimeter of the poster?
3 A bottle of water holds 1 litre to the nearest centilitre.
a What is the smallest possible amount in the bottle?
b What is the greatest possible amount that 10 bottles could hold?

Answers
1 a 1950 Bees < 2049 b 195 Honey < 205 ml c 69.5 Width < 70.5 m d 0.55 Mass < 0.65 kg
2 a 2.45 Length < 2.55 m b 1.45 Width < 1.55 m c 7.8 Perimeter < 8.2 m
3 a 99.5 cl b 10.05 litres

HarperCollinsPublishers Ltd 2003 87


9B3LP_07.qxd 18/09/2003 15:20 Page 88

LESSON Framework objectives Upper and lower bounds 2


Understand upper and lower bounds for discrete data and continuous data.
7.5

Oral and mental starter


Recall the methods of multiplying integers and decimals
0.8 0.3 9 0.6
expressed to one significant figure. For example,
0.7 6 = 4.2, 0.03 8 = 0.24. 0.7
Use this to ask for answers to a variety of similar questions. As
5
these can be hard to verbalise and for students to conceptualise, a
grid, as shown on the right, could be used. 0.02
Particular squares could be pointed at and individual students
0.01
asked, for the product.
Reverse the process by asking for the missing number in,
for example, 0.4 ? = 0.32 (0.8), or the answer to 0.048 0.6 = ? (0.08).
Repeat with examples such as 0.04 ? = 0.08 (12), 0.5 ? = 0.035 (0.07).

Main lesson activity


This lesson is concerned with making calculations with limits.
Following the previous plenary, the students will have some idea of how the
maximum and minimum values are combined. To help them the following table
could be copied into their books.
For two numbers a and b with upper and lower bounds amax a < amin and
bmax b < bmin
Operation Maximum Minimum
a+b amax + bmax amin + bmin
ab amax bmin amin bmax
ab amax bmax amin bmin
ab amax bmin amin bmax
Work through the following examples.
A jar contains 850 ml of oil (accurate to the nearest 10 ml). A cup containing
250 ml (to the nearest 10 ml) is poured out. What are the upper and lower
bounds of the amount of oil remaining in the jar?
Least amount left: 845 255 = 590.
Greatest amount left: 855 245 = 610
Upper and lower bounds of amount left: 590 Amount left < 610 ml
A runner sprints 400 metres in 55 seconds, each measurement to the nearest
unit. What are the upper and lower bounds of the runners speed?
Lowest speed: 399.5 55.5 = 7.20 m/s
Greatest speed: 400.5 54.5 = 7.35 m/s (3 sf)
The same runner runs for 40 minutes at a speed of 8 mph, both values to two
significant figures. What are the upper and lower bounds of the distance run?
Least distance: 39.5 7.5 = 296.25 60 = 4.94 miles
Greatest distance: 40.5 8.5 = 344.25 60 = 5.74 miles (3 sf)
The class can now do Exercise 7E from Pupil Book 3.

88 HarperCollinsPublishers Ltd 2003


9B3LP_07.qxd 18/09/2003 15:20 Page 89

Exercise 7E Answers

1 a 13.75 Area < 22.75 cm2 b 11.25 Area < 21.25 cm2
c 97.75 Area < 118.75 cm2
2 a 11.5 a < 12.5, 17.5 b < 18.5, 23.5 c < 24.5 b i 201.25 a b < 231.25
ii 1.4 a a < 1.61 iii 681.5 (a + b)c < 759.5 iv 552.25 c2 < 600.25
3 a 82.4 mph (1.37 miles per min) b 77.7 mph (1.29 miles per min)
4 a 1137.75 miles b 1062.75 miles
5 a 2.903 b 3.103 g
6 9.75 Side < 10.25 cm (3 sf)
a+b
7 a 94.62 a b c < 101.3 (4sf) b 1.109 < 1.156 (4sf)
c
c 0.75 a + b + c < 1.05 d 1.44 (b a)2 < 1.96
8 9.61 Width < 10.4 cm (3 sf)
9 1.65 Mass coat < 2.75 kg
10 No, rod of 4.05 cm would be required to fit into hole which is 3.5 cm

Extension Answers

1 352.3 Surface area < 402.8 cm2 (4 sf)


2 214.3 Surface area < 219.1 cm2 (4 sf)

Plenary Key Words


Ask the class to consider which combination of maximum and minimum value
upper bound
of a, b, c and d would give the maximum and minimum values of an expression
such as: lower bound
ab a2 strict inequality
or d
cd (b c) greater than/
Ensure that the students understand the need to combine upper and lower less than
bounds in such a way that they maximise and minimise the value of such
combined expressions.
Homework

1 a = 10, b = 20 and c = 30. All values to the nearest whole number.


a Write down the upper and lower bounds of a, b and c.
b Work out the upper and lower bounds of each of the following.
i ab ii c a iii (a b) + c iv c2
2 A rectangle has an area of 120 cm2, measured to the nearest 10 cm2. The length is 15 cm, measured
to the nearest cm.
a What is the greatest possible width of the rectangle?
b What is the least possible width of the rectangle?

Answers
1 a 9.5 a < 10.5, 19.5 b < 20.5, 29.5 c < 30.5 b i 185.25 a b < 215.25
ii 2.81 c a < 3.21 (3 sf) iii 214.75 (a b) + c < 245. 75 iv 870.25 c2 < 930.25
2 a 8.62 cm (3 sf) b 7.42 cm (3 sf)

HarperCollinsPublishers Ltd 2003 89


9B3LP_07.qxd 18/09/2003 15:20 Page 90

LESSON Framework objectives Recurring decimals


Know that a recurring decimal is an exact fraction. Use algebraic methods to
7.6 convert a recurring decimal to a fraction in simple cases.

Oral and mental starter


The class will need calculators. 12 37 7
20 32 13

18
Using a target board, like that shown on the right, ask the class whether they can
11
7 43
38 97
predict which of the fractions, when converted to a decimal, will terminate and 30 12 50

which will recur. 3 34 1 12


56
11 28 25
They have met these ideas before but may need to be reminded of them.
After they have made their predictions, ask them to work out the decimals on 9
25 7
60
31

40 45 3
13

their calculators.
These can then be written on the board and the recurring notation explained to
32
the class. For example: 154 = 0.26, 33
= 0.96.
Ask if there is any relationship between the denominators which give
terminating decimals? They are all multiples of powers of 2 (2, 4, 8, 16, ),
powers of 5 (5, 25, 125, ), powers of 10 (10, 100, 1000, ), or products of
these, such as 40 (4 10), 50 (5 10) .

Main lesson activity


This follows on from the Oral and mental starter.
Ask: How would you write a recurring decimal as a fraction: for example, 0.454

545 = 0.45?
Give them the chance to see whether they can find the answer by trial and
improvement. They may find the answer of 115.
Outline the following method.
Let F = 0.454 545 45 (1)
Then 100F = 45.454 545 (2)
Subtract equation (1) from (2): 99F = 45
45 5
Divide through by 99: F = = (Cancel by 9).
99 11
Ask: Why multiply by 100?
Establish that we have to multiply by the power of 10 equivalent to the number
of recurring digits.
26
Now repeat with 0.7 ( 97 ), 0.234 ( 111
).
Ask: Can anyone see a connection or a short cut?
Establish that when there is just one recurring digit, the denominator will be 9;
when there are two recurring digits, the denominator will be 99; and when there
are three recurring digits, the denominator will be 999.
Be careful! This rule works only when the recurring digits are the only digits after

the decimal point. It will not work with a number such as 0.377 777 7 = 0.37.
Give some examples:
6 2 39 13 675 25
0.6 = = 0.39 = = 0.675 = =
9 3 99 33 999 37

Put this recurring decimal on the board: 0.277 777 77 = 0.27.
Give the students a few moments to see whether they can find its fraction by trial
and improvement.
Now work through the procedure.
Let F = 0.277 777 77 (1)
Multiply by through10: 10F = 2.777 777 77 (2)
Subtract equation (1) from (2): 9F = 2.5
2.5 5
Divide through by 9: F = =
9 18
Repeat with 0.166 66 (16 ) and 0.416 666 (12 5 )

The class can now do Exercise 7F from Pupil Book 3.

90 HarperCollinsPublishers Ltd 2003


9B3LP_07.qxd 18/09/2003 15:20 Page 91

Exercise 7F Answers


1 a 0.571 428 b 0.7524 c 0.69 d 0.3 e 0.2

2 9 = 0.1, 9 = 0.2, 9 = 0.3, 9 = 0.4, 9 = 0.5, 9 = 0.6, 9 = 0.7, 89 = 0.8, 99 = 0.9
1 2 3 4 5 6 7

Recurring digits are the same as original numerators


2
1 = 0.09, 11
4 6
3 11 = 0.18, 113 = 0.27, 11 = 0.36, 115 = 0.45, 11 = 0.54, 117 = 0.63
8
= 0.72, 11
9
= 0.81, 11
10
= 0.90

11
Recurring digits form the nine times table up to 90
4
4 17 = 0.142 857, 27 = 0.285 714, 37 = 0.428 571, 11 = 0.571 428, 75 = 0.714 285,
6
7 = 0.857 142

First recurring digits are in numerical order
5 a 115 b 107
333 c 89 d 37 27
e 33 4 f 11 9 g 1111
123
h 26
33
i 89
111 j 99 = 1
4
28 578
352
23
6 a 90 b 495 c 999 d 4995 e 666
389
b 118 1 c 5185 39
2021

7 a 2 990 d 2 4950

Extension Answers

1
2 3 4 5
13 = 0.076 923, 13 = 0.153 846, 13 = 0.230 769, 13 = 0.307 692, 13 = 0.384 615,
6 7 8 9 10
= 0.769 230 ,

13 = 0.461 538, 13 = 0.538 461, 13 = 0.615 384 , 13 = 0.692 307, 13
11
12
= 0.923 076, 13
= 1
13 = 0.846 153, 13 13

Plenary Key Words



Ask the class to write down from their calculators the values of , 2 and 5.
recurring
Ask them if they can spot the difference between these decimals and those with
which they have been working. decimal
terminating
The values of , 2 and 5 do not terminate nor do they recur.
Define these as irrational numbers. That is, decimals which do not terminate or decimal
recur.
The other numbers are known as rational numbers. That is, numbers which can
be expressed as a terminating decimal or a recurring decimal.
Homework

1 Write each of the following fractions as a recurring decimal.


a 47 b 85

101 c 17

33

2 Write each of the following recurring decimals as a fraction in its simplest form.

a 0.54 b 0.246 c 0.2 d 0.12 e 0.37

Answers

1 a 0.571 428 b 0.8415 c 0.51
6 82
b 333

2 4 17
2 a 11 c 9 d 33
c 45

HarperCollinsPublishers Ltd 2003 91


9B3LP_07.qxd 18/09/2003 15:20 Page 92

LESSON Framework objectives Efficient use of a calculator


Use a calculator efficiently and appropriately to perform complex calculations with
7.7 numbers of any size, knowing not to round during intermediate steps of a calculation.
Use the constant, and sign change keys, the function keys for powers, roots and
fractions, brackets and the memory.

Oral and mental starter


Due to the variety of calculators in use and the problems this may cause, there is
no Oral and mental starter. However, some of the main keys listed in the
Framework objectives could be discussed and found for the different makes of
calculator. Some students may need help to locate the inverse/shift keys, which
are needed on many calculators in order to do powers.

Main lesson activity


This is a review lesson on the effective use of calculators.
The class should have met most of the keys before.
Do a variety of problems using the appropriate keys. Make sure the students can
find these keys on their calculators.

Work out, for example, 5.32 . This requires the square root key, the key
and the square key. The answer is 0.334 425 254 9 0.33.
Discuss the need to round answers to a sensible degree of accuracy.
Now take 4.55. This requires the power key. The answer is 1845.281 25 1845.
As the next example, work out:
(78 12
5 ) ( 15
11 7 )
+ 12
55
This requires the fraction key and the brackets keys. The answer is
158.
Finally, work out:
3.56(43.2 17.48)

(4.53 1.2) 0.92
This requires the brackets keys. The answer is 29.887 452 67 29.89 30.
The class can now do Exercise 7G from Pupil Book 3.

92 HarperCollinsPublishers Ltd 2003


9B3LP_07.qxd 18/09/2003 15:20 Page 93

Exercise 7G Answers

1 a
153.9 b 0.627 c 820
2 a
13.69 b 32 768 c 1.95 d 0.000 002 2
3 a
3 b 8 c 11 d 1.5
4 2 b 4 c 5 d 0.6
a
5 a 15.8 b 70.4 c 512 d 104.22 e 7.87
6 a 138
99
1
b
210 c 413

20

Extension Answers

0.618

SATs Answers

1 Using film of 24 photos costs 56.10. Using film of 36 photos costs 61.40. For 360
photos, 24 photo films are 5.30 cheaper
2 a 0.636 62 b 0.528 68
3 a = 1500, b = 20
4 13 403.076 92 gallons, 13 000 gallons
5 a 1.01 105 N/m2 b 16.66:1 c 5.79 1010 km3
6 a 4 104 b 4 105 c 4.4 104
7 a ii Time, in hours, taken by spaceship to travel to Proxima Centauri
iii time in years, taken by spaceship to travel to Proxima Centauri
b 11 years

Plenary Key Words


This is the last time that calculators will be the focus of the lesson, so each
student needs to be sure that he/she can use his/her particular model. Ensure
fraction
that they can identify the following keys: brackets, memory (M in, M+, M), , brackets
square root, square, power, fraction. square root
This leaves only three principal keys that will be needed in future: sine, cosine square
and tangent. power
pi
memory keys
Homework

1 Use a calculator to evaluate each of these.


63.4 21.02 19 7
a [2.42 + (6.7 1.04)]2 b c
2.9(4.5 1.72) 21 18
2 Use the power key to evaluate each of these.
2

a 2.75 b 42.8753

Answers
65
1 a 130.4164 b 165.3 c
126
2 a 143.5 b 12.25

HarperCollinsPublishers Ltd 2003 93


9B3LP_08.qxd 18/09/2003 15:20 Page 94

CHAPTER
8 Algebra 4
LESSON Framework objectives Index notation with algebra:
negative powers
8.1 Know and use the index laws (including in generalised form) for multiplication and
division of positive integer powers; begin to extend understanding of index notation
to negative and fractional powers, recognising that the index laws can be applied
to these as well.

Oral and mental starter


Ask the class approximately how many litres there are to a gallon. The correct answer is about four and a half.
Then ask how many litres there are in 8 gallons. The answer is 36.
Ask them how they arrived at this answer and discuss the various methods used.
One way is to multiply the 4 by 8, getting 32, then to find the result of 8 halves, which is 4. Next, add the two
together, giving 32 + 4 = 36.
Now ask how many litres there are in: 12 gallons (54 litres); 22 gallons (99 litres); 17 gallons (7612 litres).
Now ask approximately how many gallons are in 60 litres.
Discuss the strategy. For an approximation, you would not try to divide by 4.5. Rather, you would divide by
4 and also by 5 and go for a midway answer. Here this gives 60 4 = 15 and 60 5 = 12. Approximately
halfway between 15 and 12 is 13.5, which is rounded up to 14.
Ask the class how many gallons there are in: 100 litres (22 gallons); 70 litres (16 gallons); 40 litres (9 gallons);
134 litres (30 gallons).

Main lesson activity


Write on the board x2 + x3. Ask the class if they can simplify this expression. The expression cannot be
simplified because it contains two unlike terms.
Now, write on the board x2 x3. Ask the class if they can simplify this expression. If someone comes up with
the correct answer of x5, ask him/her to explain how.
Have a discussion with the class about multiplying expressions containing powers and adding the powers. Ask
if anyone can generalise the rule. You are looking for something like:
xa xb = xa + b
Ask if anyone can prove this rule. You want something like:
k2 k3 = (k k) (k k k) = k k k k k = k 5 = k2 + 3
Now, write on the board 4m2 3m. Ask the class to simplify this expression. If someone comes up with the
correct answer of 12m3, ask him/her to explain how, i.e., that they multiplied the coefficients and added the
powers. If not, remind the class of this and that m is the same as m1 and work through some more examples.
Next, move on to dividing powers. Write on the board the expression 8m5 2m3 and ask if anyone can tell you
what the answer should be. Again, if someone comes up with the correct answer of 4m3, ask him/her to explain
how, i.e., that they divided the coefficients and subtracted the powers. If not, work through this with the class
and introduce some further examples.
You should explore the proof of this with an example such as:
kkkkkk
k 6 k2 = = k k k k = k4 = k 6 2
kk
Now write on the board the expression m3 m5 and ask the students to simplify this expression. You should get
the response m2. Make sure that the students understand that numbers and variables with negative indices
follow the same rules as numbers and variables with positive indices.
Ask the students what they think m2 represents. If the class covered standard form recently, they may bring this
knowledge to this discussion.
Introduce the investigation on page 139 of Pupil Book 9.3. Go through the first two parts with the class, then let
them complete the investigation individually or in pairs. Discuss the results with the whole class.
1
Finally, ensure that all of the students understand that nn = n .
x
The class can now do Exercise 8A from Pupil Book 3.

94 HarperCollinsPublishers Ltd 2003


9B3LP_08.qxd 18/09/2003 15:20 Page 95

Exercise 8A Answers

1 a 4x5 b 7m4 c n2 d x5 e 4m4 f 6x4 g 15t4 h 5m3 i g9 j m4


k 4t2 l 15m8 m 48q6 n abm5 o cdy4
1 1 1 1 5 4 8 a
2 a b 2 c d 4 e 2 f g 3 h
m k x3 n m y x b
3 a 51 b 41 c 3x1 d x2 e m6 f 91 g 5x4 h Am3
4 a 4x2 b 7m2 c n3 d 4x3 e 4m5 f 4x2 g 15t2 h 5m2 i g1
5 a x7 b m7 c n2 d 2x8 e 3m3 f 3x3 g 12t7 h 6m5 i 6g5
6 a 3x2 b 30m c 6n d 12x10 e 6m2 f 24x2 g 60t3 h 8m1 i 9g
j 4m1 k 30t7 l 24m7 m 6q12 n kpm6 p dey8

Extension Answers

1 a Any value larger than 1 or smaller than 0 b Values between 0 and 1


c x = 0 and x = 1
2 a Any value larger than 5 or less than 0 b Values between 0 and 5
c x = 0 and x = 5

Plenary Key Words


Discuss with the class why powers are used they shorten very large numbers
power
and also make very small numbers accessible.
Remind the class of the large number 10100, called the googol. Tell them that this index
was the answer to the million pound question on the television programme Who
Wants to be a Millionaire, when Major Charles Ingram was suspected of
cheating.
Homework

1 Expand the following, and find their value (use a calculator if necessary).
a 26 b 35 c 64 d 45 e 172 f 143 g 272 h 114
2 Write down the following in index form:
a tttt b ttttt c mm d qqq
3 a Write m + m + m + m + m + m as briefly as possible.
b Write t t t t t t as briefly as possible.
c Show the difference between 6m and m6.
d Show the difference between t4 and 4t.
4 Simplify each of the following:
a 2x3 4x7 b 12t6 3t c 20m5 5m3 d 3y 2y5 e x2 x3
5 Simplify each of the following, leaving your answer in fraction form:
a x3 x5 b 4m2 m5 c 8x4 2x d 2x5 3x8 e Ax Bx5 f Ax Bx5

Answers
1 a 64 b 243 c 1296 d 1024 e 289 f 2744 g 729 h 14641
2 a t4 b t5 c m2 d q3
3 a 6m b t6 c 6m = m + m + m + m + m + m, m6 = m m m m m m
d t4 = t t t t, 4t = t + t + t + t
4 a 8x10 b 4t5 c 4m2 d 6y6 e x
1 4 4 2 AB Ax6
5 a b c d e f
x2 m3 x5 3x3 x4 B

HarperCollinsPublishers Ltd 2003 95


9B3LP_08.qxd 18/09/2003 15:20 Page 96

LESSON Framework objectives Square roots, cube roots and other


fractional powers
8.2 Estimate square roots and cube roots.
Begin to extend understanding of index notation to negative and fractional powers,
recognising that the index laws can be applied to these as well.

Oral and mental starter


Tell the class that approximation is a valuable skill. It enables a quick check to
be made on calculations maybe even to ensure that they have used their
calculators correctly.
For example, ask the class if they can estimate the answer to 41 672 divided
by 8. Ask for quick approximations and write them up on the board. After a few
have been given, discuss with the class any strategy which they used to estimate
division by 8.
One way is to halve the dividend, then to halve the result, and finally to halve
the second result to complete the division. Doing this with 41 672 gives the
approximate sequence:
41 672 20 000 10 000 5000
Finally, ask the class to estimate the following (all divisions by 8):
629 537 300 000 150 000 80 000
173 492 90 000 45 000 22 000
23 774 12 000 6000 3000
117 008 60 000 30 000 15 000

Main lesson activity


Ask the class for the square root of 25.
If only 5 is given, remind the class that 5 is also a square root of 25.
Now ask the class to estimate the square root of 20. Put their suggestions on the
board.
Discuss with them any answers that are obviously wrong, and how they know
they are wrong. For example, the square root of 20 must be larger than 4 as
4 4 = 16, which is less than 20. Similarly, the square root of 20 must be less
than 5.
Hence, the square root of 20 must lie between 4 and 5. It looks halfway, so a
good estimate would be 4.5.
Ask the class to work out the correct answer using their calculators. You can use
this as an opportunity to show or remind them how to find square roots on their
calculators.
The calculator will show the square root of 20 to be 4.472 135 955. This rounds
to 4.5 so, the approximation was correct. Ensure that the class know the

correct symbol, , for square root.
Ask the class for the cube root of 8. You will need to explain that the cube root is
the number that gives 8 when multiplied by itself three times. It is written as
3
8 . In this example, the answer is 2.
Ask the class why they think that there are no negative answers to cube roots.
Write on the board a few of the better-known cube roots: the cube root of 27 is
3; the cube root of 125 is 5, and the cube root of 1000 is 10. Discuss why the
numbers for which the roots are given have grown so quickly.
Introduce the investigation on page 141 of Pupil Book 3. Go through the first
example with the class, then let them complete the investigation.

Finally, discuss with the class the generation x n = nx.
The class can now do Exercise 8B from Pupil Book 3.

96 HarperCollinsPublishers Ltd 2003


9B3LP_08.qxd 18/09/2003 15:20 Page 97

Exercise 8B Answers

1 a x = 3, x = 3 b x = 6, x = 6 c x = 7, x = 7 d x = 11, x = 11
2 a x2 = 50 2 = 25, hence x = 5 and x = 5 b x2 = 36 4 = 9, hence x = 3 and x = 3
3 a 4 b 5 c 9 d 10 e 2 f 10 g 4 h 5 i 5 j 3 k 6 l 2 m 3 n 5 p 2 q 9
4 a 5.1 b 7.4 c 9.7 d 10.4 e 16.6
5 a 2 b 1 c 5 d 3 e 10 f 4 g 1 h 10 i 0.1 j 0.2
6 b i 4.4 ii 4.6 iii 3.6 iv 6.4 v 12.6

3 50

7 a b 30 c 20 d 35 e 15 f 40
8 b i 3.6 ii 1.7 iii 7.9 iv 6.3 v 9.1 c i 4.0 ii 1.9 iii 7.4 iv 6.8 v 9.5
9 a 33 b 19 c 42 d 34 e 44

Extension Answers

a Only for A = B = 0
b Always true
c Never true
d Always true

Plenary Key Words


Ask the class which is bigger, the square root of ten thousand or the cube root of
a million. The answer is they are both the same, 100.
square root
Go through the better-known square roots and cube roots as a quick cube root
competition, seeing which set of students (row, column, table) remembers the
most. Use the list of roots in Pupil Book 3, page 141, but ensure that all students
have their textbooks shut.
Homework

1 Estimate the square root of each of the following. Then use a calculator find the result to one
decimal place and see how close you were.

a 46 b 31 c 74 d 129 e 215
2 Without a calculator, state the cube roots of each of the following numbers.
a 64 b 343 c 216 d 729 e 512
3 a Estimate the integer closest to the cube root of each of the following.
i 96 ii 110 iii 55 iv 297 v 3000
b Use a calculator to find the accurate value of the above. Give your answers to one decimal
place.
4 State which, in each pair of numbers, is the larger.

a 20 , 3 55 b 28 , 3 149 c 18 , 3 79
5 Estimate the cube root of each of these numbers without a calculator.
a 15 b 61 c 400 d 150 e 850
6 Try to estimate the cube root of each of these numbers without using a calculator.
a 25 000 b 8000 c 57 000 d 41 000 e 83 000
7 Write down the value of each of the following without using an index.
1
1
1
1
1

a 492 b 5123 c 164 d 10245 e (343)3

Answers
1 a 6.8 b 5.6 c 8.6 d 11.4 e 14.7 2 a 4b 7 c 6 d 9
e 8
3 i 4.6 ii 4.8 iii 3.8 iv 6.7 v 14.4 4 a 20 b 3 149 c 3 79
5 a 2.5 b 3.9 c 7.4 d 5.3 e 9.5 6 a 29 b 20 c 38 d 34 e 44
7 a 7 b 8 c 2 d 4 e 7

HarperCollinsPublishers Ltd 2003 97


9B3LP_08.qxd 18/09/2003 15:20 Page 98

LESSON Framework objectives Quadratic graphs


Plot graphs of simple quadratic functions.
8.3
Oral and mental starter
Ask if anyone has been to France or Germany recently. If someone has, ask how
many euros he/she got for a pound.
There are approximately 1.50 euros to the pound. Hence, an easy way to make
a comparison while shopping in Europe is to divide the price in euros by 3, then
double the result to get the approximate price in pounds.
Go through a few examples, such as:
25 euros: Dividing by 3 gives about 8, doubling this gives 16. So 25 euros
is about 16.
37 euros: Dividing by 3 gives about 12, doubling this gives 24. So 37 euros
is about 24.
Now ask the students to try this themselves, to convert the following euro prices
to the approximate equivalent in pounds: 9 euros (6), 15 euros (10), 22 euros
(14), 29 euros (20), 49 euros (32).

Main lesson activity


Put on the board y = mx + c, and ask the class what type of equation this is.
You want the answer linear. Then ask them what is special about the graph
drawn from an equation like this.
You want the answer: Its a straight line. If they do not also mention that m is
the gradient and c is the y-axis intercept, try to tease this from them.
Then put on the board the equation y = ax2 + bx + c, and ask if anyone knows
what type of equation this is.
You want the answer quadratic. This may not be known, so explain the term,
and that it is used because the highest power is a square.
Tell the class that, today, they are going to look at the graphs of equations like
this. Put the equation y = x2 4x + 3 on the board. Then tell them that you are
going to go through the drawing of this graph with them.
Start by putting a table on the board, as below.
x 1 0 1 2 3 4
x2 1 0 1 4 9 16
4x 4 0 4 8 12 16
3 3 3 3 3 3 3
y= x2 + 2x + 3 8 3 0 1 0 3

Complete this table with help from the class. Plot the coordinates given by the
table on a pair of axes drawn on the board. Plot the coordinates to show a
smooth curve.
Explain that the U-shaped curve is typical for a graph from a quadratic equation,
but do tell them that when the x2 part is negative, the graph will be an inverted
U-shape.
Ensure the class realise that quadratic graphs will always be curved, with no
straight lines and no kinks.

The class can now do Exercise 8C from Pupil Book 3.

98 HarperCollinsPublishers Ltd 2003


9B3LP_08.qxd 18/09/2003 15:20 Page 99

Exercise 8C Answers

1 a 9, 4, 1, 0, 1, 4, 9 b 13, 8, 5, 4, 5, 8, 13 c 12, 6, 2, 0, 0, 2, 6
d 3, 1, 3, 3, 1, 3, 9
2 a x 3 2 1 0 1 2 3
y 18 8 2 0 2 8 18
b x 4 3 2 1 0 1 2
y 8 3 0 1 0 3 8
c x 4 3 2 1 0 1 2
y 5 0 3 4 3 0 5
d x 3 2 1 0 1 2 3
y 4 0 2 2 0 4 10
3 b All the curves go through the origin, but the larger the value of a, (coefficient of
x2), the wider the graph becomes
4 b They are all parallel curves, with c being the y-axis intercept

Extension Answers

1 b The graph is an inverted U shape


2 b y = x2

Plenary Key Words


Talk about the characteristic U-shape of a quadratic graph, and inverted U-shape
quadratic
when the x2 coefficient is negative.
Compare the difference between a quadratic graph and a linear graph. In intercept
particular, the fact that in both cases, for each value of x, there is only one value gradient
of y. However, when given a value of y, there is only one possible value for x in
a linear graph, but in a quadratic graph there could be no value for x, one value
for x or two values for x. You will need to point out where this happens on a
graph.
Point out to the class that this will be useful to them when they come to looking
at solutions of quadratic equations.
Homework

1 A sledge sliding down a slope has travelled a distance, d metres, in time, t seconds, where
d = 5t + t2.
a Draw a graph to show the distances covered up to 6 seconds.
b Find the distance travelled after 3.8 seconds.
c Find the time taken to travel 50 metres.
2 The cost, C pence, for plating knives of length L cm is given by the formula C = 50L + 7L2.
a Draw a graph to show the cost of plating knives up to 10 cm long.
b What would be the cost of plating a knife 8.7 cm long?
c What would be the length of a knife costing 4 to plate?

Answers
1 b 33.4 m c 5 s
2 b 9.65 c 4.8 cm

HarperCollinsPublishers Ltd 2003 99


9B3LP_08.qxd 18/09/2003 15:20 Page 100

LESSON Framework objectives Cubic graphs


Plot graphs of simple cubic functions.
8.4
Oral and mental starter
Ask the class whether anyone knows the square root of 400.
After some guesses, someone may come up with the correct answer of 20. Ask
him/her how they worked it out.
The quick way to get the solution is to recognise that 400 = 4 100. The square
root of each of these numbers is known, which gives the answer 2 10 = 20.
Ask for the square root of 900. The correct answer of 30 should now be
forthcoming, but may still create a discussion before everyone sees the
reasoning.
Continue in this way, asking for square roots of: 1600 40; 4900 70;
12 100 110; 810 000 900; 90 000 300.
Now ask if anyone can estimate the square root of 377.
Let the question provoke discussion in the class with different suggestions being
put on the board. If the correct answer of 19 is offered, ask how the answer was
worked out.
Explain that the number can be broken down into 3.77 100. The square root of
100 is 10, and the square root of 3.77 can be estimated at between 1 and 2, but
quite close to 2. The approximation 1.9 is quite reasonable, giving the root as
1.9 10 = 19.
Repeat with other examples, such as: 805 28; 548 23; 971 31;
457 21.

Main lesson activity


Put on the board y = mx + c. Ask the class what type of equation this is and what
is special about its graph. They should remember that it is linear and has a
straight line graph.
Then put on the board the equation y = ax2 + bx + c. Ask if anyone knows what
type of equation this is and what is special about its graph.
You want the answer quadratic, and that the graph has a U-shape or an
inverted U-shape.
Then put on the board the equation y = x3 + 4x2, and ask the class if they know
what type of equation this is.
Explain that it is called a cubic equation because the highest power in the
equation is a cube.
Tell the class you are going to go through the drawing of this graph with them.
Start by putting a table on the board, as below.
x 5 4 3 2 1 0 1 2
x3 125 64 27 8 1 0 1 8
4x2 100 64 36 16 4 0 4 16
y = x3 + 4x2 25 0 9 8 3 0 5 24
Complete this table with help from the class. Plot the coordinates given by the
table on a pair of axes drawn on the board. Join up the coordinates to show a
smooth curve with two turning points.
Explain that this is a typical shape for a graph from a cubic equation. However,
sometimes the two turning points coincide, giving the graph a single twist. One
example of such a graph is given by y = x3 + 3x2 + 3x +1 (see Pupil Book page
146). Also tell them that if the x3 part is negative, the graph will start in the
second quadrant (top left-hand corner) instead of in the third quadrant (bottom
left-hand corner.)
Ensure the class realise that this type of graph will always be curved, having
either two bends or a single twist.
The class can now do Exercise 8D from Pupil Book 3.

100 HarperCollinsPublishers Ltd 2003


9B3LP_08.qxd 18/09/2003 15:20 Page 101

Exercise 8D Answers

1 Values of y: 8, 1, 0, 1, 8 2 Values of y: 0, 10, 8, 0, 8, 10, 0


3 Values of y: 3, 3, 3.375, 3, 2.625, 3, 9 4 Values of y: 32, 10, 4, 8, 1, 22
5 x = 1.2 y = 2.7
6 t 0 1 2 3
v 0 2 9 28

SATs Answers

2
1 k = 3, m = 6 2 a 2n b c n2
n
3 a A wider U-shape. Goes through (0,0) and the y-axis is a line of symmetry.
b y = x2 c y = x2 + 1 d y > x2 and y < 2
4 a 0.04 b 1.6 m

Extension Answers

a y b y c y

y = 1x y = x1
2 y = x1
3

x x x

Plenary Key Words


Talk about the shape of a cubic graph always having two turning points even
though they may coincide and act as only one.
cubic
Start a discussion about how many turning points they might expect in the graph turning points
of y = Ax4 + Bx3 + Cx.
Following the pattern linear, no turning points; quadratic, one turning point;
cubic, two turning points; an equation whose highest power was four would
probably have three turning points.
Someone might like to find out!
Homework

1 By drawing suitable graphs, solve this pair of simultaneous equations:


2x + y = 5 y = x3 1
There is only one solution.
2 The distance, d metres, a rocket is above the ground is given by
d = 2t + t3
where t is the time in seconds.
Draw the distancetime graph for the first 3 seconds.

Answers
1 x = 1.45, y = 2.1
2 b t 0 1 2 3
d 0 3 12 33

HarperCollinsPublishers Ltd 2003 101


9B3LP_09.qxd 18/09/2003 15:20 Page 102

CHAPTER
9 Handling Data 2
LESSON Framework objectives Probability statements
Use the vocabulary of probability in interpreting results involving uncertainty and
9.1 prediction.

Oral and mental starter


Use a counting stick as a probability scale.

0 1
Explain to the class that the stick is a probability scale. Tell them that one end is zero. Ask them for the value of
the other end.
Now point at the centre of the stick and ask them to tell you an event which has a probability of 0.5.
(For example, obtaining a Head on a fair coin.)
Now say to the class that you will point out a probability of something that happens and they will have to show
you the probability that it does not happen. For example, you point to 0.1 on the stick, the class would point to
0.9. Repeat this quickly for different values.
Finish by asking why Event happens and Event does not happen is not always at 0.5 (as there are two
choices). Ask the class to give you examples to explain the reason, such as dice P(6) = 16 , P(not 6) = 65 .

Main lesson activity


Remind the class that they should be able to find the probability of a range of situations, all of which they have
already met. Draw their attention to the tabular summaries in Pupil Book 3.
Move on to a brief discussion of independent events, giving them this definition:
Two events are said to be independent when the outcome of one of them does not affect the outcome of
the other event.
Tell the class that most of the combined events they have dealt with so far have been independent. Give as an
example a dice being rolled at the same time as a coin is tossed. Ask them whether they think that the score on
the dice may affect how the coin lands Heads or Tails.
Two other aims of this lesson are to give students an understanding of when statements can be misleading even
though they have apparently been deduced logically, and to increase their familiarity with the vocabulary of
probability.
Write on the board a = b and b = c. Ask the class to give you an equation connecting a and c. They should
respond a = c.
Now replace the statements with the following sentence.
Amy likes Bob and Bob likes chips.
Ask the class to give you a statement connecting Amy and chips! Hopefully, the class will realise that the logic
here does not necessarily work.
Explain that sometimes statements may seem logical but are not necessarily true.
Tell the class that you have five names on five separate pieces of paper, four boys and one girl. Ask them which
statement is true when one piece of paper is picked at random.
There is an even chance of picking the girl because there are both boys and girls (meaning there are two
choices).
There is no chance of picking the girl because she is outnumbered.
The probability of picking the girl is one fifth because there is a one in five chance.
Explain to the class that these are the sort of statements they will have to consider. Their answers will require an
explanation, not just responses such as It is incorrect. Their explanations could include what the true
statements should be.
The class can now do Exercise 9A from Pupil Book 3.

102 HarperCollinsPublishers Ltd 2003


9B3LP_09.qxd 18/09/2003 15:20 Page 103

Exercise 9A Answers

1 a Probability of rolling 6 on a fair dice is 61. The fact that Ashad thinks he is unlucky does not affect his chance of
starting the game
b If this were true, it would rain forever
c Probability of snow depends which part of the world you are in and at what time of year
d Only true if there were equal numbers of mint, chocolate and plain sweets
2 a Incorrect, it could happen today
b Correct, assuming it is a fair coin
c Incorrect, it is possible that the bus could be late tomorrow
d Incorrect, there is an equal chance of picking out red or blue
3 a Not possible to know, as number of winning squares not given for each case
b Grid 1
c Proportion of winning squares is Grid 1 41, Grid 2 31, Grid 3 21. So, greatest chance of winning is using Grid 3
4 a Not independent because when Jonathan writes computer programs he is unlikely to be watching TV at the same
time
b Independent c Independent

Extension Answers

7 (0.28) to produce a winning square


5 by 5 grid with 7 winning squares has a probability of 25

6 by 6 grid with 10 winning squares has a probability of 10 (0.27) to produce a winning square
36
27

10 by 10 grid with 27 winning squares has a probability of 100 (0.27) to produce a winning square
Greatest chance of finding a winning square is on 5 by 5 grid

Plenary Key Words


Ask the class to think of different sentences which have some logic to them but
which are incorrect. (You could give them a hint by asking for a sentence about,
certain
for example, eye colour or shoe size.) even chance
Prompt them with an example of your own, such as: probability
There is a 50% chance that Jon is wearing size 10 shoes because either he probable
is size 10 or he isnt.
Ask a student to explain why the statement is incorrect.
Homework

1 Write down a reason why each of these statements is incorrect.


a A bag contains black and white cubes, so there is a 50% chance of picking a black cube.
b A bag contains black and white cubes. Last time I picked out a black cube, so this time I will pick
out a white cube.
c A bag contains one black cube and many white cubes. So, I have no chance of picking out the
black cube.
2 Here are three different bags of cubes.
A There are four black cubes and four white cubes in the bag.
B There are two black cubes and five white cubes in the bag.
C There are seven black cubes and five white cubes in the bag.
Here are three statements about the bags of cubes.
X There is a probability of 52 that I will pick a black cube.
Y There is an even chance that I will pick a black cube.
Z There is a probability of 125 that I will pick a white cube.
For each bag, say whether the statements are correct or incorrect.

Answers
1 a This would only be correct if there were an equal number of black and white cubes
b Provided there are still some black cubes in the bag, there is a chance that black might be picked out
c As in part b, there is a chance. It would only be impossible if the black cube had been taken from the bag
2 I Incorrect, C Correct
A B C
X I I I
Y C I I
Z I I C

HarperCollinsPublishers Ltd 2003 103


9B3LP_09.qxd 18/09/2003 15:20 Page 104

LESSON Framework objectives Mutually exclusive events and


exhaustive events
9.2 Identify all the mutually exclusive outcomes of an experiment. Know that the sum
of probabilities of all mutually exclusive outcomes is 1 and use this when solving
problems.
Use efficient methods to add, subtract, multiply and divide fractions.

Oral and mental starter


Use a target board as shown on the right.
Ask the class to pick out fractions and decimals which add up to one. 12 0.25 14 0.85 18
Encourage them to give additions which involve more than two 0.15 34 0.125 58 0.625
fractions/decimals: for example, 0.5 + 0.125 + 38 .
Ask them how they worked it out when they mixed fractions with 78 0.1 38 0.5 0.75
decimals.
Now give them one value and ask them to make it add up to 1, with two other
fractions or decimals. They may use either the target board values or their own.
For example: you say 0.2 and they give 0.5 + 0.3.

Main lesson activity


Give the class the example of a bag which contains three blue, two yellow and
five green balls, from which only one ball is allowed to be picked at random.
The probabilities for picking each colour are:
3
P(blue) = 10
P(yellow) = 10 2 = 15
P(green) = 10 = 12
5

The events picking a blue ball, picking a yellow ball and picking a green ball
can never happen at the same time, given that only one ball is allowed to be
taken out. Such events are call mutually exclusive, because they do not overlap.
Using the same example, deal with the probability of an event which will not
happen. Thus, the probability of not picking out a blue ball is give by:
P(not blue) = 10 7
because there are seven outcomes which are not blue balls. Then note that
P(blue) + P(not blue) = 10 3 + 10
7 = 1
So, knowing P(Event happening), then:
P(Event not happening) = 1 P(Event happening)
Now ask the class to sum the probabilities of picking the three coloured balls.
Hopefully, they will get the correct answer of 1. Then tell them that because
there are no other possibilities, they are called exhaustive events.
Tell them that the events are also mutually exclusive and that when events are
both exhaustive and mutually exclusive, their probabilities always add up to 1.
The class can now do Exercise 9B from Pupil Book 3.

104 HarperCollinsPublishers Ltd 2003


9B3LP_09.qxd 18/09/2003 15:20 Page 105

Exercise 9B Answers
Key Words
1 a 0.4 b 0.3 c 0.95
2 a 25b 16
c 25
25
9 d 35 mutually
3 a 0.55 b 0.45 c 0.75 d 0.25 exclusive
4 a Mutually exclusive b Mutually exclusive c Both d Neither
exhaustive
probability
Extension Answers
expectation

5, 20, 40, 24, 60, 90

Plenary
Tell the class that in a box you have a set of mathematical instruments and other equipment: for example, pens,
pencils, pairs of compasses, protractors, and small and large rulers.
You say that in one hand you have, for example, pencils, and in the other hand you have rulers. The class have
to tell you whether the content of your hands is exhaustive, mutually exclusive or neither.
Repeat this with different combinations.
Homework

1 Ten pictures are shown, which are all face down. A picture is picked at random.

a What is the probability of choosing a picture of a guitar?


b What is the probability of choosing a picture of a guitar or a boat?
c What is the probability of choosing a picture of a horse or a doll?
d What is the probability of choosing a picture which is not of a boat?
2 A bag contains a large number of discs, each labelled either A, B, C or D. The probabilities that
a disc picked at random will have a given letter are shown below.
P(A) = 0.2 P(B) = 0.4 P(C) = 0.15 P(D) = ?
a What is the probability of choosing a disc with a letter D on it?
b What is the probability of choosing a disc with a letter A or B on it?
c What is the probability of choosing a disc which does not have the letter C on it?

Answers
3 b 10
1 a 10 7 c 10
3 d 35
2 a 0.25 b 0.6 c 0.85

HarperCollinsPublishers Ltd 2003 105


9B3LP_09.qxd 18/09/2003 15:20 Page 106

LESSON Framework objectives Combining probabilities and tree


diagrams
9.3 Identify all mutually exclusive outcomes of an experiment.
Use efficient methods to add, subtract, multiply and divide fractions. Cancel
common factors before multiplying or dividing.

Oral and mental starter


Write some fractions on the board: for example 62, 10 5 and 86.
Ask the class to tell you each fraction in its simplest form, or ask them to cancel
each fraction down to its simplest form.
Then ask them how they did it.
Now ask them to give you equivalent fractions to, for example, 21 , 51 and 72.
Next, invite the class to give you the answer to 21 32. You could remind them that
this is the same as 21 of 32. Ask them how they worked it out. Hopefully, their
answers will cover different methods, such as: knowing that 21 of 32 is 31;
multiplying the numerators together and then the denominators to obtain
62 which is cancelled down to 31; and cancelling the 2s first to obtain 11 31 and
then 31.
Now write 92 43 on the board. Tell the class that the answer they give must be in
simplest form.
This activity can be repeated for different fractions. Hopefully, the class will
recognise that it is easier and more efficient to simplify the fractions before doing
the multiplication.

Main lesson activity


Explain that the aim of this lesson is to look at probability questions: in
particular, to solve problems which involve two or more events, using different
methods. The class will also be working with fractions and may wish to refer to
chapter 2 to remind themselves of the various methods for simplifying fractions.
Start by explaining the meaning of the word independent. You could ask the
class to write down a definition. For example: Two events are independent if the
outcome of one is not affected by the outcome of the other.
Draw a blank sample space diagram on the board for two events, such as rolling
a fair dice and throwing a fair coin.
Dice
1 2 3 4 5 6
Head
Coin
Tail
Point out that there are twelve different outcomes so that the probability of, for
example, a Head and a 6 is 12 1.
Now draw a tree diagram on the board, as shown on the right. Coin Dice
Explain that it is important for the diagram to have all the labels, 6
16
Coin, Dice, Head, Tail, etc. and all the probabilities, completed.
The class should help to complete the details. Head
12 56
Now explain that when working across the branches the Not a 6
probability of getting a Head and a 6 is 21 61 = 12
1, as before.
12 6
16
Ask the class to tell you the other combinations. Then explain Tail
that this information could also be put into a table, as shown below.
56
Dice Not a 6
P(6) = 61 P(Not a 6) = 65

P(Head) = 21 21 61 = 12
1 21 65 = 12
5
Coin
P(Tail) = 21 21 =
61 1
12 21 65 = 12
5

Explain to the class that they are all valid methods. Tell them that unless they are
told what to do, they may use whichever method they prefer.
The class can now do Exercise 9C from Pupil Book 3.

106 HarperCollinsPublishers Ltd 2003


9B3LP_09.qxd 18/09/2003 15:20 Page 107

Exercise 9C Answers

1 First dice Second dice 2 a 43 b 13 c 16


Even 3 a
12 Second bag
Red 15 Blue 45
5 or 6 12 1 1
13 First bag
Red 2
10 25
Not even
1 1 25
Blue 2
10
23 Even
Not 5 or 6 12 1
b 10
c 101 + 25 = 12

12
Not even
13 12 = 16

Extension Answers

a First game Second game Third game b WWL, WLW, LWW, WWW 4 + 27
c 27 4 + 27
4 + 27
8 = 27
20

23 Win
23 Win 13 Lose
13 Lose 23 Win
23 Win
13 Lose
13 Lose 23 Win
23 Win
13 Lose
13 Lose 23 Win

13 Lose

Plenary Key Words


Explain to the class that they have been looking in Exercise 9C at two events. Then
show them the tree diagram in the extension question which takes in three events. tree diagram
Ask the class how many different combinations there are in this example. Point mutually
out that 23 = 8. Then invite them to tell you how many combinations they would exclusive
expect for four events (16). independent
Explain that they can either work out the probabilities for every possible two-way table
combination or just the combinations required to answer the question.
Homework

1 A builder is working on a patio. The probability that the weather is fine is 0.6, and the probability
that he has all the materials is 0.9. To complete the job in a day, he needs the weather to be fine
and to have all the materials.
a Draw a tree diagram to show all the possibilities.
b Calculate the probability that he completes the job in a day.
c Calculate the probability that it is not fine and he does not have all the materials.
2 A game is played three times. The probability of winning each time is 12.
a Show that the probability of winning all three games is 18.
b What is the probability of winning exactly one game?

Answers
1 a Weather Materials
0.9 Has all
materials
0.6 0.1 Does not have
Fine all materials
0.4 Not fine 0.9 Has all
materials
0.1 Does not have
all materials
b 0.54 c 0.04 2 a 12 12 12 = 18 b 38

HarperCollinsPublishers Ltd 2003 107


9B3LP_09.qxd 18/09/2003 15:20 Page 108

LESSON Framework objectives Estimates of probability


Estimate probabilities from experimental data.
2.2
9.4 Understand relative frequency as an estimate of probability and use this to compare
outcomes of experiments.

Oral and mental starter


Tell the class that in a test someone scored 7 out of 10. Ask a student to give you the score as a fraction of 10.
Ask another student to give you this score as a decimal.
9, 10
Repeat this for other simple scores, leading to, for example, 10 4, 100
24
32
and 50
.
Gradually increase the level of difficulty up to scores which will need rounding: for example, 20 out of 30.
Encourage the students to round to two decimal places.
7 of 100, 52 of 20.
Finally, ask them to work out mentally simple fractions of quantities: for example, 10

Main lesson activity


The aim is to recall the work on experimental probability, to introduce the term relative frequency and to move
on to expectation.
Ask the class to consider a coin which is tossed ten times and lands on Heads nine times. Ask them whether the
coin is biased. They will probably say that it is.
Now suggest that when the coin is tossed a further ten times, it landed on Heads only once. Ask the class
whether they still think that it is biased. They will probably say that you cannot tell.
Ask the class how they could improve the experiment. Hopefully, they will tell you to carry out more trials.
Tell them that relative frequency is about carrying out repeated trials and obtaining estimates of probability
from experimental data.
Write down on the board the formula:
Number of successful trials
Relative frequency =
Total number of trials
Emphasise that the greater the number of trials, the closer the estimates of probability get to the theoretical
probability.
Draw a graph showing relative frequency against number of trials on the board and show the class how to
record the results. Use the coin example, plotting at 10 trials, 20 trials and so on. Point out to the class that a
relative frequency graph will usually approach a value and that this value is the best estimate of the probability.
Ask the class why it is not valid to use the graph to read off at say, 15 trials. Hopefully they will realise that that
data between 10 and 20 trials may not follow the steady trend line.
Now say to the class that an experiment has been carried out many times and you believe that the estimate is
70
reliable. For example, the coin landed on Heads 70 times out of 100, so the relative frequency is 100 . Ask them
how many times they would expect this coin to land on Heads were it tossed 200 times. Stress that you are
using the estimate of probability and not the theoretical probability.
Then tell them that the expected number of successes can be calculated using the formula:
Expected number of successes = Relative frequency Number of trials
The class can now do Exercise 9D from Pupil Book 3.

Exercise 9D Answers

32
1 a 100
b Yes, it is probably biased: more 4s than other scores c 160 (32 5)
2 a Number on throws 10 20 30 40 50 b 0.64 c 0.64 200 = 128
Number of times it lands point up 6 13 20 24 32
Relative frequency of landing point up 0.6 0.65 0.67 0.6 0.64

3 a Number of trials 10 25 50 100 b 0.28 c 0.28 75 = 21


Number of times blue cube chosen 3 8 15 28
Relative frequency 0.3 0.32 0.3 0.28

4 a Relative frequency of Heads 7 = 0.7 12


= 0.6 18
= 0.6 22
28
= 0.56
= 0.55 50
10 20 30 40

108 HarperCollinsPublishers Ltd 2003


9B3LP_09.qxd 18/09/2003 15:20 Page 109

Exercise 9D (Contd)

b 5 a
1 1

Relative frequency
Relative frequency 0.8 0.8

0.6 0.6

0.4 0.4

0.2 0.2

0 0
0 10 20 30 40 50 0 2 4 6 8 10
Numbers of throws Number of games

c 0.56 d 112 b Only know one out of first two games was won, not
e Probably, as best estimate is not 0.5 which one c 0.70
d Number of wins 1 3 4 6 7

Extension Answers

a 6 b 0.65 c 65, assuming player performs as on graph


d Relative frequency not plotted for 15 throws, so cannot find number of hits

SATs Answers

7 b 336 (accept 332 to 340)


1 a 25 4 a 12 12 12 = 18 b WWL or WLW or LWW = 18 + 18 + 18 = 38
2 a This would give 2.5 blue counters 5 a Win, Lose, Lose b 45% c 0.6
b 1 blue, 6 red, 9 green, 4 yellow d 0.6
171
c 167, 125, 8
3 a Sue, most throws b 300
d Experiment, therefore subject to random variation,
and theoretical results do not give whole-number
1
0.4
answers in this case e 1296

Plenary Key Words


Refer to the question in the extension work, which involves taking relative
frequency from a graph. relative
Explain to the class that relative frequency questions can be asked using frequency
experimental data either collected in a table or presented as a graph. Point out estimate
that the relative frequency plots can be read from a graph and that the number of probability
successful trials can be calculated by working backwards. expectation
Discuss briefly why it is not possible to read values at intermediate points on a
limit
graph.
Homework

A spinner has different coloured sectors. It is spun 100 times and the number of times it lands on blue
is recorded at regular intervals. The results are shown in the table.
Number of spins 20 40 60 80 100
Number of times lands on blue 6 10 15 22 26
Relative frequency 0.3
a Copy and complete the table.
b What is the best estimate of the probability of landing on blue?
c How many times would you expect the spinner to land on blue in 2000 spins?
d If there are two sectors of the spinner coloured blue, how many sectors do you think there are
altogether? Explain your answer.
Answers
a Relative frequency 0.3 0.25 0.25 0.275 0.26
b 0.26 c 0.26 2000 = 520
d Given spinner is fair, 0.26 41, so a quarter of sectors are blue. So, altogether eight sectors.

HarperCollinsPublishers Ltd 2003 109


9B3LP_10.qxd 18/09/2003 15:21 Page 110

CHAPTER
10 Shape, Space and Measures 3
LESSON Framework objectives Fractional enlargements
Know that if two 2-D shapes are similar, corresponding angles are equal and
10.1 corresponding sides are in the same ratio.
Enlarge 2-D shapes, given a fractional scale factor; recognise the similarity of the
resulting shapes; understand the implications of enlargement for area and volume.

Oral and mental starter


Imagine a cube which has an edge length of 2 cm. What is the volume of the
cube? (Answer: 8 cm3)
Now imagine the cube is twice as big. What is its volume now?
(Answer: 64 cm3)
Make sure that the students appreciate that twice as big means that the edges of
the cube are multiplied by two.
This starter can be extended by asking the class to make the original cube three
times or four times larger and allowing them to work out their answers on paper.

Main lesson activity


Remind the class about enlargement, which was covered in Year 8.
It may be necessary to revise enlargement, using the ray A
method, by drawing on the board the diagram shown A Image
on the right, or show a prepared OHT. Object
State that the object triangle ABC has been enlarged by O C
a scale factor of 2 about the centre of enlargement, O, B C
to give the image triangle ABC. The dashed lines are B
called the rays or guidelines for the enlargement. The
object and image are on the same side of O. The scale
factor is positive. This is called positive enlargement. A
Next, explain to the class that there is fractional enlargement. A
Draw on the board the diagram shown on the right, Image Object
or show a prepared OHT. O B B
Explain that each side of ABC is half the
length of the corresponding side of ABC. C
C
1 1 1
Also that OA = 2 of OA, OB = 2 of OB and OC = 2 of OC.

That is, ABC has been enlarged by a scale factor of 12 about the centre of
enlargement, O, to give the image ABC. The object and the image are on the
same side of O, with the image smaller than the object. The scale factor is a
fraction. This is called fractional enlargement.
Point out that, under any enlargement, corresponding angles on the object and
image remain the same.
Now demonstrate fractional enlargement on a grid. Explain that the grid may or
may not have coordinate axes x and y.
Show the class how the image points can be easily found by counting grid units
in the vertical and horizontal directions. This can be an alternative to drawing
rays.
When there are coordinate axes, the centre of enlargement is sometimes given
as the origin (0, 0). The coordinates of the image shape are then the coordinates
of the object shape multiplied by the fractional scale factor.
The class can now do Exercise 10A from Pupil Book 3.

110 HarperCollinsPublishers Ltd 2003


9B3LP_10.qxd 18/09/2003 15:21 Page 111

Exercise 10A Answers

3 a Vertices at (3, 5), (5, 3), (1, 3) b Vertices at (1, 3), (3, 3), (3, 2), (1, 2)
c Vertices at (1, 3), (3, 2), (1, 2)
4 a 12 b (2, 2) c 16 cm2 and 4 cm2, 4:1 d 14

Extension Answers

2 a i 22 cm2 ii 6 cm3 b i 88 cm2, 4 ii 48 cm3, 8


c i 198 cm2, 9 ii 162 cm3, 27 d i k 2 ii k 3

Plenary Key Words


Ask what the class knows about each of these when making enlargements.
object
The relationship between lengths on the image and lengths on the object.
The relationship between angles on the image and angles on the object.
image
Fractional scale factor (makes a shape smaller on the same side of the centre centre of
of enlargement). enlargement
scale factor
fractional
enlargement
similar
Homework

1 Draw copies of (or trace) each of the following 2 Copy the diagram below onto a coordinate
shapes. Enlarge each one by the given scale grid and enlarge the triangle by scale factor 112
factor about the centre of enlargement O. about the origin (0, 0).
a Scale factor 13 8

7
O
6

4
b Scale factor 12 3

2
O
1

0
0 1 2 3 4 5 6 7 8

Answers
2 Vertices at (3, 3), (3, 6), (7.5, 3)

HarperCollinsPublishers Ltd 2003 111


9B3LP_10.qxd 18/09/2003 15:21 Page 112

LESSON Framework objectives Trigonometry: The tangent of an angle


Begin to use (sine, cosine and) tangent in right-angled triangles to solve problems in
10.2 two dimensions.

Oral and mental starter


On the board draw the table below, or use a prepared OHT.
1:10 1:8 1:12 2:80 1:4
8:200 1:2 20:720 1:5 8:800
1:20 10:300 5:50 1:36 5:25
1:40 20:80 1:30 100:800 10:500
40:80 4:48 1:100 10:200 1:25
Ask individual students to cross out the pairs of ratios that are identical.
The last student asked should then simplify the remaining ratio.

Main lesson activity


NB You may wish to use more than one lesson to cover the material in this lesson plan and the associated
pupil book.
Explain to the class that the next few lessons will be about an important branch of Mathematics called
Trigonometry which is used to calculate the lengths of sides and the size of angles in right-angled triangles. Its
main use is in areas of engineering, navigation and surveying.
Draw the right-angled triangle on the right on the board.
Explain the following terms for the right-angled triangle ABC:
The side opposite the right angle is always the longest side Hypotenuse (H)
and is known as the hypotenuse (H) Opposite side (O)
The side opposite the angle in question (labelled ) is called the 
opposite side (O) Adjacent side (A)
The side next to the angle in question is called the adjacent side (A)
Draw another triangle on the board with A labelled . Ask the students to point out the opposite and adjacent
sides.
Introduce the investigation on page 171 of Pupil Book 3. It will be useful if students can work in pairs or small
groups, as some discussion is helpful.
Summarise what the investigation has shown by explaining that this value is called the tangent of angle A, and
is shortened to tanA.
Tell the students that the value of the tangent of any angle is stored in their calculators.
Show the students how to find the tangent of angles using their calculators. Some students may need individual
help as different models have different key sequences. The pupil book shows some of the more common
sequences. Work through some examples with the students to ensure that they have all mastered using their
calculators for this.
Opposite H
Using a diagram similar to the one shown here, explain that the formula tan = O
Adjacent
can be used in conjunction with the tan key to find the lengths of sides and the size of 
angles in right-angled triangles. A
Explain that once they have found tan, they will need to find the value of  and to do this they will need to
use the inverse tan function on their calculators. Ensure that all of the students know how to do this and work
through some examples, such as: find the value of  when tan = 0.6.
Show the class how to calculate  in the diagram below and explain that angles are usually given to 1 decimal
place.
Opposite
Using the formula: tan =
Adjacent 4 cm
4
= = 0.667
6 
6 cm
So,  = 33.7 (1dp)
Make sure that the students know how to use their calculators to do this.

112 HarperCollinsPublishers Ltd 2003


9B3LP_10.qxd 18/09/2003 15:21 Page 113

Now show the class how to calculate x in the diagram below and explain that
lengths are usually given to three significant figures.
Opposite
Using the formula: tan =
Adjacent
x
x
tan60 =
4 60
Multiplying both sides by 4, gives: 4tan60 = x 4 cm
So, x = 6.93 cm (3sf)
Make sure that the students know how to use their calculators to do this.
The class can now do Exercise 10B from Pupil Book 3.

Exercise 10B Answers

1 a 0.532 b 1.732 c 1 d 0.158 e 0.759 f 11.664


2 a 26.6 b 13.0 c 40.2 d 56.3 e 66.8 f 84.3
3 a 1.51 b 3.73 c 10.0 d 39.9 e 6.09 f 86.6
4 a 16.7 b 26.6 c 24.4 d 74.1 e 45.0 f 66.8
5 a 4.25 cm b 3.25 cm c 14.0 cm d 32.0 cm e 3.29 cm f 40.5 cm
6 31.0 7 10.7 cm

Extension Answers Key Words


1 a 6.93 cm b 17.0 cm c 2.77 cm d 28.4 cm adjacent
2 a 56.2 m b 78.7
angle of
depression
angle of
Plenary elevation
Explain that tan is the ratio between the opposite side and the adjacent side in a hypotenuse
right-angled triangle. opposite
Tell the students that next lesson they will be looking at two other trigonometric tangent (tan)
ratios that will involve using the hypotenuse.
Homework

1 Find the value of  for each of the following. Give your answers to 1 decimal place.
a tan = 0.12 b tan = 0.956 c tan = 3.45
2 Find the value of each of the following. Give your answers to 3 significant figures.
a 5tan31 b 14tan51 c 23tan58.9
3 Calculate the angle marked  in each of the following triangles. Give your answers to 1 decimal place.
a b c
4 cm
 10 cm 12 cm
10 cm
 
18 cm 5 cm
4 Calculate the length of the side marked x in each of the following triangles. Give your answers to
3 significant figures.
a b c
x
x x
25
7 cm 15
62
9.5 cm
14.2 cm

Answers
1 a 6.8 b 43.7 c 73.8 2 a 3.00 b 17.3 c 38.1
3 a 21.8 b 29.1 c 67.4 4 a 3.26 cm b 2.55 cm c 26.7 cm

HarperCollinsPublishers Ltd 2003 113


9B3LP_10.qxd 18/09/2003 15:21 Page 114

LESSON Framework objectives Trigonometry: The sine and cosine of an


angle
10.3 Begin to use sine, cosine (and tangent) in right-angled triangles to solve problems in
two dimensions.

Oral and mental starter


Draw the triangle on the right on the board or on a prepared OHT.
Let the class work in pairs or small groups and ask them to work out how the sin H
and cos keys on their calculators can be used to find lengths and angles. O
Allow a five-minute discussion. 
A

Main lesson activity


NB You may wish to use more than one lesson to cover the material in this
lesson plan and the associated pupil book.
O
Remind the class that the tangent of the angle  = tan = and that this is the H
A O
ratio of two sides of a right-angled triangle.
Point out to the students that the tangent does not include the hypotenuse, 
A
H, and tell them that there are two trigonometric ratios, which do include H.
Using the same diagram, tell the students that:
O
the ratio is called the sine of the angle  and is written as sin
H
A
the ratio is called the cosine of the angle  and is written as cos
H
Explain that the sine and cosine are used in the same way as the tangent to find
the lengths of sides and size of angles in right-angled triangles when the length
of the hypotenuse is involved.
Explain to the class that the sin and cos keys on their calculators are used in
exactly the same way as the tan key.
The three trigonometric ratios should now be summarised as follows:
O A O
sin = cos = tan =
H H H A
O

A
Show the class how to do the following two examples:
Example 1 Calculate the angle marked  in the diagram on the right. Give
your answer to 1 decimal place.
10 cm
The adjacent side and the hypotenuse are given, so use cosine:
Adjacent 8
cos = = = 0.8 
Hypotenuse 10 8 cm
So,  = 36.9 (1dp)
Example 2 Calculate the length of the side marked x on the diagram on the
right, giving your answer to 3 significant figures.
5 cm
The angle and the hypotenuse are given, and the opposite side is x
required, so use sine:
38
Opposite
sin =
Hypotenuse
x
sin 38 =
5
Multiply both sides by 5 to give: 5sin38 = x
So, x = 3.08 cm (3sf)
The class can now do Exercise 10C from Pupil Book 3.

114 HarperCollinsPublishers Ltd 2003


9B3LP_10.qxd 18/09/2003 15:21 Page 115

Exercise 10C Answers

1 a 0.309 b 0.500 c 0.912 d 0.875 e 0.500 f 0.080


2 a 5.7 b 32.0 c 58.8 d 66.4 e 47.2 f 16.7
3 a 1.59 b 3.54 c 12.2 d 1.98 e 3.57 f 9.00
4 a 17.5 b 34.8 c 28.0 d 75.5 e 40.9 f 69.7
5 a 3.80 cm b 4.62 cm c 14.8 cm d 1.97 cm e 8.23 cm f 12.3 cm
6 19.0 cm
7 49.3

Extension Answers

1 a 12.8 cm b 15.9 cm c 9.23 cm d 49.0 cm


2 a  0 10 20 30 40 50 60 70 80 90
sin 0.00 0.17 0.34 0.50 0.64 0.77 0.87 0.94 0.98 1.00
cos 1.00 0.98 0.94 0.87 0.77 0.64 0.50 0.34 0.17 0.00
b sin = cos(90 )

Plenary Key Words


Ask the class to give a summary of the three trigonometric ratios.
adjacent
hypotenuse
opposite
cosine (cos)
sine (sin)
Homework

1 Find the value of  for each of the following. Give your answers to 1 decimal place.
a sin = 0.25 b sin = 0.854 c cos = 0.752 d cos = 0.235
2 Find the value of each of the following. Give your answers to 3 significant figures.
a 5sin62 b 12sin52.6 c 21cos86 d 3.7cos42.3
3 Calculate the angle marked  in each of the following triangles. Give your answers to 1 decimal
place.
a b c
9 cm
15 cm 28 cm
3 cm


 25 cm
13 cm
4 Calculate the length of the side marked x in each of the following. Give your answers to 3
significant figures.
a b c
11 cm 4.8 cm
x x 42 cm
31 17
55
x

Answers
1 a 14.5 b 58.6 c 41.2 d 76.4
2 a 4.41 b 9.53 c 1.46 d 2.74
3 a 19.5 b 29.9 c 26.8
4 a 5.67 cm b 1.40 cm c 24.1 cm

HarperCollinsPublishers Ltd 2003 115


9B3LP_10.qxd 18/09/2003 15:21 Page 116

LESSON Framework objectives Solving problems using trigonometry


Begin to use sine, cosine and tangent in right-angled triangles to solve problems in
10.4 two dimensions.

Oral and mental starter


Draw the triangle on the right on the board or on a prepared OHT.
Ask the class to write down in their books or on their whiteboards the three H
trigonometric ratios for the angle . O
Check their answers. 
A

Main lesson activity


Explain to the class that the lesson is about solving problems using trigonometry.
Go through the following stages which should be taken when solving a problem
involving trigonometry:
1. Draw a sketch of the right-angled triangle in the problem. Tell the students
that even when a diagram or picture accompanies the problem, it is a good
idea to redraw the triangle.
2. Mark on the sketch all the known sides and angles, including the units.
3. Identify the unknown side or angle by labelling it x or .
4. Decide and write down which ratio you need to use to solve the problem. At
this point you may wish to introduce some mnemonics which the students
can use to help them remember the three ratios.
5. Solve the problem and give your answer to a suitable degree of accuracy.
This is usually three significant figures for lengths and one decimal place for
angles.
Demonstrate how to follow these stages by working through an example, such
as the one given below.
Example A window cleaner has a ladder that is 6 m long. He leans it against
a wall so that the foot of the ladder is 2 m from the wall. Calculate
the angle the ladder makes with the wall.
1/2 Draw a sketch for the problem and write on all the known
sides and angles:

6m

2m
3 Identify the angle required by labelling it :


6 m (H)

2 m (O)
4 Identify the trigonometric ratio to be used:
The opposite and hypotenuse are known, so sine should be
O
used to solve the problem. The ratio required is sin = .
H
5 Solve the problem:
2
sin = = 0.333.
6
So,  = 19.5 (1dp).

The class can now do Exercise 10D from Pupil Book 3.

116 HarperCollinsPublishers Ltd 2003


9B3LP_10.qxd 18/09/2003 15:21 Page 117

Exercise 10D Answers

1 7.22 m 2 69.7 m 3 40.0 4 058 5 24.4 m 6 58.1


7 a 7.19 cm b 7.01 cm c 25.2 cm2

Extension Answers

1 a 238 km b 26.5 km c 239 km


2 a 3.44 cm b 8.60 cm2 c 43.0 cm2

SATs Answers

1 a 4 cm b 40 c 12 cm
2 a 10:14 = 5:7 and 8:12 = 2:3 so corresponding sides are not in the same ratio
b 11.2 cm
3 a 9.98 m b 2.9
4 a 056 b 1.40 km
5 a AC2 = 62 + 82 = 100, so AC = 10 cm b 11.7 cm c 5.9

Plenary Key Words


Write on the board SOHCAHTOA or a similar mnemonic.
cosine (cos)
Ask individual students to explain how the mnemonic can be used to remember
the three trigonometric ratios. sine (sin)
tangent (tan)
Homework

1 The stays on a flagpole are 10 m long and make an angle of 65


with the horizontal ground.
Calculate the height of the flagpole. 10 m

65

2 The diagram on the right shows a ramp for wheelchairs. 25 cm


Calculate the angle the ramp makes with the ground. 1.3 m
3 A helicopter takes off from an army base on a bearing of 075 and
flies for 52 km.
a How far east has the helicopter flown?
b How far north has the helicopter flown?
4 A plane takes off from an airport, climbing at a constant angle. When the plane has flown for
3.2 km, it reaches an altitude of 1000 m. Calculate the angle at which the plane is climbing.
5 The diagram on the right shows a wooden truss of a roof.
Calculate the height, h, of the roof. h
25 25
9.8 m

Answers
1 9.06 m 2 10.9 3 a 50.2 km b 13.5 km 4 18.2 5 2.28 m

HarperCollinsPublishers Ltd 2003 117


9B3LP_11.qxd 18/09/2003 15:21 Page 118

CHAPTER
11 Algebra 5
LESSON Framework objectives Expansion
Simplify or transform algebraic expressions by taking out single-term common
11.1 factors.
Square a linear expression, expand the product of two linear expressions of the
form x n and simplify the corresponding quadratic expression. Establish identities
such as a2 b2 = (a + b)(a b).

Oral and mental starter


Ask the class how many pounds equal one kilogram in weight. Accept the answer 2 as a start, and work
towards (or point out) the correct answer, which is 2.2.
Now ask what the equivalent of 6 kg is in pounds. Ask them how they worked this out, probably 6 2 = 12
added to 6 0.2 = 1.2, giving 13.2.
Using a target board such as the one shown below, work your way around the class asking individual students
for pound equivalents of kilogram weights.
8 5 15 4 7
3 12 9 11 20
10 13 100 2 3

Main lesson activity


Draw a rectangle on the board, showing the dimensions 4 cm and 5 cm. Ask the class to give you the area of
the rectangle. When someone gives you the correct answer (20 cm2), ask them how they calculated it
(4 cm 5 cm).
Draw a second rectangle on the board, this time showing the dimensions 4x and 3 cm. Again, ask the class for
the area of the rectangle. This should provoke some discussion leading to the correct answer of 12x. (The units
are not an important issue for this lesson, so ignore them.)
Now draw on the board a rectangle with the dimensions 3x + 2 and 5. Again, ask the class for the area of the
rectangle. The correct answer (15x + 10) will be seen by some students more easily than others.
Show how this rectangle can be split up, using the 3x and the 2 to give two rectangles one with the
dimensions 3x and 5, and the other with the dimensions 2 and 5. This should help all the students see that they
need to find the area of each smaller rectangle and add them together to give the total area (15x + 10).
Show that the last example is the same as 5(3x + 2) and link this to the finding of the area using two rectangles.
Show that the brackets are expanded by multiplying each term by 5 to arrive at 15x + 10.
Now draw on the board a rectangle with the dimensions 4x + 3 and 2x. Again, ask the class for the area of the
rectangle. Lead the students through the splitting of the rectangle to give two rectangles one with the
dimensions 4x and 2x and the other with the dimensions 3 and 2x. These can then be multiplied to find the
area of each rectangle and added together to give the total area 8x2 + 6x. Again, show the link to the expansion
of the bracket 2x(4x + 3).
Now put on the board the expression 3(4x + 2) and ask the class what this might mean. You want the response:
The area of a rectangle with dimensions 3 and 4x + 2. You also want them to refer to the expansion of the
bracket to give 12x + 6.
Repeat the process for x(3x + 2) to give 3x2 + 2x.
Put on the board the expression 3(4x 1) and ask the class what this might represent. One answer is a rectangle
with sides 4x 1 and 3. Show how, if this is split into two rectangles, the larger one has the dimensions 4x by 3
with the smaller one, the dimensions 1 and 3, taken from it.
Show that the expansion of 3(4x 1) can be done in the same way as before to give 12x 3.
The class can now do Exercise 11A from Pupil Book 3.

118 HarperCollinsPublishers Ltd 2003


9B3LP_11.qxd 18/09/2003 15:21 Page 119

Exercise 11A Answers

a1 3x + 6 b 5t + 20 c 4m + 12 d 2y + 14 e 12 + 4m f 6 + 3k g 5 + 5t h 14 + 7x
a2 2x 6 b 4t 12 c 3m 12 d 6y 30 e 20 5m f 6 2k g 8 4t h 15 3x
a3 8x + 8 b 18t 24 c 10m 15 d 9y + 21 e 9 9m f 8 + 16k g 6 12t h 4 + 6x
a4 12t + 3 b 15x + 10 c 10x 2 d 24x 8 e 28t 14
a5 x2 + 3x b t 2 + 5t c m2 + 4m d y 2 + 8y e 2m + m2 f 3k + k2 g 2t + t2 h 5x + x2
a6 x2 2x b t2 4t c m2 3m d y2 6y e 5m m2 f 2k k2 g 3t t2 h 6x x2
a7 4x2 + 3x b 2t2 3t c 3m2 2m d 4y2 + 5y e 4m 5m2 f 3k + 2k2 g 4t 3t2 h x + 4x2
a8 2x2 + 3x b 5t 3t2 c 4m + 5m2 d 7k2 2k
a9 9x + 14 b 10t + 27 c 18 8m d 2k + 26 e 4x 12 f 9x 7 g 6 x h 8 2x i 13m + 2 j 16m 3
k 4x 7 l 6x 36 m 9x 14 n 14x 19
10 a AB = y 5 b AB = 3y c AB = y + 1 d AB = 4y + 1
CD = 4x 1 CD = x + 3 CD = 3x 3 CD = 3x + 1

Extension Answers

1 One possible method for each is shown.


a 1a + 1b = ab a = (b
b + ab + a)
ab = ab
(a + b)

a c ad bc (ad + bc)
b b + d = bd + bd
= bd

2 b It is always the same as the starting number.
c x [multiply by 3] = 3x [add 15] = 3x + 15 [divide by 3] = x + 5 [subtract 5] = x

Plenary Key Words


Put on the board a rectangle with the dimensions 3x + 7 and 4x. Ask the class for
dimension
the area of the rectangle.
Now put on the board a rectangle with the dimensions x + 7 and x + 3. Again, expansion
ask the class for the area of the rectangle.
This should provoke discussion whereby the rectangle with dimensions x + 7
and x + 3 is split into four rectangles with a total area of x2 + 7x + 3x + 21.
Homework

1 Expand each of the following.


a x(3x + 4) b t(3t 1) c m(4m 3) d y(5y + 3)
e m(5 4m) f k(1 + 6k) g t(3 4t) h x(2 + 5x)
2 Expand and simplify each of the following.
a 3(m + 2) + 2(1 3m) b 4(2k + 3) + 2(1 3k)
c 5(3x 2) + 3(2 4x) d 4(5x + 2) + 5(1 5x)
3 Write down the missing lengths in each of the following rectangles.
a 3x A ? B
b 3x + 1 A ? B

2y
3
4y D
3y + 5
C ?
?
D C
7x + 5 8x + 4

Answers
1 a 3x2 + 4x b 3t 2 t c 4m2 3m d 5y2 + 3y e 5m 4m2 f k + 6k2 g 3t 4t2 h 2x + 5x2
2 a 8 3m b 2k + 14 c 3x 4 d 13 5x
3 a AB = 4x + 5, CD = 4y 3 b AB = 5x + 3, CD = y + 5

HarperCollinsPublishers Ltd 2003 119


9B3LP_11.qxd 18/09/2003 15:21 Page 120

LESSON Framework objectives Factorisation


Simplify or transform algebraic expressions by taking out single-term common
11.2 factors; add simple algebraic expressions.
Square a linear expression, expand the product of two linear expressions of the
form x n and simplify the corresponding quadratic expression. Establish identities
such as a2 b2 = (a + b)(a b).

Oral and mental starter


A formula for the approximate conversion of temperatures from degrees
Fahrenheit to degrees Celsius is C = 12 (F 32), where C is the temperature in
degrees Celsius and F is the temperature in degrees Fahrenheit.
Ask the class to use this formula to estimate the equivalent of 100 Fahrenheit in
degrees Celsius (Answer 34). Discuss with the class the strategy they used to do
this mentally.
Work through a couple more examples, such as:

66 30 36 2 34 2 17

90 30 60 2 58 2 29

Using a target board such as the one shown below, work your way around the
class asking the students to convert temperatures in degrees Fahrenheit to their
approximate equivalents in degrees Celsius.
34 109 38 40 73
55 42 76 89 50
61 32 57 71 88
99 93 103 67 72

Main lesson activity


Put on the board a rectangle with 7 cm2 written inside the outline. Ask the class
what dimensions the rectangle could have. The simplest answer is 1 cm by
7 cm. Explain that coming up with these numbers involves finding a pair of
factors.
Now put on the board a rectangle with the area 12 cm2 written inside. Again,
ask the class what dimensions the rectangle could have. There is more than one
simple choice here, and any factor pair will do (for example, 2 cm by 6 cm,
3 cm by 4 cm).
Now put on the board a rectangle with the expression 3x + 3 written inside. Tell
the class that this is the area of the rectangle and ask what dimensions the
rectangle could have. Lead the discussion so that the class reaches the correct
answer of 3 and x + 1.
In order to reach the answer above, a pair of factors has to be found which,
when multiplied together, give 3x + 3. Explain that this is called factorisation
and is the opposite process of expansion, which they covered in the previous
lesson.
Next, put on the board the expression 6x + 9. Invite them to imagine that this is
the area of a rectangle. Ask them what the dimensions of the rectangle could be.
They need to factorise the expression to create a bracket with two factors inside
and a term outside. For this example, factorisation gives 3(2x + 3). Show that this
expands to give 6x + 9.
Now put on the board the expression x2 + 5x and again ask what dimensions a
rectangle of this area could have. Help the class to see that this factorisation will
be x(x + 5), again showing that the expansion will give the original expression.
The class can now do Exercise 11B from Pupil Book 3.

120 HarperCollinsPublishers Ltd 2003


9B3LP_11.qxd 18/09/2003 15:21 Page 121

Exercise 11B Answers

1 a 3( x + 2) b 2(2t + 3) c 4(m + 2) d 5( y + 2) e 2(4 + m) f 3(1 + 2k)


g 5(1 + 3t) h 3(4 + x)
2 a 2( x 2) b 4( t 3) c 3(m 3) d 3(2y 3) e 7(2 m) f 3(7 k)
g 4(3 2t) h 3(5 x)
3 a 3(4x + 1) b 2(3t 2) c 3(3m 1) d 3( y + 2) e 3(5 m) f 4(3 + k)
g 2(3 t) h 3(9 + x)
4 a 3x + 4 b 5 + 3t c 2m 3 d 4 2t
5 a x( x + 3) b t( t + 4) c m(m + 5) d y( y + 7) e m(3 + m) f k(4 + k)
g t(3 + t) h x(1 + x)
6 a x( x 3) b t(3t 5) c m(m 2) d y(4y 5) e m(2 m) f k(4 3k)
g t(5 t) h x(7 4x)
7 a x(3x + 4) b t(5t 3) c m(3m 2) d y(4y + 5) e m(4 3m) f k(2 + 5k)
g t(4 3t) h x(2 + 7x)
8 a 3x + 4 b 2 + 3m c 3 2t d 4x 1
9 a n, n + 1, n + 2 b 3n + 3 c 3(n + 1)
d Sum of 3 integers = 3n + 3 = 3(n + 1). (n + 1) is an integer, hence 3(n + 1) is
always a multiple of 3.

Extension Answers

a 2 and x2 + 2x, x and 2x + 4, 2x and x + 2


b For x = 1, all pairs of values multiply to give an area of 6. For x = 2, all pairs of
values multiply to give an area of 16. For x = 3, all pairs of values multiply to give
an area of 30.
c 2 and 6x2 + 9x, 3 and 4x2 + 6x, 6 and 2x2 + 3x, x and 12x + 18,
2x and 6x + 9, 3x and 4x + 6, 6x and 2x + 3

Plenary Key Words


Ask the class what is meant by the term factorisation. You want responses factorisation
which show their understanding of how to break down an expression into two
terms which will multiply together to give the original expression.
Show them again the two stages which they have gone through today with some
examples such as:
6 + 9x = 3(2 + 3x)
5x2 3x = x(5x 3)
Homework

1 Factorise each of the following.


a 3x + 9 b 4t + 12 c 2m + 8 d 5y + 15
e 10 + 2m f 4 + 6k g 10 + 15t h 12 + 9x
i 6x 4 j 8t 12 k 6m 9 l 20y 8
m 21 7m n 18 3k p 12 10t q 15 5x
2 Factorise each of the following.
a x2 + 5x b t2 + 3t c m2 + 4m d y2 + 8y
e 6m + m2 f 2k + k2 g 7t + t2 h x + x2
i x2 4x j 2t2 3t k m2 5m l 5y2 4y
m 3m m2 n 6k 5k2 p 6t t2 q 8x 5x2

Answers
1 a 3( x + 3) b 4(t + 3) c 2(m + 4) d 5( y + 3) e 2(5 + m) f 2(2 + 3k) g 5(2 + 3t) h 3(4 + 3x)
i 2(3x 2) j 4(2t 3) k 3(2m 3) l 4(5y 2) m 7(3 m) n 3(6 k) p 2(6 5t) q 5(3 x)
2 a x( x + 5) b t( t + 3) c m(m + 4) d y( y + 8) e m(6 + m) f k(2 + k) g t(7 + t) h x(1 + x)
i x( x 4) j t(2t 3) k m(m 5) l y(5y 4) m m(3 m) n k(6 5k) p t(6 t) q x(8 5x)

HarperCollinsPublishers Ltd 2003 121


9B3LP_11.qxd 18/09/2003 15:21 Page 122

LESSON Framework objectives Quadratic expansion


Square a linear expression, expand the product of two linear expressions of the
11.3 form (x n) and simplify the corresponding quadratic expression.

Oral and mental starter


This starter is concerned with calculating percentages.
Set percentages in context by talking about getting a 20% reduction on the price
of a certain item you purchased recently.
Ask the class how they would calculate 20% of a value? They might use the
strategy of finding the value of 10% then doubling it, or they might divide the
original value by 5.
Give them an example, such as: What is 20% of 34. Work through the
example, using the first method which most students will regard more
straightforward. That is, 10% of 34 is 3.40, doubling it gives 20% as 6.80.
Using a target board such as the one shown below, work your way around the
class, asking individual students to work out 20% of the given value.
45 35 kg 50 minutes 82 29 kg
65 minutes 67 18 m 2 hours 39
130 kg 180 minutes 89 75 kg 49 m
234 83 m 24 hours 26 130 kg

Main lesson activity


Put on the board a rectangle with sides labelled as (x + 2) and (x + 4). Ask the
class if they know what the area of the rectangle is.
This should lead to a discussion on splitting the rectangle up into 4 parts, with
areas of x2, 4x, 2x and 8. Add these up to get x2 + 6x + 8, which is the area of the
original rectangle.
Next, put on the board the pair of brackets (x + 5)(x + 3). Ask the class if anyone
can expand, or multiply out this pair of brackets. Let the discussion flow from
the geometric application of finding the area of a rectangle to a possible way of
multiplying out the brackets term by term.
Show both methods. Tell the students they must use whichever method they find
simpler for them. Ultimately, they will be expected to multiply out brackets with
no props at all.
Then, put on the board a pair of brackets with a negative sign involved: for
example (x + 6)(x 2). Show the class that the easiest way to do this is to
multiply term by term, to get x2 2x + 6x 12, which simplifies to x2 + 4x 12.
Finally, put on the board (x 4)2. Ask if anyone can square this bracket. Look out
for the response of x2 + 16, which is the most common incorrect answer.
Demonstrate this expansion, which is (x 4)(x 4), giving x2 8x + 16.
The class can now do Exercise 11C from Pupil Book 3.

122 HarperCollinsPublishers Ltd 2003


9B3LP_11.qxd 18/09/2003 15:21 Page 123

Exercise 11C Answers

1 x2 + 7x + 12 2 x2 + 6x + 5 3 x2 + 9x + 14 4 x2 2x 8 5 x2 + x 12
6 x2 4x 5 7 x2 x 6 8 x2 + 5x 6 9 x2 x 12 10 x2 3x + 2
11 x2 9x + 18 12 x2 9x + 20 13 4 3x x2 14 10 3x x2 15 18 + 3x x2
16 x2 + 10x + 25 17 x2 6x + 9 18 4 4x + x2
19 f (x + y)(x y) = x2 xy + xy y2 = x2 y2
20 a 800 b 400 c 280 d 75 e 400 f 35 g 62 h 58 i 1997

Extension Answers

1 8x2 + 14x + 3 2 12x2 + 23x + 10 3 8x2 + 2x 3 4 10x2 + 26x 12


5 8x2 22x + 12 6 25x2 30x + 9

Plenary Key Words


Put on the board (x + a)(x + b) and discuss with the class what its expansion will
expand
give: x2 + ax + bx + ab.
Next, put on the board (x + a)2 and discuss with the class what its expansion will quadratic
give: x2 + 2ax + a2.
If you feel that the class is ready for the next step, then write on the board
(ax + b)(cx + d ). Discuss with the class what its expansion will yield:
acx2 + adx + bcx + bd.
Homework

1 Expand and simplify each of the following expressions.


a (x + 4)(x + 7) b (x + 3)(x 6) c (x 5)(x + 7)
d (x + 3)(x 5) e (x 4)(x 3) f (x 8)(x + 5)
g (x + 3)2 h (x 5)2 i (x + 4)(x 4)
2 Without using a calculator, find the result of each of the following calculations.
a 752 252 b 9.72 0.32 c 18.72 1.32

Answers
1 a x2 + 11x + 28 b x2 3x 18 c x2 + 2x 35 d x2 2x 15 e x2 7x + 12 f x2 3x 40
g x2 + 6x + 9 h x2 10x + 25 i x2 16
2 a 5000 b 94 c 348

HarperCollinsPublishers Ltd 2003 123


9B3LP_11.qxd 18/09/2003 15:21 Page 124

LESSON Framework objectives Quadratic factorisation


Simplify or transform algebraic expressions by taking out common factors.
11.4
Oral and mental starter
Ask the class: Who sends text messages to their friends?. Ask several of the
students who do, how much they cost.
Put on the board the different rates for text messages. At the time of printing,
these are generally 8p, 10p or 12p. But do use those which your students offer,
unless they only offer 10p!
Ask how many text messages each student sends per day. Using the price per
message that the student quoted earlier, work out how much students spend on
text messaging per day.
Ask other students how many messages they send per week. Again, using the
price per message, work out how much various students spend on text
messaging per week.
Talk about top up cards costing 10, 15 or 20. How many text messages can
the students send from one top up at the different values?
This is a rich source of mental work that goes best when real live data is being
used: that is, current charges. Try not to be exclusive. If there are some students
not able to text, ensure that they are included in the discussions.
This can be expanded into estimating how many text messages each student
might make in a year and therefore how much this will cost.

Main lesson activity


Put on the board the expression x2 + 8x + 12. Ask the class if they can simplify
this expression. Lead the discussion to factorising with a pair of brackets.
The x2 indicates that each bracket must start with two single xs, and the pair of
brackets will be in the form (x + a)(x + b).
Talk about the last plus sign in the expression, which indicates that the signs in
the brackets will be the same, and the same as the first sign in the expression.
Hence, the two signs in the pair of brackets will be pluses.
Ask what information the 12 gives. Help the class to see that this represents the
product of a and b. Hence, a factor pair of 12 is required: 1, 12 or 2, 6 or 3, 4.
Ask what information the 8x gives. Help the class to see that this shows that
a + b must be 8. So, a factor pair which adds up to 8 is required. This leads to
the conclusion that a and b are 2 and 6 respectively. (They can be either way
round.) Hence, the factorisation is: (x + 2)(x + 6).
Go briefly through the expansion to show that this does indeed give
x2 + 8x + 12.
Now put on the board the expression x2 8x + 16 and ask the class to factorise it.
Discuss the fact that because the second sign of the expression is a plus, the
signs will be the same, and the same as the first sign in the expression, which is
a minus. Hence, the brackets will be of the form (x a)(x b).
This means that a factor pair of 16 is required which adds up to 8, leading to
(x 4)(x 4) or (x 4)2.
Next, put on the board the expression x2 3x 10, and lead the class into a
discussion on the factorisation of this expression.
As the second sign of the expression is a minus, the two signs in the brackets
must be different. The first sign of the expression is negative, which indicates
that the larger of the factor pair must also be negative.
This means that as the signs are different, the factor pair of 10 must have a
difference of 3, which leads to 5 and 2. Hence, the factorisation is (x + 2)(x 5).
The class can now do Exercise 11D from Pupil Book 3.

124 HarperCollinsPublishers Ltd 2003


9B3LP_11.qxd 18/09/2003 15:21 Page 125

Exercise 11D Answers

1 (x + 4)(x + 6) 2 (x + 2)(x + 12) 3 (x + 3)(x + 6) 4 (x 2)(x 9)


5 (x 3)(x 4) 6 (x 2)(x 6) 7 (x + 6)(x 4) 8 (x + 11)(x 4) 9 (x 2)(x + 6)
10 (x + 4)(x 11) 11 (x 9)(x + 7) 12 (x 10)(x + 9) 13 (x + 5)2 14 (x 6)2
15 (x 1)2 16 (x + 2)(x 2) 17 (x + 5)(x 5) 18 (x + 10)(x 10)

Extension Answers

1 (3x + 1)(x + 1) 2 (3x + 1)(x 2) 3 (3x + 2)2 4 (2x 1)(x 5) 5 (4x + 5)(x 5)
6 (3x + 4)(2x 5)

Plenary Key Words


Put on the board x2 + Ax + B.

factorisation
Ask what clues should be looked for in the factorisation of these expressions.
Discuss the signs. When the last sign in the expression is a plus, both bracket
signs will be the same as the first sign in the expression. When the last sign in
the expression is a minus, the bracket signs will be different.
Discuss the factor pair needed for B. When the last sign in the expression is a
plus, the factor pair must add up to A. When the last sign in the expression is a
minus, the factor pair must have a difference of A.
Homework

Factorise each of the following.


1 x2 + 6x + 8 2 x2 9x + 20 3 x2 + 3x 4
4 x2 4x 12 5 x2 + 4x + 4 6 x2 14x + 49
7 x2 16 8 x2 1 9 x2 4x 21

Answers
1 (x + 4)(x + 2) 2 (x 5)(x 4) 3 (x + 4)(x 1) 4 (x + 2)(x 6) 5 (x + 2)2 6 (x 7)2 7 (x + 4)(x 4)
8 (x + 1)(x 1) 9 (x 7)(x + 3)

HarperCollinsPublishers Ltd 2003 125


9B3LP_11.qxd 18/09/2003 15:21 Page 126

LESSON Framework objectives Change of subject


Derive and use more complex formulae and change the subject of a formula.
11.5
Oral and mental starter
Ask the students to put up their hands if they can multiply by 11. Many hands will go up.
Tell them to keep their hands up if they can work out 11 28 in their heads. Discuss the different strategies the
students have used to get their various answers.
One strategy is to do it in two parts and add the results together:
11 28 = (10 28) + (1 28) = 280 + 28 = 308.
Go through another example using this method with them:
11 34 = (10 34) + (1 34) = 340 + 34 = 374.
Using a target board such as the one shown below, work your way around the class asking individual students
to multiply the number at which you point by 11.
17 25 51 67 92
42 78 19 34 83
94 85 36 48 23

Main lesson activity


This lesson is about changing the subject of a formula. Put on the board the formula C = 250 + 5W. Explain that
this formula is used to calculate the cost of advertisements in a certain newspaper, where C is the cost in pence
of the advertisement and W is the number of words in the advertisement. Explain that C is the subject of the
formula because it is the variable (letter) in the formula which stands on its own, usually on the left-hand side
of the equals sign.
Ask how much it would cost to place an advertisement with 20 words in the newspaper. Use this example to
verify that all the students can substitute W = 20 into the formula to get C = 250 + 5 20 = 250 + 100 = 350,
giving the cost as 3.50.
Now tell the class that you want to place an advertisement in the newspaper, on which you want to spend 10.
Ask them how many words you would use in the advertisement.
In order to work this out, they need to rearrange the formula so that W is the subject. Remind the class that the
same rules apply which they have used previously with equations. That is, the same thing is done to both sides
of the formula.
Work through the example. To get W on its own, start by simplifying the right-hand side of the formula:
C = 250 + 5W
Subtracting 250 from both sides gives: C 250 = 5W
C 250
Dividing both sides by 5 gives: = W
5
C 250
Turning the expression round gives: W =
5
1000 250
So, if C = 1000, then: W = = 150.
5
The class can now do Exercise 11E from Pupil Book 3.

SATs Answers

1 a i 100 cm2, 80 cm2, 32 cm2 ii 252 cm2 iii 252 b i n2 cm2, 2n cm2, 6 cm2 ii n2 + 5n + 6
2 a 5(2y + 4) and 2(5y + 10) b 12( y + 24) c 7( y + 2) d 2y2(3y 1)
3d
3 a a = 1500; b = 200 b
5
4 a ( y + 3)2 = ( y + 3)( y + 3) = y2 + 6y + 9 y2 + 9 b i y2 + 7y + 10 ii y2 12y + 36 iii 6y2 y 40

126 HarperCollinsPublishers Ltd 2003


9B3LP_11.qxd 18/09/2003 15:21 Page 127

Exercise 11E Answers

V V (S U) (S U)
1 a i I = ii R = b i U = S FT ii F = iii T =
R I T F
(P 2w) (P 2b) 2A 2A
c i b = ii w = d i b = ii h =
2 2 h b
5(F 32)
2 a C = b i 53.9 C ii 19.2 C iii 13.6 C
9
3 a 24.5 cm2 b 11.8 cm2 4 a 1.60 b 0.75 cm
5 a 125.7 cm2 b 9.6 cm (9.5 cm if key on calculator used) c 1.5 cm
6 a N = 5, R = 3, A = 6, so N + R A = 5 + 3 6 = 2, so N + R A = 2.
c N + R A = 2, so N + R = 2 + A, so N + R 2 = A, i.e. A = N + R 2. N = 10, R = 9, so A = 17
7 a y = 2x 3 b y = 3x 5 c y = 9 2x d y = 14 3x e y = 4x + 3 f y = 5x + 1
8 a i y: 1, 1, 3, 5, 7 ii y: 4, 1, 2, 5, 8 iii y: 7, 5, 3, 1, 1
9 They are all parallel to each other ( y = x) and cut the y-axis at the negative value of the constant in the equation.
10 They are all parallel to each other ( y = x) and cut both the x-axis and the y-axis at the negative value of the constant
in the equation.

Extension Answers

They are all parallel to each other ( y = 6x) and they cut the y-axis at the negative value
of the product of the constant in the equation multiplied by the denominator of the
y coefficient.

Plenary Key Words


Discuss with the class the similarities between the process of changing the
subject of a
subject of a formula and that of solving an equation.
To close the lesson, work with the class to make T the subject of the following formula
formula:
3K + 5T 2
W =
4
Homework

1 Change the subject of each of the following formulae as indicated.


a Make I the subject of the formula W = IPT.
b i Make P the subject of F = P + MK. ii Make M the subject of F = P + MK.
c i Make m the subject of T = 3m + 2n. ii Make n the subject of T = 3m + 2n.
abh
d Make b the subject of V = .
3
19R
2 The formula C = + 40 is used to calculate the cost in pounds of making a boiler of radius
8
R (cm).
a Make R the subject of the formula.
b Use this formula to find the radius of a boiler that cost 150 to make.

3 Draw a graph of each of the following equations on the same pair of axes.
a y 2x 1 = 0 b y 2x 3 = 0 c y 2x + 1 = 0 d y 2x + 3 = 0
Comment on the similarities and differences between the graphs.

Answers
W (F P) (T 2n) (T 3m) 3V
1 a I = b i P = F MK ii M = c i m = ii n = d b =
PT K 3 2 ah
8(C 40)
2 a R = b 46.3 cm
19
3 All graphs should be parallel to each other ( y = 2x) and cut the y-axis at the negative value of the constant in
the equation.

HarperCollinsPublishers Ltd 2003 127


9B3LP_12.qxd 18/09/2003 15:21 Page 128

CHAPTER
12 Solving Problems and Revision
LESSON Framework objectives Fractions, percentages and decimals
Revision of Number:
12.1 Solve increasingly demanding problems and evaluate solutions.
Solve problems involving percentage changes.
Use proportional reasoning to solve a problem, choosing the correct numbers to
take as 100%, or as a whole.
Recognise when fractions or percentages are needed to compare proportions; solve
problems involving percentage changes.
Enter numbers and interpret the display in context (negative numbers, fractions,
decimals, percentages, money, metric measures, time).

Oral and mental starter


The following is a 10-question, SATs-style, mental test on the theme of fractions.
Repeat each question twice and allow 10 seconds to answer.
1 What fraction of one metre is thirty five centimetres? [Write 1 m and
35 cm on the board.]
2 Look at these numbers. Which one of them is the decimal equivalent of
three-eighths? [Write the decimals on the board.]
0.38 3.8 0.375 3.125 3.08
3 What is half of 4.7?
4 What is the sum of three-eighths and one-quarter?
5 Add one point six to one quarter.
3 2
( )
6 What is three-quarters squared? [Write on the board.]
4
7 What is a quarter of 6.2?
8 What decimal is equivalent to the fraction five-eights?
9 Four-ninths of a number is thirty six. What is the number?
10 What is the square root of 16
?
49
35 7 2 0.375 3 2.35 4 58 or equivalent (0.625)
Answers 1 100 or equivalent 20
5 1.85 6 16 7 1.55 8 0.625 9 81 10 47
9

Main lesson activity


This is a revision lesson on Number, principally covering fractions, decimals and
percentages.
The questions in the Pupil Book exercise are graded from Level 5 to Level 8 as
follows:
Q. 1, 6, 7 Level 5
Q. 2, 8, 9 Level 6
Q. 3, 4, 10 Level 7
Q. 5, 11, 12, 13 Level 8
Before letting students start the questions you can go through key points (as
suggested below) or discuss some specific questions with the class to remind
them of the methods used.
General
Equivalence of fractions, percentages and decimals

Fractions
Equivalent fractions
Cancelling
Converting mixed numbers to top heavy fractions and vice versa
Adding and subtracting fractions

128 HarperCollinsPublishers Ltd 2003


9B3LP_12.qxd 18/09/2003 15:21 Page 129

Decimals
Ordering
Changing between metric measurement units

Percentages
Finding a percentage of a quantity
Finding one quantity as a percentage of another
Calculating percentage increase and decrease
Compound interest
Reverse percentage

The class can now do Exercise 12A from Pupil Book 3.

Exercise 12A Answers

14 7 3
1 a
15
b 18 c 4
20
2 134.4
3 a i 0.08 ii 2000 b 400450
4 a 45 0.85 0.9 b 1.13
5 a Always even b Always an integer
6 74.5 kg
7 63.05
8 36.21
9 a 200 000 b 425 000 c 3.9%
10 a 31.4% b 36% c Unleaded 76.2p per litre. Lead replacement 77p per litre
11 a 800 b 12.2%
12 a 836 b 499 c 295.53 d 6 days
13 a 16.521 174 86 b 16.5

Plenary
Go through the answers to the exercise. Discuss and clarify those with which
students had difficulty.

Homework
It is assumed that during the revision period students will be given a past SATs
paper to work through at home. Students will have seen some questions before in
the Maths Frameworking Pupil Books, so a mock SATs paper is provided in this
Teachers Pack, after the Chapter 12 lesson plans. The mock paper consists of SATs-
style questions which students will not have encountered before.
Additional homework questions are provided below, for further practice on the
topics covered in this lesson.
Homework

1 In a sale, a hi-fi is reduced by 15%. The sale price of the hi-fi is 459, what was the original price?
2 For each part of the question, where n is always an integer, write down the answer which is true and
explain your choice.
n2 2
a When n is even, is:
2
Always odd Always even Sometimes odd, sometimes even
n2 2
b When n is odd, is:
2
Always an integer Always a fraction Sometimes an integer, sometimes a fraction

Answers
1 540 2 a Always odd b Always a fraction

HarperCollinsPublishers Ltd 2003 129


9B3LP_12.qxd 18/09/2003 15:21 Page 130

LESSON Framework objectives The four rules; ratio; standard form


Revision of Number:
12.2 Solve increasingly demanding problems and evaluate solutions.
Interpret and use ratio in a range of contexts, including solving word problems.
Enter numbers and interpret the display in context (negative numbers, fractions,
decimals, percentages, money, metric measures, time).

Oral and mental starter


The following is a 10-question, SATs-style, mental test on the theme of
percentages. Repeat each question twice and allow 10 seconds to answer.
1 What is ten percent of thirty five pounds?
2 What is twenty percent of two hundred pounds?
3 Thirty percent of a number is nine. What is the number?
4 A CD costing thirteen pounds is reduced in a sale by ten percent. What is the
new price of the CD?
5 There are 20 chocolates in a box, of which 12 have soft centres. What
percentage of the chocolates have soft centres?
6 Fifteen percent of a number is twelve. What is the number?
7 After a 10% reduction, a cooker was priced at 180. What was the original
price?
8 What percentage is equivalent to the fraction seven-eighths?
9 Six percent of a number is nine. What is the number?
10 What percentage of forty five is twenty seven?
Answers 1 3.50 2 40 3 30 4 11.70 5 60% 6 80 7 200
8 87.5% 9 150 10 60%

Main lesson activity


This is a revision lesson on Number, principally covering the four rules, ratios
and standard form.
The questions in the Pupil Book exercise are graded from Level 5 to Level 8 as
follows:
Q. 1, 7, 8 Level 5
Q. 2, 3, 9, 10 Level 6
Q. 4, 5, 11 Level 7
Q. 6, 12, 13 Level 8
Before letting students start the questions you can go through key points (as
suggested below) or discuss some specific questions with the class to remind
them of the methods used.
General
Basic knowledge of tables up to 10 10

Four rules
Setting out in columns for addition and subtraction
Using box method or column methods for long multiplication
Using chunking for long division

Directed numbers
Using a number line
Combining signs when adding and subtracting: ++, +, etc

Ratio
Adding ratios
Dividing up an amount into a given ratio
Multiplying by a ratio to get individual amounts
Standard form
Writing numbers in standard form
Calculating with numbers in standard form

The class can now do Exercise 12B from Pupil Book 3.


130 HarperCollinsPublishers Ltd 2003
9B3LP_12.qxd 18/09/2003 15:21 Page 131

Exercise 12B Answers

1 17 bins with 7 left over


2 18
3 a 14.4 b 90 c 0.06
4 a = 4, b = 8, c = 3
5 a 59 b 26 and 82 c 44 d (1)2, (2)6, 82 and 59
6 a iii 32 103 is larger than 33 102 b 3.6 103 and 36 102 c 54 103
7 a 15 b 56 175 c 80.25
8 a 52 mph b 4 hours and 40 minutes
9 a 400 kg b 8 bags
10 347 tickets
11 1:4
12 a 1 106 b 5 104
13 a 3.32 107 b 8.46 108 c 1.93 107

Plenary
Go through the answers to the exercise. Discuss and clarify those with which
students had difficulty.
Homework

1 p = 1.2 107, q = 2.5 108, r = 6.3 103


Work out each of the following, giving your answer in standard form.
a p+q b pq c r2
2 Light green paint is made by mixing yellow paint and blue paint in the ratio 2 : 3.
Dark green paint is made by mixing yellow paint and blue paint in the ratio 1 : 3.
One litre of light green paint and one litre of dark green paint are poured into a large bucket.
How much more yellow paint needs to be added to the bucket to produce light green paint?

Answers
1 a 2.62 108 b 3 1015 c 3.969 105
2 250 ml

HarperCollinsPublishers Ltd 2003 131


9B3LP_12.qxd 18/09/2003 15:21 Page 132

LESSON Framework objectives Rules of algebra and linear equations


Revision of Algebra:
12.3 Represent problems and synthesise information in algebraic form.

Oral and mental starter


The following is a 20-question, SATs-style, mental test on the theme of the four rules. Repeat each question
twice and allow 10 seconds to answer.
1 Four boxes of pencils cost six pounds. How much will seven boxes of pencils cost?
2 Multiply nought point seven by ten.
3 A chocolate bar costs one pound forty pence. I buy four bars. How much change will I get from a ten
pound note?
4 I am thinking of two numbers. When I add them together I get nine. When I multiply them together I get
twenty. What are the numbers ?
5 Multiply nought point six by nought point five.
6 Double sixty six.
7 What is the total cost of five video tapes at four pounds ninety five pence each?
8 How many seconds are there in fifteen minutes?
9 Work out the value of this. [Write 23 32 on the board.]
10 What is three minus nought point two?
11 How much must be added to this number to make one hundred. [Write 63.5 on the board.]
12 Multiply together the first three prime numbers.
13 I have saved thirty seven pounds in twenty pence coins. How many coins is that?
14 Divide thirty by nought point three.
15 What number is nought point nought one less than five point three? [Write 0.01 and 5.3 on the board.]
16 Multiply together nought point nought four and nought point two. [Write 0.04 and 0.2 on the board.]
17 Divide forty point two by nought point one. [Write 40.2 and 0.1 on the board.]

18 What is the value of this. [Write 225 23 on the board.]
19 Work out nought point nought three squared. [Write 0.03 on the board.]
20 Divide eight by fifty [Write 8 and 50 on the board.]
Answers 1 10.50 2 7 3 4.40 4 4 and 5 5 0.3 6 132 7 24.75 8 900 9 72
10 2.8 11 36.5 12 30 13 185 14 100 15 5.29 16 0.008 17 402 18 120
19 0.0009 20 0.16

Main lesson activity


This is a revision lesson on Algebra, principally covering the basic rules and solving linear equations.
The questions in the Pupil Book exercise are graded from Level 5 to Level 8 as follows:
Q. 1, 2, 3 Level 5
Q. 4, 5, 6, 7 Level 6
Q. 8, 9, 10 Level 7
Q. 11, 12, 13 Level 8
Before letting students start the questions you can go through key points (as suggested below) or discuss some
specific questions with the class to remind them of the methods used.
Basic algebra
Using letters to represent variables
The difference between a term, an expression and an equation

Manipulative algebra
Substituting numbers into expressions
Expanding brackets
Factorising
Collecting like terms
Cancelling

132 HarperCollinsPublishers Ltd 2003


9B3LP_12.qxd 18/09/2003 15:21 Page 133

Linear equations
Rearranging collecting together variables and numbers on the LHS and RHS
respectively
Inverse operations (change sides, change signs)
Checking answers by substituting into original equation

The class can now do Exercise 12C from Pupil Book 3.

Exercise 12C Answers

1 a n2, 4n, 16 b n2 + 8n + 16, (n + 4)2


2 a 4x 20 b 11x + 3 c 5x + 2 d 17x + 16 e 5x + 22
3 a i 21 ii 10 iii 50 b i z = 3 ii z = 22 iii z = 1
4 a 6x + 3 = 12, x = 1.5 b 3y 6 = y + 7, y = 6.5
5 a 2n + 4 b n + 6 c 2 n d 3n 1
6 a i 45 ii 26 iii 11 b i 3(x + 2y) ii x(x + 1) iii 2a(2b + 3)
7 a x = 1.5 b x = 2 c x = 0.5
8 a 3(4x 6) and 6(2x 3) b 6( y 2) c 3y(3y 2)
2n 4 9 16
9 a b , , c 1
2n + 3 5 10 17
2
10 a i 3 ii 4 b xy2 c i x 24 ii 9y + 2x
3
11 a 7 b 2.5 c 0 or 4
12 a x2 8x + 16 b x2 + x 20 c 12x2 5x 2
13 2

Plenary
Go through the answers to the exercise. Discuss and clarify those with which
students had difficulty.
Homework

1 a Explain why (x 4)(x 4) x2 16


b Expand and simplify each of the following.
i 2(x 3) + 3(2x 1) ii (x + 4)(x 7)
2 Solve each of the following equations.
2x + 3
a 6 + 2x = 8 + 4x b = 5 c 5(1 + x) = 3(x + 2)
3

Answers
1 a (x 4)(x 4) = x2 4x 4x + 16 = x2 8x + 16
b i 8x 9 ii x2 3x 28
2 a 1 b 6 c 0.5

HarperCollinsPublishers Ltd 2003 133


9B3LP_12.qxd 18/09/2003 15:21 Page 134

LESSON Framework objectives Graphs


Revision of Algebra:
12.4 Represent problems and synthesise information in graphical form.

Oral and mental starter


The following is a 10-question, SATs-style, mental test on reading diagrams.
Each student will need a sheet with the diagrams below to refer to. Teachers may
find the Teachers Pack CD useful to prepare these.
Repeat each question twice and allow 10 seconds to answer.
1 The bar chart shows the number of children in some families. How many
families are represented?
2 This is a centimetre grid. What is the area of the shaded square?
3 Which diagram shows strong positive correlation?
4 By looking at the timetable, work out how long the journey from Barnsley to
High Green takes.
5 What number is the arrow pointing to?
6 Add one more square to the grid so that it has rotational symmetry of order 2.
7 Which diagram shows the graph x + y = 5?
8 What is the exterior angle of a regular hexagon?
9 The pie chart shows the proportion of men and women in a
sports club. Work out the angle which represents the women.
10 Look at the octagon. Which diagram shows the octagon after
a ninety degree rotation in a clockwise direction?
Answers 1 21 2 17 cm2 3 c 4 44 minutes 5 1.4 6 7 c

8 60 9 72 10

1 3
Number of families

6 a b c d
4

0
1 2 3 4 5
Number of children 4 5
Barnsley 09 32
Birdwell 09 55
2 High Green 10 16
Sheffield 10 36 2 1 0

6 7 a b
5

5
c 5 d5

8 9 10
Women (20%)

Men (80%)

134 HarperCollinsPublishers Ltd 2003


9B3LP_12.qxd 18/09/2003 15:21 Page 135

Main lesson activity


This is a revision lesson on Algebra, principally covering graphs.
The questions in the Pupil Book exercise are graded from Level 5 to Level 8 as follows:
Q. 1, 2 Level 5
Q. 3, 4, 5, 6 Level 6
Q. 7, 8, 9 Level 7
Q. 10, 11 Level 8
Before letting students start the questions you can go through key points (as suggested below) or discuss some
specific questions with the class to remind them of the methods used.
Basic graphs
y = mx + c as the formula of a straight line
Significance of m and c for gradients, parallel lines and intercept with yaxis
Quadratic graphs

Real life graphs


Travel graphs or distancetime graphs
Gradient of line represents speed
Horizontal line represents no motion

The class can now do Exercise 12D from Pupil Book 3.

Exercise 12D Answers

1 a 7:15 b 7:45 c 8 : 40 d 9:30 5 a y = 2 b y = 2x c x = 0.5 and y = 1


2 a (6, 0) and (1, 7) 6 a 6 b 13.5 square units
b 6 4
4 2
2
0
0 6 4 2 0 2 4 6
6 4 2 0 2 4 6 2
2 y = 3
4
4
6
y = 3x x=2
6
3 a true b cannot be sure, graph is levelling off 7 a A and E b C and D c A and F d D and F
c cannot be sure there may not be a causal link 8 a 9:30 b 160 kph
4 a b 6 c (3, 5) 9 a Test (8, 0) and (0, 4) in the equation b x + y = 6
10 Draw line at y = 2, x = 2.5
4
11 a y x, y 1, x 2 b R
2 4
0
3
6 4 2 0 2 4 6
2
2
1
y=x+2 4
0
6 0 1 2 3 4
y = 2x + 2 y = 2x 1

Plenary
Go through the answers to the exercise. Discuss and clarify those with which
students had difficulty.
Homework

1 By drawing the graphs y = 2x, y = 2 and x = 3, work out y


the area of the triangle enclosed by all three lines. 4
2 Give the four inequalities which describe the shaded region. 3
2
1
0 x
0 1 2 3 4

Answers
1 16 square units
2 y 4, y x, x 1, x 3

HarperCollinsPublishers Ltd 2003 135


9B3LP_12.qxd 18/09/2003 15:21 Page 136

LESSON Framework objectives Shape, Space and Measures


Revision of Shape, Space and Measures:
12.5 Represent problems and synthesise information in geometric form.
Solve problems using properties of angles, of parallel and intersecting lines, and of
triangles and other polygons, justifying inferences and explaining reasoning with
diagrams and text.

Oral and mental starter


The following is a 10-question, SATs-style, mental test on the theme of shape,
space and measures. Repeat each question twice and allow 10 seconds to
answer.
1 How many lines of symmetry does a parallelogram have?
2 I face south-east and turn anticlockwise through 270 degrees. What direction 130
am I now facing?
3 I drive six and a half kilometres in 10 minutes. What is my average speed in a
kilometres per hour?
4 What is the value of angle a? [Draw the diagram shown on the 5 cm 4 cm
board.]
5 What is the area of this triangle? [Draw the triangle shown on the board.] 6 cm
6 Draw the shape that you get after rotating this T shape by 90 clockwise.
[Draw the shape shown on the board.]
7 What is the approximate area of a circle with a radius of 2 centimetres?
8 The distance from home to school is about 12 kilometres. Approximately
how many miles is that?
5 cm
9 What is the area of this triangle? [Draw the triangle shown on the board.] 3 cm
10 This picture shows a shape cut in half along a line of symmetry.
[Draw the picture shown on the board.] What is the name of the
original shape?
Answers 1 0 2 south-west 3 39 km/h 4 50 5 12 cm2
6 7 1214 cm2 8 78 miles 9 6 cm2 10 Kite

Main lesson activity


This is a revision lesson on Shape, Space and Measures.
The questions in the Pupil Book exercise are graded from Level 5 to Level 8 as
follows:
Q. 1, 5, 6 Level 5
Q. 7, 8, 9, 10 Level 6
Q. 2, 3, 11 Level 7
Q. 4, 12, 13 Level 8
Before letting students start the questions you can go through key points (as
suggested below) or discuss some specific questions with the class to remind
them of the methods used.
Volume and area
Recall of formulae for area of rectangle, triangle, parallelogram, circle
Recall of formulae for volume of cubes, cuboids and prisms

Symmetry
Line symmetry
Rotational symmetry

Angles
Definition of acute, obtuse and reflex
Measuring angles
Angles at a point and on a straight line
Alternate and corresponding angles

136 HarperCollinsPublishers Ltd 2003


9B3LP_12.qxd 18/09/2003 15:21 Page 137

Enlargements
Scale factor and centre of enlargement
Similar triangles

Right-angled triangles
Pythagoras
Trigonometry

The class can now do Exercise 12E from Pupil Book 3.

Exercise 12E Answers

1
a = 54, b = 82, c = 152
27.5 cm2
2
a 300 cm3 b 25 cm2
3
4
x = 18 cm, y = 10 cm
a 288 cm2 b 4 c 16 : 1
5
a 80 km b no, 100 kph 62 mph
6
c 93 miles, 50 km is about 31 and 3 31 = 93
7 a check sides measure 5 cm, 8 cm and 7 cm b 82
8 a a = 39, b = 39, c = 43 b angle ADB = angle DBE so AD is parallel to BE
9 40 cm
10

11 a 6.32 cm b 5.66 cm
12 a Area = 33.5 cm2, Perimeter = 24.4 cm b 4.09 cm
13 x = 16.6 cm y = 36.7

Plenary
Go through the answers to the exercise. Discuss and clarify those with which
students had difficulty.
Homework

1 Find the length x in the triangle shown on the right.


20 cm
13 cm

x cm

2 Find the length x and the angle y in each of the triangles shown below.
a b
57 12 cm
7 cm

y
x cm 7 cm

Answers
1 15.2 cm 2 a 10.8 cm b 54.3

HarperCollinsPublishers Ltd 2003 137


9B3LP_12.qxd 18/09/2003 15:21 Page 138

LESSON Framework objectives Handling Data


Revision of Handling Data:
12.6 Interpret graphs and diagrams and draw inferences to support or cast doubt on
initial conjectures; have a basic understanding of correlation.
Analyse data to find patterns and exceptions, look for cause and effect and try to
explain anomalies.

Oral and mental starter


The following is a 10-question, SATs-style mental test on the theme of Handling
Data. Repeat each question twice and allow 10 seconds to answer.
1 What is the mean of these numbers? [Write 10, 10 and 25 on the board.]
2 What is the range of these numbers? [Write on the board: 4, 8, 2, 9, 7,
12, 1, 3, 7, 8, 2 and 3.]
3 A dance class contains both boys and girls. The probability that a member of
the class, picked at random, is a girl is 0.7. What is the probability that a
member picked at random is a boy?
4 A bag contains only red, blue and green balls. The table shows the Red Blue Green
probability of choosing each colour, when a ball is picked from the bag at 0.35 0.2 0.45
random. [Draw the table shown.]
What is the probability that the ball picked is blue or green?
5 An ordinary, fair, six-sided dice is rolled. What is the probability that the dice
shows a score of 7?
6 Two ordinary, fair, six-sided dice are rolled. What is the probability that the
total score is 3?
7 What is the median of these numbers? [Write on the board: 7, 8, 10, 13, 15
and 20.]
8 The table shows the number of pets owned by ten students. Number of pets Frequency
How many pets are owned altogether? 1 2
9 The mean mass of thirty text books is nought point seven kilograms. 2 5
[Write 0.7 on the board.] What is the total mass of the textbooks? 3 3
10 A referendum returned one hundred yes votes and five hundred no
votes. Which pie chart best represents these data?
a b c d

2 1
Answers 1 15 2 11 3 0.3 4 0.65 5 0 6
36 = 18
7 11.5 8 21
9 21 kg 10 c

Main lesson activity


This is a revision lesson on Handling Data.
The questions in the Pupil Book exercise are graded from Level 5 to Level 8 as
follows:
Q. 1, 2, 3 Level 5
Q. 4, 5, 8, 9 Level 6
Q. 6, 10 Level 7
Q. 7, 11 Level 8
Before letting students start the questions you can go through key points (as
suggested below) or discuss some specific questions with the class to remind
them of the methods used.
Probability
Language and definition of probability
Writing probabilities as fractions, decimals or percentages
Sample space diagrams

138 HarperCollinsPublishers Ltd 2003


9B3LP_12.qxd 18/09/2003 15:21 Page 139

Averages
The three averages used for discrete data
Range of a set of data
Mean of a table of discrete data
Mean of a table of grouped data

Surveys
Methods of sampling
Unbiased questions with unambiguous response boxes

Scatter diagrams
Types of correlation
Using line of best fit for predicting values

The class can now do Exercise 12F from Pupil Book 3.

Exercise 12F Answers

1 Any combination where the number of red to blue is in the ratio 2:1, e.g. 20 red 10 blue
2 a Q b R c P and R
3 a 6 b 5 c i False, there is no mode to start with ii False, both old and new marks were above median
iii True, total will be 2 more
4 0.3
5 a Not representative b Overlap of responses
6 a 3x b 3x + 1
7 a 56 b 39 to 71 = 32 c 33%
8 The percentage value of a car decreases as the mileage increases or there is a negative correlation
b 50% c 28 000 miles
9 a 30 b 4.9 10 3.75 min or 3 min 45 s 11 a 0.36 b 0.48 c 80

Plenary
Go through the answers to the exercise. Discuss and clarify those with which
students had difficulty.
Homework

1
1 When two dice are rolled the probability of a double one is .
36
a When two dice are rolled what is the probability of a double 2?
b Which answer shows the probability of a treble six when three dice are rolled.
1 1 3 1

18 216 216 42
2 The bar chart shows the distances that 50 students threw a discus.

15 14
Number of pupils

10 11
9
10
6
5

0
0 10 20 30 40 50
Distance (m)

a What is the probability that a pupil chosen at random will have thrown the discus more than
30 metres?
b What is the probability that a pupil chosen at random will have thrown the discus more than
45 metres?
c Work out the mean length of throw for the 50 pupils.

Answers
1 b 216
1 a 36 1

2 a 0.4 b 0.09 c 26.4 m

HarperCollinsPublishers Ltd 2003 139


9B3LP_12SATs.qxd 18/09/2003 15:21 Page 140

PRACTICE PAPER THIS IS A NON-CALCULATOR PAPER

Level 5

1 A 3 3 3 cube is made from 27 different coloured small cubes.

The small cubes with 3 faces showing are coloured grey.

The small cubes with 2 faces showing are coloured white.

The small cubes with 1 face showing are dotted.

The small cubes with 0 faces showing are striped.

(a) Complete the table below to show the number of small cubes of each colour that
are used.

Grey cubes ..........................

White cubes ..........................

Dotted cubes ..........................

Striped cubes ..........................


Total: 27
3 marks

(b) A 4 4 4 cube is made in the same way. Complete the table below to show the
number of small cubes of each type it has and the total number of cubes used.

Grey cubes ..........................

White cubes ..........................

Dotted cubes ..........................

Striped cubes 8

Total: ..........................
2 marks

140 HarperCollinsPublishers Ltd 2003


9B3LP_12SATs.qxd 18/09/2003 15:21 Page 141

2 This is how Helen works out 25% of 240 in her head.

10% of 240 is 24
20% of 240 is 48
5% of 240 is 12
so 25% of 240 is 48 + 12 = 60

(a) Show how Helen can work out 25% of 180 in her head.

2 marks

(b) This is how Jim works out 25% of 240 in his head.

50% of 240 is 120


so 25% of 240 is 60

Show how Jim can work out 25% of 460 in his head.

2 marks

3 (a) A school needs 250 pencils.

The pencils come in packs of 12.

How many packs must the school order?

Show your working.

packs
2 marks

HarperCollinsPublishers Ltd 2003 141


9B3LP_12SATs.qxd 18/09/2003 15:21 Page 142

(b) Rulers cost 22p each.

How much do 250 rulers cost?

Show your working. Give your answer in pounds.


3 marks

4 (a) There are n cubes in this tower.

Linda adds another 7 cubes to the tower.

Write an expression to show the total number of cubes in


Lindas tower.

Tower N

1 mark

(b) Qayser builds another tower.

This tower is m cubes high.

Write an expression to show the total number of cubes


in Qaysers tower.

Tower M

1 mark

142 HarperCollinsPublishers Ltd 2003


9B3LP_12SATs.qxd 18/09/2003 15:21 Page 143

(c)

n m

Tower N
Tower M

The number of cubes in each tower is the same.

Which expression below is true?

Put a tick ( ) by the correct expression.

n=m2
n=m4
n=m2
n=m4

1 mark

(d) Brian builds two more towers like Tower N and Tower M. Brians towers have an
equal number of cubes in them, and their heights are related by this expression:
n = m + 6.
Work out the value of m.

m=
1 mark

HarperCollinsPublishers Ltd 2003 143


9B3LP_12SATs.qxd 18/09/2003 15:21 Page 144

5 The graphs below show the number of goals scored per match by Team A and Team B
over 50 games.

Team A Team B

20 20
Number of games

Number of games
15 15

10 10

5 5

0 0
0 1 2 3 4 5 0 1 2 3 4 5
Number of goals Number of goals

(a) How many goals did Team A score altogether in the 50 games?

Goals:
2 marks

(b) Which team scored three or more goals in 20% of their games?

Explain your answer.

2 marks

(c) Eli says that the graphs show that Team B is more successful than Team A.

Give a reason why this may not be true.

1 mark

144 HarperCollinsPublishers Ltd 2003


9B3LP_12SATs.qxd 18/09/2003 15:21 Page 145

Level 6

6 Three people A, B and C run a race.

Below are five different distancetime graphs.

Distance Distance Distance

Time Time Time


Graph 1 Graph 2 Graph 3

Distance Distance

Time Time
Graph 4 Graph 5

Fill in the gaps below to show which runner matches up with which graph.

Runner A sets off quickly, slows down and then speeds up

Graph

Runner B runs at a steady speed

Graph

Runner C sets off quickly and then slows down

Graph

3 marks

HarperCollinsPublishers Ltd 2003 145


9B3LP_12SATs.qxd 18/09/2003 15:21 Page 146

7 Use the graphs to match each line with its equation.

The first one has been done for you.

y C

A
y=x
10
LINE A
x=5

5 B
LINE B
y=5

LINE C

0 5 10
x x+y=5
D
LINE D
y = 5x

3 marks

8 Each shape in this question has an area of 20 cm2.

No diagram is drawn to scale.

(a) Calculate the length of the base of the parallelogram.

5 cm

base

area = 20 cm2 base = ................... cm

1 mark

146 HarperCollinsPublishers Ltd 2003


9B3LP_12SATs.qxd 18/09/2003 15:21 Page 147

(b) Calculate the height of the triangle.

height

10 cm

area = 20 cm2 height = ................ cm

1 mark

(c) What is the value of h in this trapezium?

area = 20 cm2 a + b = 4 cm h = ......................... cm

1 mark

A different trapezium has an area of 20 cm2 and a height of 5 cm. What is the value
of a + b in the trapezium?

area = 20 cm2 h = 5 cm a + b = .................... cm

1 mark

HarperCollinsPublishers Ltd 2003 147


9B3LP_12SATs.qxd 18/09/2003 15:21 Page 148

(d) Look at this rectangle:

3x + 1

area = 20 cm2

5x 5

Calculate the value of x and use it to find the perimeter of the rectangle.

Show your working.

perimeter = ...................... cm

2 marks

9 This is a series of patterns with black and white squares.

Pattern Number 1 Pattern Number 2 Pattern Number 3

(a) Complete this table:

pattern number number of black squares number of white squares


5
12

2 marks

148 HarperCollinsPublishers Ltd 2003


9B3LP_12SATs.qxd 18/09/2003 15:21 Page 149

(b) Complete this table by writing expressions:

pattern number expression for the expression for the


number of black squares number of white squares
n

2 marks

(c) Write an expression to show the total number of tiles in pattern number n.

Simplify your expression.

1 mark

(d) A different series of patterns is made with squares.

Pattern Number 1 Pattern Number 2 Pattern Number 3

For this series of patterns, write an expression to show the total number of tiles in
pattern number n. Show your working and simplify your expression.

2 marks

HarperCollinsPublishers Ltd 2003 149


9B3LP_12SATs.qxd 18/09/2003 15:21 Page 150

10 (a) Each of these calculations has the same answer, 75.

Fill in each gap with a number.

2.5 30

0.25 ...................
= 75

75 1

7.5 ...................

2 marks

(b) Solve these equations to find the values of a, b and c.

4a + 3

2b 25 = 75

c2 6

a = b = c =

3 marks

150 HarperCollinsPublishers Ltd 2003


9B3LP_12SATs.qxd 18/09/2003 15:21 Page 151

Level 7

11 In the scale drawing, the shaded area represents a rectangular flowerbed.

There is a path all around the flowerbed.

The shortest distance from the flowerbed to the edge of the path is always 2 m.

On the diagram, draw accurately the position of the edge of the path.

Scale: 1 cm to 1 m

2 marks

12 This is what a student wrote:

a b a+b
+ =
2 3 5

Show that the student was wrong.

2 marks

HarperCollinsPublishers Ltd 2003 151


9B3LP_12SATs.qxd 18/09/2003 15:21 Page 152

13 Passengers have been complaining to a bus company about how long they have to
wait for a bus.

An inspector records the waiting times of 100 passengers on one of the companys bus
routes on one day.

Results

40

35
Number of Passengers

30

25

20

15

10

0
0 5 10 15 20 25 30

Waiting Time (minutes)

(a) Use the graph to estimate the probability that a passenger chosen at random will
wait for 15 minutes or longer.

1 mark

(b) Use the graph to estimate the probability that a customer chosen at random will
wait for 7.5 minutes or less.

1 mark

152 HarperCollinsPublishers Ltd 2003


9B3LP_12SATs.qxd 18/09/2003 15:21 Page 153

(c) Calculate an estimate of the mean waiting time per passenger.

Show your working.

You may complete the table below to help you with the calculation.

Waiting time Mid-point of bar Number of fx


(minutes) (x) passengers ( f )
0 2.5 22
5
10
15
20
2530

minutes

2 marks

(d) The inspector wants to improve the survey.

She records the waiting times of more customers.

Give a different way the inspector could improve the survey.

1 mark

HarperCollinsPublishers Ltd 2003 153


9B3LP_12SATs.qxd 18/09/2003 15:21 Page 154

14 (a) Find the values of a and b when p = 5.

2 p3
a =
5

a=
1 mark

p2( p + 1)
b =
3p

b=
1 mark

(b) Simplify this expression as fully as possible:

15cd

3d

1 mark

(c) Multiply out and simplify these expressions:

3(x + 4) 2(3 2x)

1 mark

(x + 3)(x + 5)

1 mark

(x + 1)(x 2)

1 mark

(x 4)2

1 mark

154 HarperCollinsPublishers Ltd 2003


9B3LP_12SATs.qxd 18/09/2003 15:21 Page 155

Level 8

15 (a) The diagram shows the graph with equation y = x3.

On the same axes sketch the graph of y = x3 + 2.

y
y = x3

1 mark

(b) Curve A is a reflection in the x-axis of y = x3.

y
A y = x3

What is the equation of curve A?

1 mark

(c) Curve A can also be obtained from the graph of y = x3 using a different
transformation.

Describe this transformation.

1 mark

HarperCollinsPublishers Ltd 2003 155


9B3LP_12SATs.qxd 18/09/2003 15:21 Page 156

(d) The shaded region is bounded by the curve y = x3 and the line y = 2x.

y
y = 2x

y = x3

Circle two inequalities which together fully describe the shaded region.

y < x3 x<0 y<0 y < 2x

y > x3 x>0 y>0 y > 2x

2 marks

16 (a) Which number is the greater?

3 104 4 103

Explain your answer.

1 mark

(b) Circle the number that has the same value as 5 102.

52 52 500 0.5 103 5 102

2 marks

156 HarperCollinsPublishers Ltd 2003


9B3LP_12SATs.qxd 18/09/2003 15:21 Page 157

(c) (3 102) (2 102) can be written more simply as 6 104.

Write these values as simply as possible.

(4 105) (2 101)

1 mark

8 1010

4 102

1 mark

17 Here are 10 shapes.

(a) Complete the table.

Number of shapes with Number of shapes with Number of shapes with


straight edges only curved edges only both straight edges and
curved edges

1 mark

HarperCollinsPublishers Ltd 2003 157


9B3LP_12SATs.qxd 18/09/2003 15:21 Page 158

(b) A shape is chosen at random.

What is the probability that it does not have both straight and curved edges?

1 mark

(c) What is the probability that a shape is chosen that has straight edges if you already
know that it has curved edges?

1 mark

(d) Two of the shapes are chosen at random.

Which calculation shows the probability that both shapes have curved edges?

6 6 6 5 6 6
+
10 10 10 10 10 10

6 5 6 5 6 5
+ +
10 10 10 9 10 9

1 mark

158 HarperCollinsPublishers Ltd 2003


9B3LP_12SATs.qxd 18/09/2003 15:21 Page 159

18 This shape is made using two circles.

The radii of the circles are 3a and 2a.

3a 2a

(a) Find the area of each circle, in terms of a and , and show that the total area of the
shape is 5a2.

3 marks

(b) The shaded area, 5a2, of the shape is 20 cm3.

Write an equation in the form a = , leaving your answer in terms of .


Show your working to simplify your equation.

2 marks

HarperCollinsPublishers Ltd 2003 159


9B3LP_13.qxd 18/09/2003 15:22 Page 160

CHAPTER
13 Handling Data 3
LESSON Framework objectives Revision of statistical techniques
Discuss how data relate to a problem; identify possible sources, including primary
13.1 and secondary sources.
Find summary values that represent the raw data, and select the statistics most
appropriate to the problem.

Oral and mental starter


The class should work in small groups for this activity.
Explain to the class that the lesson will be used to revise handling data topics that they have already covered, in preparation
for an investigation next lesson. Tell the class that this activity is called The AtoZ of handling data.
Ask each group to think of as many handling data topics and words beginning with different letters of the alphabet as they
can, excluding those to do with probability. You may wish to ask for a few suggestions and write them on the board, such as
average, bias, data, graph, etc.
Ask the groups to write down an example for each word they come up with. For example: the average of 2 and 4 is 3.
Give the groups a few minutes to discuss their answers, then ask the groups to give you the topics to write on the board,
covering the meaning of each one.

Main lesson activity


Continuing from the oral and mental starter, ask the students to give you responses to the following types of questions.
1 What sort of things should you think about when 7 What is a frequency table?
planning a question and response section for a 8 Name as many different types of frequency diagrams
questionnaire? as you can.
2 What is a census? 9 What is a stemandleaf diagram?
3 Name some ways of recording data. 10 How do you work out a mode, a median, a mean and a
4 What is the difference between primary and range for a set of data?
secondary data? 11 How do you estimate the median and interquartile range
5 How can you choose a random sample from a group from a cumulative frequency graph (for large sets of
of people? data).
6 What is a two-way table? 12 How do you estimate the mean for a table of grouped data.
You should look for answers which refer to the following.
1 Avoid: leading questions; missing or overlapping responses; any form of bias.
2 A census is a survey of a whole population.
3 Tally chart, data collection sheet or observation sheet.
4 Primary data is data obtained directly by the person carrying out the research whereas secondary data has already been
collected by someone else.
5 A random sample of a group could be obtained by putting everyones name in a bag, then drawing a portion of the
names out.
6 A two-way table records two sets of related information within one table, for example make of car and colour of car.
7 A frequency table is a table showing the number of times (frequency) that each particular value or item is recorded in a
survey or experiment.
8 Pictogram, bar chart, pie chart, line graph, frequency polygon, histogram.
9 A stemandleaf diagram is an ordered set of numerical data, grouped to show how the data is distributed. It is in effect
a bar chart using a list of numerical data.
10 The mode is the most common value; the median is the middle value of an ordered set of data; the mean is the total of
all the values divided by the number of items of data; the range is the difference between the largest and smallest values.
n n 3n
11 The median, lower and upper quartiles are obtained by reading off at , , , where n is the total
2 4 4
frequency. The interquartile range is the difference between the upper and lower quartile values.
12 The estimate of the mean is calculated by multiplying the mid-class values by the frequency for that class, summing up
these answers and dividing by the total frequency.
Ask the class to look through the table of vocabulary in the Pupil Book, to make sure they are familiar with the terms there.
The class can now do Exercise 13A from Pupil Book 3.

Extension Answers

The vertical axis (population) starts at 56 500 000, making population appear to more than double
in ten years. In fact it has only risen by about 2.4% as stated.

160 HarperCollinsPublishers Ltd 2003


9B3LP_13.qxd 18/09/2003 15:22 Page 161

Exercise 13A Answers

1 a Other categories not given, e.g. cycle or other b The categories overlap c 8.15 AM is in two categories
2 a Boys Girls b 45% c 18 T < 20 d 14 T < 16
12 T < 14 2 2
14 T < 16 4 1
16 T < 18 3 2
18 T < 20 0 4
20 T < 22 1 1
3 a 0 5 6 8 9 b 14 c 15 d 8
1 0 1 1 2 2 4 4 5 6 6 7 8 9 9
2 0 Key 0 | 6 means 6 students
4 a There are longer bars for females over 55 years old.
b Females may live longer generally in France; more men than women were killed in WWII (and WWI).
5 a mode = 1, median = 4, mean = 11.7 b mode = 5, median = 5, mean = 5.4 c mode = 4.50, median = 3.50, mean = 3.41
d mode = 18, median = 20, mean = 20.8
6 a Check that pie chart shows the following data: Class A B C D
b For this to be true the classes would have to
have equal numbers of students in.
Angle () 72 54 144 90 Key Words
7 a Height, h (cm) Cumulative frequency 8 Mean = 132.35 cm
h 100 0 data collection
h 120 7
h 130 39
raw data
h 140 80 primary source
h 150 97 secondary source
h 160 100
frequency table
c Median = 132.7 cm, IQR = 138.7 125.6 = 13.1 cm
frequency
diagram
Plenary population
pyramid
Explain to the class that it is important to be able to draw appropriate statistical diagrams
and calculate statistics such as averages, but that it is equally important to be able to
scatter graph
interpret them, commenting on how useful they are. cumulative
Ask the class if it is appropriate to use a pie chart to represent 20 categories of data [with frequency
so many categories, a bar chart might be easier to interpret], or ask the class if it is
sensible to use a scatter graph for 5 pairs of data [there are probably not enough data to lower quartile
produce a meaningful graph]. upper quartile
Briefly refer to the mode, median and mean and explain the effect of an extreme value on
the mean. Use the phrase, it would be inappropriate to use the mean as one value has
interquartile
distorted the data. For example, the data 50, 50, 50, 50, 100 give a mean of 50 when the range
100 is excluded, but a mean of 60 when the 100 is included.
Homework

1 The weights (in kg) of 24 men are given below. a Use the data to copy and complete the
62 48 55 67 81 40 45 59 58 62 frequency table.
72 65 70 82 66 48 59 68 71 65 Weight, W (kg) Tally Frequency
54 57 76 74 40 W < 50
b In which class is the median weight? 50 W < 60
c Complete a table of cumulative frequencies, 60 W < 70
draw the cumulative frequency graph and use it 70 W < 80
to calculate the median and interquartile range. 80 W < 90
d Explain why these weights are not representative of the whole adult population.
2 These tables show the average monthly temperatures for Paris and Madrid over the course of one year.
Paris Jan Feb Mar Apr May Jun Jul Aug Sep Oct Nov Dec
C 3.7 3.7 7.3 9.7 13.7 16.5 19.0 18.7 16.1 12.5 7.3 5.2
Madrid Jan Feb Mar Apr May Jun Jul Aug Sep Oct Nov Dec
C 5.3 6.7 9.7 12.0 16.1 20.8 24.6 23.9 20.5 14.7 9.3 6.0
a Draw suitable graphs to represent both sets of data.
b Comment on the differences between the average monthly temperatures in Paris and Madrid.
Answers
1 a Weight, W (kg) Tally Frequency c Weight (kg) Cumulative frequency
40 W < 50 |||| 4 W 40 0
50 W < 60 |||| | 6 W 50 4
60 W < 70 |||| || 7 W 60 10
70 W < 80 |||| 5 W 70 17
80 W < 90 || 2 W 80 22
b 60 W < 70 W 90 24
d Only males in results. Median = 62.9 kg, IQR = 72.0 51.6 = 20.4 kg.
2 a Histograms or line graphs drawn. b Average temperatures consistently higher in Madrid.

HarperCollinsPublishers Ltd 2003 161


9B3LP_13.qxd 18/09/2003 15:22 Page 162

LESSON Framework objectives A handling data project


Select, construct and modify, on paper and using ICT, suitable graphical
13.2 representation to progress an enquiry.
Interpret graphs and diagrams and draw inferences to support or cast doubt on
initial conjectures; have a basic understanding of correlation.
Communicate interpretations and results of a statistical enquiry using selected
tables, graphs and diagrams in support, using ICT as appropriate.
Solve substantial problems by breaking them into simpler tasks, using a range of
efficient techniques, methods and resources, including ICT.
Identify possible sources of bias and plan how to minimise it.
Identify what extra information may be required to pursue a further line of enquiry.
Analyse data to find patterns and exceptions, look for cause and effect and try to
explain anomalies.
Examine critically the results of a statistical enquiry, and justify choice of statistical
representation in written presentations, recognising the limitations of any
assumptions and their effect on conclusions drawn.

Oral and mental starter


Write on the board, A solo pop singer is more likely to be female than male, but a singer in a band is more
likely to be male than female.
Ask the class how they could investigate this.
Answers which they might say or be prompted to say could include:
Make a list of known singers and bands
Do a survey using the pop charts
You could discuss the fact that the chart data will only look at the most successful singers and therefore may
give a biased result. You could then suggest that the initial hypothesis needs to say that it refers to the most
successful singers.

Main lesson activity


The activities given in this section could easily take two lessons, depending on the amount of detail asked for
when carrying out the investigation. You may wish to ask the students to collect certain data prior to the lesson,
or you may decide to provide the students with secondary data.
Continuing from the oral and mental starter, show the class the handling data cycle and the related checklist for
completing a handling data investigation, on p. 218 of Pupil Book 3. The list is also reproduced opposite.
Go through and consider how each point applies to the pop singers example from the oral and mental starter:
[Statement of topic] Compare number of male singers and female singers in the charts.
[Hypothesis] In the charts, a solo singer is more likely to be female than male, but a singer in a band is
more likely to be male than female.
[Sample size] Look at, say, the top 50.
[Foreseen problems] Charts will change from week to week may need to use charts over several weeks.
[Identify any sources of bias and plan how to minimise them] A chart may only look at one type of music,
so consider different types of chart.
[How to obtain data] Use pop charts from different sources (this information could be obtained from the
Internet).
[Identify what extra information may be required] Ask yourself the question, How can I extend this
problem, using more complex techniques which will provide more reliable results?
[Data collection sheet] Record the number of bands and the number of solo artists, record the number of
males and females in each case.
[Analysis] Calculate average numbers per week. Represent in percentage bars or pie charts.
[Factors affecting results] Songs stay in the charts for several weeks, so the weeks looked at should really be
far apart in the year, rather than consecutive.
[Limitations of any assumptions made] The mean number of weeks may be distorted by an extreme value
(one singer may be in the charts for a relatively long time).
[Conclusion] State whether you agree with initial hypothesis based on your results.

The class can now do one of the investigations in Exercise 13B from Pupil Book 3, working in small groups.

162 HarperCollinsPublishers Ltd 2003


9B3LP_13.qxd 18/09/2003 15:22 Page 163

Plenary
Key Words
Having observed the students working on a choice of investigations, you may
wish to give one or more groups the opportunity to present their findings so far conjecture
to the rest of the class.

hypothesis
It is important that the students are encouraged to be critical of their own work
and that they can recognise any limitations. sample
Discuss how the problem can be extended using more complex data, for bias
example, analysing large sets of continuous data and carrying out more complex investigation
calculations, for example, using the interquartile range rather than the range in extension
order to overcome problems with extreme values. enquiry
limitations
Checklist for completing a handling data investigation
Specify the problem and plan
statement of problem or topic to investigate
hypothesis stating what you think the investigation will show
how you will choose your sample and size
any practical problems you foresee
identify any sources of bias and plan how to minimise them
how you will obtain your data
identify what extra information may be required to extend the project
Collect data from a variety of sources
follow initial plan and use a suitable data-collection sheet
Process and represent data
analysis of your results using appropriate statistical calculations and diagrams
Interpret and discuss data
comparison of results with your original hypothesis
list of any factors which might have affected your results and how you could
overcome these in future
consider the limitations of any assumptions made
a final conclusion
Homework

Choose one of the following tasks.


1 Complete the investigation started in the lesson by writing up the report.
2 Collect data in order to investigate the pop singers example.
3 Carry out and write up a detailed investigation of your own choice.

HarperCollinsPublishers Ltd 2003 163


9B3LP_14.qxd 18/09/2003 15:22 Page 164

CHAPTER
14 Shape, Space and Measures 4
LESSON Framework objectives Shape and space revision
Use units of measurement to calculate and solve problems in a variety of contexts.
14.1 Know and use the formulae for the circumference and area of a circle.
Calculate the surface area and volume of right prisms.

Oral and mental starter


The class should work in pairs or small groups for this activity.
Tell them that they are going to revise perimeter, area and volume, which they
have already covered.
Ask each pair or group to write down the formulae for the perimeter and the
area of any 2-D shapes.
Now ask each pair or group to write down the formulae for the surface area and
the volume of any 3-D shapes.
Give the class a few minutes to discuss their answers.

Main lesson activity


Once the class has finished writing down the formulae, ask individual students
to draw a shape on the board or OHT and below it give the formula for its
perimeter, area or volume.
Continue this activity until all the shapes are covered.
The class can now do Exercise 14A from Pupil Book 3.

164 HarperCollinsPublishers Ltd 2003


9B3LP_14.qxd 18/09/2003 15:22 Page 165

Exercise 14A Answers

1 a i 12 cm ii 9 cm2 b i 28 cm ii 45 cm2 c i 16 cm ii 12 cm2


d i 52 cm ii 128 cm2 e i 30 cm ii 30 cm2
2 a i 18.8 cm ii 28.3 cm2 b i 28.3 cm ii 63.6 cm2
c i 31.4 cm ii 78.5 cm2 d i 39.6 cm ii 124.7 cm2
3 a i 150 cm2 ii 125 cm3 b i 160 cm2 ii 100 cm3 c i 108 cm2 ii 48 cm3
d i 736 cm2 ii 960 cm3
4 a 226 cm3 b 295 cm3 c 7.39 m3
5 a 6.4 cm b 31.8 cm2
6 7.1 cm

Extension Answers

366 m, 6060 m2

Plenary Key Words


Tell the class that in the next lesson they will be working on a shape and space
area
investigation, and that they may need to use some of the formulae which they
met during the present lesson. circumference
surface area
volume
Homework

1 Find the area of each of the following shapes.


a b c d 15 cm

5 cm 8 cm 12 cm 6 cm

9 cm 6 cm 15 cm 5 cm

2 Calculate i the circumference and ii the area of each of the following circles. Take = 3.14 or use
the key on your calculator. Give your answers to one decimal place.
a b

8 cm 20 cm

3 Calculate the volume of this prism.

3m

2m 12 m
5m

Answers
1 a 45 cm2 b 24 cm2 c 180 cm2 d 60 cm2
2 a i 50.3 cm ii 201.1 cm2 b i 62.8 cm ii 314.2 cm2
3 96 m3

HarperCollinsPublishers Ltd 2003 165


9B3LP_14.qxd 18/09/2003 15:22 Page 166

LESSONS Framework objectives Shape and space investigation


Present a concise, reasoned argument, using symbols, diagrams and related
14.2 explanatory text.
Suggest extensions to problems, conjecture and generalise: identify exceptional
cases or counter-examples, explaining why.
14.3
Oral and mental starter
Ask individual students to give the formulae for the perimeter, the area or the volume of various 2-D and 3-D
shapes.

Main lesson activity


Each investigation given in this section will take up to two lessons to complete.
The class can work in pairs or small groups. The whole class may undertake the same investigation, or pairs or
groups may choose their own investigation from the four given.
Before the investigations are started, briefly go through the methods of doing an investigation:
Draw some easy examples first, making all diagrams clear with all measurements shown.
Put your results in a table with suitable headings.
Look for any patterns among the entries in the table.
Describe and explain any patterns which you spot.
Try to find a rule or formula to explain each pattern.
Try another example to see whether your rule or formula does work.
Summarise your results with a conclusion.
If possible, extend the investigation by introducing different questions.
Note: the students may need centimetre-squared paper.
The class can now do Exercise 14B from Pupil Book 3.

Exercise 14B Answers

The layout below for each investigation is minimal. The students should always explain their choice of presentation and
link all their diagrams with the text. They may use different approaches to the investigations and these should be noted. The
use of algebra would also enhance the students work and should be encouraged wherever possible. All the investigations
are helpful to the preparation for the GCSE coursework assessment.
1 It is expected that the students will draw a sequence of
Size of square 1 1 2 2 3 3 4 4 5 5
squares and complete a table similar to the one on the right.
Note that units are not necessary for this investigation. Perimeter (P) 4 8 12 16 20
From the table, the students will notice that the 4 4 square Area (A) 1 4 9 16 25
has the same value for its perimeter and area. They should see
that this is the only solution, by noticing that, after the 4 4 square, the area is always greater than the perimeter. They
should then check this using another example. (It could also be shown by drawing a graph.)
Some students may be able to show that this is the only solution by using an algebraic approach.
For a square of side l, P = 4l and A = l 2.
l For the square to have the same value for P and A, 4l = l 2.
This is true only for l = 0 or l = 4. Clearly, l = 0 has no meaning, so l = 4 is the only solution.
l To extend the investigation, the students could consider rectangles.
By drawing a number of rectangles, they should find that the value of the perimeter equals the
value of the area when l = 6 and w = 3.
They may think that this is the only solution, but intuition may tell them that other solutions
may exist. For completeness, this can be shown by an algebraic approach:
w
For a rectangle of length l and width w, P = 2l + 2w and A = lw.
For the rectangle to have the same value for P and A, 2l + 2w = lw, which gives:
l lw 2w = 2l
w( l 2) = 2l
2l
w =
l2
This shows that there are an infinite number of solutions. For example, when l = 10, w = 2.5. Other solutions can now
be found.
Considering circles would also extend the investigation.

166 HarperCollinsPublishers Ltd 2003


9B3LP_14.qxd 18/09/2003 15:22 Page 167

Exercise 14B (contd)

2 The students should start by completing a table of results to Size of square l P A l:P l:A
show the length (l), perimeter (P) and area (A) of the squares.
11 1 4 1 1:4 1:1
They can then find the ratios l : P and l : A.
22 2 8 4 1:4 1:2
From the table, the students can see that the ratio l : P is
always 1 : 4 and the ratio l : A is 1 : n, where n is the length of 3 3 3 12 9 1:4 1:3
the side of the square. They should now test their rules on new data.
They may be able to generalise: for a square of size n n, l : P = 1 : 4 and l : A = 1 : n.
The investigation could be extended by looking at cubes and considering the ratio of the length of a side to the surface
area and the ratio of the length of a side to the volume.
3 The students should start by considering squares of different sizes. They will realise that it will be easier to use even
numbers for the side of the square. They should then present their results in a table, as shown below, giving their final
answers to a suitable degree of accuracy. (One decimal place is suggested.)
Size of square Area of square Area of coin Area of coin as % % waste
of area of square
2 cm 2 cm 4 cm2 3.14 cm2 78.5% 21.5%
4 cm 4 cm 16 cm2 12.57 cm2 78.5% 21.5%
6 cm 6 cm 36 cm2 28.27 cm2 78.5% 21.5%
The students should now see that the percentage waste is always 21.5% for squares of any size. A further example
would show this.
An algebraic approach could also be used. For a square of side 2r, the area of the square is 4r2 and the area of the coin
is r2.
The area of the coin as a percentage of the area of the square is:
r2
100 = 100 = 78.5
4r2 4
So, the percentage waste is 21.5%.
The investigation could be extended to four coins stamped from a square or by considering rectangular sheets of metal.
4 a For a 6 2 table, there are two bounces.
b For a 6 2 table, the ball goes down pocket C.
c After drawing different sized tables, including square tables, the students Size of table Number of Pocket
should complete a table to show their results. The table on the right shows bounces
some results for 17 different tables, with the ball always starting from 11 0 C
pocket A. 22 0 C
From the table, the students should be able to spot various patterns. For 33 0 C
example: a square table has no bounces and the ball ends up in pocket C;
when the length and width of the table are reversed, the number of 21 1 B
31 2 C
bounces is the same but the ball may not end up in the same pocket. 41 3 B
The following is a summary.
For a table of length l and width w, write it as the ratio l : w. 32 3 D
When l : w cannot be simplified, the number of bounces, N, is given by 42 1 B
the formula: 52 5 D
N=l+w2 62 2 C
When l : w can be simplified to give the ratio a : b, the number of 43 5 B
bounces, N, is given by the formula: 53 6 C
N=a+b2 63 1 B
When l and w (or a and b if simplified) are both odd, the ball ends up in
pocket C. 23 3 B
When only l is odd (or a if simplified), the ball ends up in pocket D. 34 5 D
If only l is even (or a if simplified), the ball ends up in pocket B. 35 6 C
36 1 D
The students may write down these conditions, but in all cases they
should test their rules or formulae on further examples.

Plenary Key Words


Having observed the students working on the investigations, you may wish to investigate
discuss one of their choices.
generalise
Homework

Complete the investigation you started in the lesson.

HarperCollinsPublishers Ltd 2003 167


9B3LP_14.qxd 18/09/2003 15:22 Page 168

LESSON Framework objectives Symmetry revision


Visualise and use 2-D representations of 3-D objects.
14.4 Use rotations and reflections on paper. Identify reflection symmetry in 3-D shapes.

Oral and mental starter


The class should work in pairs or small groups for this activity.
Tell them that they are going to revise reflection and rotational symmetry, which
they have already covered.
Ask each pair or group to draw and write down the names of any 2-D shapes,
and then to draw on them all the lines of symmetry.
Now ask them to repeat the activity but to write down the order of rotational
symmetry below each one. Give the class a few minutes to discuss their
answers.
Then ask them to write down how to find the number of planes of symmetry for
a 3-D shape.

Main lesson activity


Once the class have finished writing down their answers, ask individual students
to draw a shape on the board or OHT and to insert its lines of symmetry or to
give its order of rotational symmetry.
Continue this activity until it is clear that the class understand the concepts.
Briefly remind them how to find the number of planes of symmetry for a 3-D
shape by showing them that a cuboid has three planes of symmetry.
The class can now do Exercise 14C from Pupil Book 3.

168 HarperCollinsPublishers Ltd 2003


9B3LP_14.qxd 18/09/2003 15:22 Page 169

Exercise 14C Answers

1 a 2 b 2 c 6 d 4 e 5
2 a 2 b 1 c 1 d 4
3 a 2 b 2 c 5 d 4 e 2
4 a 4 b 3 c 4 d 2
5 a 2 b 3 c 2 d 3
6 Cuboid with two square faces

Plenary Key Words


Tell the class that in the next lesson they will be working on a symmetry
reflection
investigation and that they may need to use some of the concepts which they
met during the present lesson. symmetry
rotational
symmetry
plane of
symmetry
Homework

Design a logo for a badge for your school, which has both reflection and rotational symmetry.

HarperCollinsPublishers Ltd 2003 169


9B3LP_14.qxd 18/09/2003 15:22 Page 170

LESSONS Framework objectives Symmetry investigation


Present a concise, reasoned argument, using symbols, diagrams and related
14.5 explanatory text.
Suggest extensions to problems, conjecture and generalise: identify exceptional
cases or counter-examples, explaining why.
14.6
Oral and mental starter
Ask individual students to explain reflection symmetry, rotational symmetry and planes of symmetry.

Main lesson activity


Each investigation given in this section will probably take two lessons.
The class can work in pairs or groups and the class can all work on the same investigation or be allowed to
make their own choice from the four investigations given.
Before the class start the investigation, briefly go through the methods of doing an investigation:
Draw some easy examples first, showing any lines of symmetry and/or stating the order of rotational
symmetry on the diagrams.
Explain anything you notice from the diagrams.
Describe and explain any patterns which you spot.
Summarise your results with a conclusion.
If possible, extend the investigation by introducing different questions.
Note: the students may need tracing paper, mirrors, centimetre-squared paper and a selection of 3-D solids.
The class can now do Exercise 14D from Pupil Book 3.

Exercise 14D Answers

The layout below for each investigation is minimal. The students should always explain their choice of presentation and
link all their diagrams with the text. They may use different approaches to the investigations and these should be noted. The
use of algebra would also enhance the students work and should be encouraged wherever possible. All the investigations
are helpful to the preparation for the GCSE coursework assessment.
1 It is expected that the students will draw diagrams to show the number of lines of symmetry for a tile with different
numbers of shaded squares. Examples are shown below. (Reflections and rotations are omitted.)

One square

1 line of symmetry 4 lines of symmetry


Two squares

1 line of symmetry 2 lines of symmetry


Three squares

1 line of symmetry 2 lines of symmetry


Four squares

1 line of symmetry

4 lines of symmetry
The students should now notice that for five shaded squares, the diagrams are the same as for four squares but with the
shading reversed. Similarly for six, seven and eight squares. This could now be usefully summarised in a table.
The investigation could be extended by using a different size of tile.

170 HarperCollinsPublishers Ltd 2003


9B3LP_14.qxd 18/09/2003 15:22 Page 171

Exercise 14D (contd)

2 There are 12 different pentominoes (excluding mirror images). The students should be encouraged to show their results
in a table.

Pentomino

Number of lines of symmetry 4 2 1 1 1 1


Order of rotational symmetry 4 2 1 1 1 1

Pentomino

Number of lines of symmetry 0 0 0 0 0 0


Order of rotational symmetry 2 1 1 1 1 1
For students who extend the investigation, there are 35 different hexominoes.
3 Allowing rotations, the T-shape will fit in the 3 3 grid four times.
The students should now draw square grids of different sizes and find the number of ways in which the T-shape fits
inside each grid. The table below shows the number of ways for three different square grids.
Size of grid 33 44 55
Number of ways 4 16 36
The students should now be able to spot the rule: the numbers of ways give the even square numbers. They should test
another example to confirm that the rule works.
The students may also be able to use algebra in this investigation and arrive at a formula.
For an n n grid, the total number of ways, T, the T-shape will fit inside the grid is given by:
T = 4(n 2)2 for n > 2
To extend the investigation, the students could consider rectangular grids or use a different shape.
4 For this investigation, it is useful for the students to have access to a collection of different solids. The table below is
not exhaustive.
3-D solid Outline Symmetry number
Cube Square 24
Cuboid Rectangle 4
Regular tetrahedron Equilateral triangle 12
Square-based pyramid Square 4
Regular octahedron Square 4
Regular triangular prism Equilateral triangle 6
Regular pentagonal prism Regular pentagon 10
Regular hexagonal prism Regular hexagon 12
Cylinder Circle
Sphere Circle
The students may notice that the symmetry number for any prism is twice the order of rotational symmetry for the
cross-section of the prism.

Plenary Key Words


Having observed the students working on the investigations, you may wish to
investigate
discuss one of their choices.
generalise
Homework

Complete the investigation you started in the lesson.

HarperCollinsPublishers Ltd 2003 171


9B3LP_15.qxd 18/09/2003 15:22 Page 172

CHAPTER
15 Handling Data 4
LESSON Framework objectives Revision of probability
Use the vocabulary of probability in interpreting results involving uncertainty and
15.1 prediction.
Identify all the mutually exclusive outcomes of an experiment; know that the sum
of probabilities of all mutually exclusive outcomes is 1 and use this when solving
problems.
Estimate probabilities from experimental data.
Compare experimental and theoretical probabilities in a range of contexts;
appreciate the difference between mathematical explanation and experimental
evidence.
Understand relative frequency as an estimate of probability and use this to compare
outcomes of experiments.

Oral and mental starter


Explain to the class that the lesson will be used to revise the probability topics they have already covered, in preparation for
an investigation next lesson.
Ask the class a few simple probabilities. Ask the students to explain how they have worked each answer out. Some
examples are given below.
rolling a 6 on a fair, six-sided dice picking a blue cube when there are 3 blue, 4 red
rolling a 3 or a 4 on a fair, six-sided dice and 3 green cubes in a bag
rolling an odd number on a fair, six-sided dice not picking a blue cube in the above example
throwing a head on a fair coin it not raining, if the probability of rain is 43

Main lesson activity


Continuing from the oral and mental starter, ask the students to give you responses to the following types of questions.
1 A box contains 60 sweets, of which 10 are strawberry.
a What is the probability of picking a strawberry sweet?
b What is the probability of picking a sweet that is not strawberry?
c A second box of sweets of the same make contains 300 sweets, how many would you expect to be strawberry?
2 A six-sided dice is rolled and lands on 6 four times out of 24 rolls. Do you think that the dice is fair? Give a reason.
3 Two coins are thrown. How many different outcomes are there?
4 What is the difference between theoretical and experimental probability?
5 What could you do to test if a spinner is biased? 8 What are exhaustive events?
6 What are independent events? 9 When do you use a tree diagram?
7 What are mutually exclusive events? 10 What is relative frequency?
You should look for answers which refer to the following.
1 a 10 or 16 b 56 c 50
60
2 It appears biased as 6 out of 24 is 41 . If fair you would expect it to be close to 16 .
3 Four, HH, HT, TH, TT
4 Theoretical probability looks at equally likely outcomes whereas experimental probability is based on the results of an
experiment or number of trials.
5 Roll it many times and compare the results to see if the experimental probabilities are close to the theoretical
probabilities based on the assumption that the spinner is fair.
6 Independent events are events where the outcome of one event is not affected by the outcome of the other event and
vice versa.
7 Mutually exclusive events are events that cannot happen at the same time.
8 Exhaustive events are events that cover every possible outcome.
9 A tree diagram is used to help calculate combined probabilities for more than one event.
10 Relative frequency is an estimate of probability based on experimental data.
Number of successful trials
Relative frequency = .
Total number of trials
Ask the class to look through the table of vocabulary in the Pupil Book, to make sure they are familiar with the terms there.

The class can now do Exercise 15A from Pupil Book 3.

172 HarperCollinsPublishers Ltd 2003


9B3LP_15.qxd 18/09/2003 15:22 Page 173

Plenary
Key Words
Explain to the class that it is important to be able to calculate probabilities from
experimental and theoretical situations in order to make and test hypotheses.
Ask the class to explain the meaning of bias. Point out that bias can be tested either by event
looking at raw data or by comparing experimental and theoretical probabilities. outcome
As an example, show the class the table below, which gives the results of spinning a
four-sided spinner 40 times. Ask the class if they think the spinner is biased, just by random
looking at these experimental frequencies. probability scale
Now ask them to work out the theoretical probabilities and compare with the
experimental probabilities.
experimental
probability
Number on spinner 1 2 3 4
Frequency 12 5 10 13 theoretical
From inspection of the data, the spinner appears to be biased against landing on 2, probability
because this occurs much less than the other numbers. Comparison of the experimental relative
5 = 0.125;
with the theoretical probability verifies this conclusion (experimental = 40
theoretical = 41 = 0.25).
frequency
Finally, point out that minor differences between expected frequencies and actual expectation
frequencies do not necessarily mean that there is bias. bias
fair
Extension Answers
sample
1 a Independent as 2nd roll is not affected by outcome of 1st roll.
sample space
b Not independent the chances of winning with 2nd ticket are increased as there is 1 ticket exhaustive
fewer to choose from.
c Not independent. independent
2 a Not mutually exclusive as 2 is both even and prime. mutually
b Mutually exclusive as the outcomes do not overlap.
c Not mutually exclusive as at least one tail includes two tails. exclusive
3 a Exhaustive as all possible outcomes are included.
b Not exhaustive as the outcome landing on the number 3 is not included.
c Exhaustive as all possible outcomes are included.

Exercise 15A Answers

1 a 8; HHH, HHT, HTH, THH, TTH, THT, HTT, TTT b 18 c 38 d 87


2 a 0.4 b 0.6 3 a 50 7 b 12
50 6 c 50
= 25 31

4 6 or 0.12 b 15 or 0.2 c Probably not fair as the probabilities are quite different. d 30 5 a 15 b 35 c 20
a 50 3
6 a b You only know that 4 out of the first 5 games were won.
c 0.76
Relative frequency

1 d Number of games 5 10 15 20 25
0.8 Relative frequency of wins 0.8 0.7 0.67 0.75 0.76
0.6 Number of wins 4 7 10 15 19
0.4
0.2
0
0 5 10 15 20 25
Number of games
Homework

Two four-sided spinners are each spun 80 times. The results are shown below.
For each spinner state whether you think it is biased by comparing i the individual frequencies ii the experimental
and theoretical probabilities.
1st spinner Number on spinner 1 2 3 4
Frequency 20 21 19 20
2nd spinner Number on spinner 1 2 3 4
Frequency 25 17 16 22

Answers
The 1st spinner is probably not biased: i as the frequencies are all close to 20
ii as the experimental probabilities are 20 21
= 0.25, 80
80 = 0.2625, 19
= 0.2375, 20
80 = 0.25, which are all close to the theoretical
80
probability of 0.25.
The 2nd spinner is possibly biased: i as the frequencies are not close to 20
25
= 0.3125, 17 = 0.2125, 16 22
= 0.2, 80
= 0.275, which are not very close to the theoretical
ii as the experimental probabilities are 80 80 80
probability of 0.25. It could still be argued here that these are sufficiently close to 0.25 to suggest the spinner is fair.

HarperCollinsPublishers Ltd 2003 173


9B3LP_15.qxd 18/09/2003 15:22 Page 174

LESSON Framework objectives A probability investigation


Compare experimental evidence in a range of contexts; appreciate the difference
15.2 between mathematical explanation and experimental evidence.
Understand relative frequency as an estimate of probability and use this to compare
outcomes of experiments.

Oral and mental starter


Write on the board, Teenagers are better at probability than adults.
Ask the class how they could investigate this.
Students might suggest writing a set of probability questions to be given to both
teenagers and adults. They could then record the results for their samples and
compare the experimental probabilities of answering particular questions
correctly for teenagers and adults.
You could discuss how they would decide which people to use in the sample.

Main lesson activity


The activities given in this section could easily take two lessons, depending on
the amount of detail asked for when carrying out the investigation. You may
wish to ask the students to collect certain data prior to the lesson or you may
decide to provide the students with secondary data.
Continuing from the oral and mental starter, show the class the handling data
cycle and the related checklist for completing a probability investigation, on
pp. 232233 of Pupil Book 3. The list is also reproduced opposite.
Go through and consider how each point applies to the example from the oral
and mental starter:
[Statement of topic] Compare the abilities of teenagers and adults at
probability.
[Hypothesis] Teenagers are better at working out theoretical probabilities
than adults.
[Sample size] Look at 30 teenagers and 30 adults. It would not be sensible to
use students in your class who have just revised probability. You may even
want to test a different area of mathematics if using your fellow students as a
sample.
[Foreseen problems] Adults may be reluctant to answer the questions given.
Choosing the sample may be difficult.
[Identify any sources of bias and plan how to minimise them] Avoid using
all one age group for adults.
[Data collection] Record the number of correct and incorrect answers, the
number of people who declined to do the questions and any other factors
which may affect your results.
[Identify what extra information may be required] Ask yourself the question,
How can I extend this problem, using more complex techniques which will
provide more reliable results?
[Analysis] Produce statistical diagrams to compare the success rates of
teenagers and adults, and calculate the experimental probability of each
answering a probability question correctly.
[Limitations of any assumptions made] Children may have had a more
recent experience than adults of work on probability, whereas adults may
quickly understand the topic, if they were given a short briefing.
[Conclusion] State whether you agree with the initial hypothesis based on
your results.
The class can now do Exercise 15B from Pupil Book 3, working in small groups.

174 HarperCollinsPublishers Ltd 2003


9B3LP_15.qxd 18/09/2003 15:22 Page 175

Plenary
Key Words
Having observed the students working on a choice of investigations, you may
wish to give one or more groups the opportunity to present their findings so far hypothesis
to the rest of the class.

experimental
It is important that the students are encouraged to be critical of their own work
and that they can recognise any limitations. data
Discuss how the problem can be extended using more complex analysis. mathematical
Discuss the effect of a small number of trials on the reliability of any relative explanation
frequencies as estimates of theoretical probability. statistical report
experimental
Checklist for completing a probability investigation probability
Specify the problem and plan theoretical
statement of problem or topic to investigate
probability
hypothesis stating what you think the investigation will show
how you will choose your sample and size
Relative
any practical problems you foresee frequency
identify any sources of bias and plan how to minimise them
how you will obtain your data
identify what extra information may be required to extend the project
Collect data from a variety of sources
follow initial plan and use a suitable data-collection sheet
Process and represent data
analysis of your results using appropriate statistical calculations and diagrams
Interpret and discuss data
comparison of results with your original hypothesis
list of any factors which might have affected your results and how you could
overcome these in future
consider the limitations of any assumptions made
a final conclusion
Homework

Choose one of the following tasks.


1 Complete the investigation started in the lesson by writing up the report.
2 Collect data in order to investigate the ability of teenagers and adults at working out theoretical
probabilities.
3 If you have completed the report of your first investigation, then carry out and write up another
detailed investigation of your own choice.

HarperCollinsPublishers Ltd 2003 175


9B3LP_16.qxd 18/09/2003 15:22 Page 176

CHAPTER
16 GCSE Preparation
LESSON Framework objectives Reinforcement of Number
Solving quadratic equations.
16.1

Oral and mental starter


Ask students to mentally solve x 1 = 0. They should find that x = 1 quickly
(answers can be written on mini white boards).
Repeat with x + 4 = 0 (4), x 3 = 0 (3), x + 3 = 0 (3).
Now ask students to give two values for a and b that solve a b = 0. Ask
students to write their answers down on mini white boards as a = ? and b = ?.
Check answers. Some could be written on the board. Discuss the common
characteristics, which should be that either a or b should be zero in each case.
Now ask students to write down a value for x that will make (x 2)(x + 4) = 0.
Once again check answers. Hopefully they will be either x = 2 or x = 4.

Main lesson activity


This is a lesson on solving quadratics that factorise.
Ask students if they can find a value for x that solves the quadratic equation
x2 + 6x 7 = 0.
They may spot the answer x = 1, but are unlikely to spot the answer of x = 7.
Outline the method. First, factorise and then solve each bracket equal to zero.
i.e. x2 + 6x 7 = 0 (x 1)(x + 7) = 0.
So, either x 1 = 0 x = 1, or x + 7 = 0 x = 7.
Repeat with other examples such as:
x2 x 6 = 0 (x = 3 or 2), x2 + 4x + 3 = 0 (x = 1 or 3),
x2 8x + 15 = 0 (x = 3 or 5).
Do more examples if necessary.
The class can now do Exercise 16A from Pupil Book 3 or Exercise 10K
(page 256) from the Higher Mathematics for GCSE textbook.

176 HarperCollinsPublishers Ltd 2003


9B3LP_16.qxd 18/09/2003 15:22 Page 177

Exercise 16A Answers

1 a x = 1 or 1 b x = 2 or 5 c x = 3 or 6 d x = 4 or 3 e x = 2 or 7
f x = 3 or 8 g x = 8 or 1 h x = 3 i x = 4
2 a x = 1 or 2 b x = 5 or 6 c x = 2 or 4 d x = 3 or 2 e x = 5 or 2
f x = 1 or 4 g x = 5 h x = 4 i x = 5 or 3 j x = 3 or 5 k x = 6 or 4
l x = 3 or 2 m x = 9 or 1 n x = 6 or 3 p x = 1

Plenary Key Words


Ask the students if they can solve 2x2 3x + 1 = 0.

quadratic
They may spot x = 1 as a solution. Give a clue that the other answer is a fraction
between 0 and 1. equations
They should find 12 fairly quickly. unitary
Discuss ways that this could be solved mathematically. coefficient
The students may establish that the factorisation is (x 1)(2x 1) = 0 and that
these brackets solve to 1 and 12.
Do more examples with a non-unit coefficient of x2, if time allows, such as
2x2 5x + 3 = 0.
Homework

1 Solve these equations.


a (x + 3)(x 4) = 0 b (x 1)(x + 6) = 0 c (x 7)(x + 6) = 0
d (x + 5)(x + 2) = 0 e (x 3)(x + 6) = 0 f (x 9)(x 3) = 0
2 First factorise, then solve these equations.
a x2 + 8x + 15 = 0 b x2 + 13x + 30 = 0 c x2 + 4x 5 = 0
d x2 9x + 14 = 0 e x2 + 4x 21 = 0 f x2 4x + 4 = 0

Answers
1 a x = 3 or 4 b x = 1 or 6 c x = 7 or 6 d x = 5 or 2 e x = 3 or 6 f x = 9 or 3
2 a x = 5 or 3 b x = 3 or 10 c x = 1 or 5 d x = 2 or 7 e x = 3 or 7 f x = 2

HarperCollinsPublishers Ltd 2003 177


9B3LP_16.qxd 18/09/2003 15:22 Page 178

LESSON Framework objectives Reinforcement of Number


Factorisation of quadratics of the form ax2 + bx + c, where a > 1.
16.2
Oral and mental starter
Give students the factors of 12 in pairs and the factors of 15 on each side of a
vertical line, i.e.
1 2 3 1 3
12 6 4 15 5
Ask the students to find a combination of products using one pair of factors from
each side that give a total of 28. This first exemplar may need to be
demonstrated i.e.
2 3 2 5 + 6 3 = 28.
6 5
Repeat with totals of 29(3 3 + 4 5), 27(3 5 + 4 3 or 1 15 + 12 1),
41(1 5 + 12 3).
Now ask for the total 8. Suggest to the students that negative values could be
used. The answer is
2 3 2 5 + 6 3 = 8.
6 5
Repeat with totals of 24(2 15 + 6 1), 3(3 5 + 4 3).
Do more examples if time available.

Main lesson activity


This is a lesson on factorising quadratics with a non-unit coefficient for x2.
Students have met the idea in previous plenaries, but a method for solution has
not been outlined.
Ask the students to factorise 2x2 + 9x + 4.
They may have an intuitive idea of the answer but they also need to have a
method outlined.
There are several methods, two of which are outlined in the Pupil Book.
Example: Find factors of the x2 coefficient and of the constant term.
For 2x2 + 9x + 4, the factors of 2 are 1 2 and the factors of 4 are 1 4.
Now find a combination that gives the coefficient of x, e.g.
1 1 11+24=9
2 4
The brackets are then the opposite to the pairs i.e. (x + 4)(2x + 1).
Example: Factorise 6x2 17x + 12.
The brackets must start (3x )(2x ) or (6x )(x ) and the constant term has
factors 3 4, 2 6, 1 12.
By trial and improvement we can find that the combination (3x 4)(2x 3) works.
Do more examples if necessary.
The class can now do Exercise 16B from Pupil Book 3 or Exercise 10L
(page 257) from the Higher Mathematics for GCSE textbook.

178 HarperCollinsPublishers Ltd 2003


9B3LP_16.qxd 18/09/2003 15:22 Page 179

Exercise 16B Answers

1 a 2x2 + 11x + 5 b 3x2 + 9x 12 c 4x2 18x + 20 d 6x2 15x 21


e 4x2 + 24x + 36 f 9x2 24x + 16 g 6x2 22x 8 h 8x2 + 10x 3 i 4x2 1
2 a (2x + 1)(x + 3) b (x + 2)(2x + 5) c (x + 4)(3x + 1) d (x 1)(2x + 1)
e (2x + 1)(3x + 2) f (x 2)(2x + 3) g (x + 3)(2x 3) h (2x + 1)2
i (4x 1)(x + 2) j (5x + 1)(x + 2) k (3x 1)(x + 1) l (4x + 1)(2x + 1)
m (x 2)(3x + 1) n (2x + 1)(3x 1) p (4x + 1)(x 3) q (2x 3)(2x + 5)
r (x 7)(2x + 5) s (x 5)(2x + 5) t (3x 1)(x + 5) u (3x + 1)2
v (2x + 3)(5x 1)

Plenary Key Words


Ask the students if they can solve the quadratic equation 2x2 3x 2 = 0.

quadratic
They should be able to put all previous ideas together to explain the process of
factorisation and solving each bracket. equations
e.g. (2x + 1)(x 2) = 0 giving x = 12 or x = 2 non-unitary
Repeat with other examples, if time allows, such as 3x2 5x 2 = 0. coefficients
Homework

1 Expand these brackets into quadratic expressions.


a (3x + 1)(x 4) b (3x 1)(x + 5) c (2x 1)(2x + 3)
d (3x 2)(3x + 2) e (3x 1)2 f (2x + 5)2
2 Factorise the following quadratic expressions.
a 2x2 7x 4 b 2x2 + 13x + 15 c 3x2 + 5x 2
d 4x2 + 23x 6 e 6x2 5x + 1 f 6x2 + 11x + 3
g 5x2 26x + 5 h 6x2 5x 6 i 4x2 16x + 15

Answers
1 a 3x2 11x 4 b 3x2 + 14x 5 c 4x2 + 4x 3 d 9x2 4 e 9x2 6x + 1 f 4x2 + 20x + 25
2 a (2x + 1)(x 4) b (2x + 3)(x + 5) c (3x 1)(x + 2) d (4x 1)(x + 6) e (3x 1)(2x 1)
f (3x + 1)(2x + 3) g (x 5)(5x 1) h (3x + 2)(2x 3) i (2x 3)(2x 5)

HarperCollinsPublishers Ltd 2003 179


9B3LP_16.qxd 18/09/2003 15:22 Page 180

LESSON Framework objectives Reinforcement of Number


Solving quadratic equations of the form ax2 + bx + c = 0, where a > 1.
16.3
Oral and mental starter
Ask students to mentally solve 2x 1 = 0. They should find that x = 12. Answers
can be written on mini white boards.
Repeat with 3x + 4 = 0 (113), 4x 3 = 0 ( 34 ), 2x + 3 = 0 (112 ), 5x 1 = 0 ( 15 ),
3x 2 = 0 ( 23 ).
Do more examples if necessary.
Now ask students to write down a value for x that will make (3x 2)(4x + 1) = 0.
Once again check answers. They should be x = 23 or x = 14 .

Main lesson activity


This is a lesson on solving quadratic equations with non-unitary coefficients.
Ask students if they can find a value for x that solves the quadratic equation
2x2 x 1 = 0.
They may spot the answer x = 1, but are unlikely to spot the answer of 12 .
Outline the method. First factorise and then solve each bracket equal to zero.
i.e. 2x2 x 1 = 0 (2x + 1)(x 1) = 0, so either x 1 = 0 x = 1 or
2x + 1 = 0 x = 12 .
Repeat with other examples such as 3x2 + 2x 1 = 0 (x = 13 or 1),
12x2 + 5x 2 = 0 ( x = 23 or 14 ), 8x2 14x 15 = 0 (x = 34 or 212 ).
Do more examples if necessary.
Ask students to solve the quadratic equation x2 + x = 12.
This is not in the correct form to factorise and solve. Encourage students to
always rearrange quadratic equations into the form ax2 + bx + c = 0.
The equation above then becomes x2 + x 12 = 0, which can be factorised and
solved to give x =3 or 4.
Repeat with 2x(x 7) = 6 3x. This must be expanded and then collected into
the correct form.
2x2 14x = 6 3x 2x2 11x 6 = 0 (2x + 1)(x 6) = 0 x = 12 or 6.
Do more examples if necessary.
The class can now do Exercise 16C from Pupil Book 3 or Exercise 10M
(page 258) from the Higher Mathematics for GCSE textbook.

180 HarperCollinsPublishers Ltd 2003


9B3LP_16.qxd 18/09/2003 15:22 Page 181

Exercise 16C Answers

1 a x = 12 or 3 b x = 2 or 13 c x = 4 or 12 d x = 112 or 12 e x = 6 or 12
f x = 2 or 14 g x = 212 or 13 h x = 13 or 12 i x = 112 j x = 14 or 15
k x = 212 or 23 l x = 13 m x = 12 n x = 56 or 1 p x = 123 or 2
2 a x = 2 or 1 b x = 1 or 1 c x = 12 or 1 d x = 23 or 1 e x = 14 or 2
f x = 12 g x = 5 or 5 h x = 12 or 113 i x = 14 or 15

Plenary Key Words


Write the following equation on the board x4 5x2 + 4 = 0.

quadratic
Explain that the equation has 4 solutions since the highest power of x is 4.
Ask students if they can spot any of the solutions. They may spot 1 and 2 but equations
may not spot 1 and 2. non-unitary
Outline the method of solution ( x2 1)( x2 4) = 0. coefficients
So, x2 1 = 0 x2 = 1 x = 1 or x2 4 = 0 x2 = 4 x = 2. roots
Repeat with 4x4 37x2 + 9 = 0.
(4x2 1)(x2 9) = 0 x = 12 or 3.
Homework

1 Solve these equations.


a 2x2 15x + 7 = 0 b 3x2 5x + 2 = 0 c 2x2 9x 5 = 0
d 24x2 + 14x 5 = 0 e 6x2 + 23x + 20 = 0 f 6x2 23x + 7 = 0
2 Solve these equations
a x2 + x = 6 b 2x(x + 4) = 3(x 1) c 8x2 3x + 4 = 2x2 + 2x + 3

Answers
1 a 7 or 12 b 23 or 1 c 12 or 5 d 14 or 56 e 212 or 113 f 312 or 13
2 a 2 or 3 b 3 or 12 c 13 or 12

HarperCollinsPublishers Ltd 2003 181


9B3LP_16.qxd 18/09/2003 15:22 Page 182

LESSON Framework objectives Reinforcement of Number


The quadratic formula.
16.4
Oral and mental starter


4 + 41
(
4 + 6.5 2.5
Ask students to estimate the value of = = 1.25 .
2 2 2 )

3 52 5 + 31 1 13
Repeat with ( 2.5), ( 0.25), ( 1.1),
4 2 4

4 + 21
( 0.25).
2

5 60
Repeat with (two answers required) ( 1.25 or 6.25),
2

1 17
( 0.75 or 1.25).
4

Main lesson activity


This is a lesson that introduces the quadratic formula.
Introduce the formula:

b b2 4ac
x =
2a
and explain that a, b and c are the coefficients of x2, x and the constant term in
the general quadratic equation ax2 + bx + c = 0.
Work through some examples but encourage students to follow some basic
principles:
Substitute values in brackets before attempting to work anything out.
Take particular care with: The change of sign of b.
Squaring b inside the root, particularly if it is negative
Working out 4 a c correctly particularly if c is negative.
Dividing the whole top line by 2a, not just the square root.
Example: Solve 2x2 + 3x 4 = 0.
First, identify a, b and c (a = 2, b = 3, c = 4.

(3) (3)2 4(2)(4)
Substitute into the formula x =
2(2)

(3) 41 3 6.403
Now evaluate the square root x = =
2(2) 4
3 + 6.403 +3.403
So, the solutions are x = = = 0.8508 or
4 4
3 6.403 9.403
= = 2.3508
4 4
Answers are normally given to 2 decimal places, so x = 0.85 or 2.35.
Repeat with 3x2 4x 2 = 0 (1.72, 0.39).
Repeat with x2 4x 1 = 0, but this time leave the answer in surd form.

4 20
The answers are or 2 5 .
2
The class can now do Exercise 16D from Pupil Book 3 or Exercise 10N
(page 259) from the Higher Mathematics for GCSE textbook.

182 HarperCollinsPublishers Ltd 2003


9B3LP_16.qxd 18/09/2003 15:22 Page 183

Exercise 16D Answers

1 a x = 4 or 6 b x = 1 or 7 c x = 2 or 12 d x = 212 or 4 e x = 0 or 5
f x = 6 or 6
2 a x = 1.14 or 2.64 b x = 1.59 or 1.26 c x = 1.32 or 5.32
d x = 2.22 or 0.22 e x = 0.16 or 6.16 f x = 0.19 or 2.69
g x = 1.82 or 0.18 h x = 2.19 or 0.69 i x = 4.56 or 0.44

(3 5 ) 4 20 4 12
3 a b or 2 5 c or 2 3
2 2 2

6 32 10 92 2 8
d or 3 8 e or 5 23 f or 1 2
2 2 2

Plenary Key Words


Ask students to solve 2x2 + 3x + 5 = 0 using the quadratic formula.
quadratic
This example could be worked through on the board by a student.
The formula gives formula
coefficients
3 31
x = which cannot be solved due to the square root.
4
Explain that when a quadratic equation is solved, the roots are where the graph
crosses the x-axis.
This gives three situations

2 roots 1 (repeated) root No roots

This will be useful for the investigation that ends this chapter.
Homework

1 Solve these equations using the quadratic formula. All answers are whole numbers or fractions.
a x2 + 4x 5 = 0 b 2x2 + 5x 3 = 0 c 6x2 19x + 10 = 0
2 Solve these equations, giving your answers to 2 decimal places.
a x2 + 7x 10 = 0 b 2x2 x 4 = 0 c 4x2 + x 7 = 0
3 Solve these equations, giving your answer in surd form.
a x2 4x 2 = 0 b x2 + 6x 1 = 0 c x2 + 5x 2 = 0

Answers
1 a 1 or 5 b 12 or 3 c 23 or 212
2 a 1.22 or 8.22 b 1.69 or 1.19 c 1.20 or 1.45

4 24 6 40 5 33
3 a or 2 6 b or 3 10 c
2 2 2

HarperCollinsPublishers Ltd 2003 183


9B3LP_16.qxd 18/09/2003 15:22 Page 184

LESSON Framework objectives Reinforcement of Number


Solving quadratics by completing the square.
16.5
Oral and mental starter
Ask students to fill in the missing number in problems such as 22 9 = ?
(Answer 13).
Repeat with other examples such as 32 + 5 (answer 4), 42 1 (answer 17).
Make sure that students understand that 32 = (3 squared) and not (3) squared.
Repeat with more examples if necessary.

Main lesson activity


This is a lesson on completing the square.
Ask students to expand (x a)2 and (x + a)2.
Hopefully they will get the answer x2 2ax + a2 and x2 + 2ax + a2.
Now ask if they can fill in the missing numbers in x2 + 4x 3 = (x + )2 .
Eventually they will probably come up with the answer (x + 2)2 7.
Discuss how this relates to the identities established at the start of the lesson.
Work through the following examples on completing the square.
Example: Write x2 + 6x in the form (x + a)2 b.
The value, a, inside the bracket is half the coefficient of x. The value b is the
square of this, So, x2 + 6x = (x + 3)2 9.
Repeat with x2 + 12x ((x + 6)2 36), x2 4x ((x 2)2 4) and
x2 + 30x ((x + 15)2 225).
Example: Write x2 + 6x 7 in the form (x + a)2 b.
The value of a is half the coefficient of x. The value b is the sum of the existing
constant term and the square as before.
So, x2 + 6x 7 = (x + 3)2 9 7 = (x + 3)2 16.
Repeat with x2 + 12x + 15 ((x + 6)2 21), x2 4x 3 ((x 2)2 7) and
x2 + 30x + 100 ((x + 15)2 125).
Demonstrate how to solve equations using completing the square.
Example: Solve x2 + 6x 4 = 0.

x2 + 6x 4 = 0 (x + 3)2 13 = 0 (x + 3)2 = 13 x + 3 = 13

x = 3 13.

Repeat with x2 6x 2 = 0 (x = 3 11 ) and x2 + 10x + 15 = 0 (x = 5 10 ).
Complete more examples if necessary.
The class can now do Exercise 16E from Pupil Book 3 or Exercise 10P
(page 260) from the Higher Mathematics for GCSE textbook.

184 HarperCollinsPublishers Ltd 2003


9B3LP_16.qxd 18/09/2003 15:22 Page 185

Exercise 16E Answers

1 a (x + 4)2 16 b (x 1)2 1 c (x 6)2 36 d (x 7)2 49 e (x + 2)2 4


f (x + 1)2 1
2 a (x + 4)2 17 b (x 1)2 + 2 c (x 6)2 31 d (x 7)2 42 e (x + 2)2 7
f (x + 1)2 6 g (x + 3)2 11 h (x + 5)2 34 i (x 3)2 6

3 a x = 4 17 b x = 3 or 1 c x = 6 31 d x = 7 42 e x = 5 or 1

f x = 3 11 g 5 24 h 3 5 i 4 11 j x = 1 2 k x = 1 6

l x = 6 43

Plenary Key Words


Ask students to solve x2 + 3x 2 = 0 using the completing the square method.

completing the
A student could be asked to come to the board to give the solution.
The odd coefficient of x will cause problems, particularly when taking a square square
root coefficient

i.e. x2 + 3x 2 = 0 (x + 112 )2 414 = 0 (x + 112 )2 = 414 x + 112 = 414

x = 112 414 .
Now ask students to solve 2x2 + 4x 3 = 0 using the completing the square
method.
2x2 + 4x 3 = 0 2(x2 + 2x) 3 = 0 2[(x + 1)2 1] 3 = 0
2(x + 1)2 2 3 = 0

2(x + 1)2 = 5, (x + 1)2 = 212 x + 1 = 212 x = 1 212 .
Homework

1 Complete the square for the following.


a x2 + 12x b x2 6x c x2 20x
2 Rewrite the following quadratic expressions by completing the square.
a x2 + 12x 9 b x2 6x + 3 c x2 20x + 100
3 Solve the following quadratic equations using the completing the square method.
a x2 + 12x 9 = 0 b x2 6x + 3 = 0 c x2 20x + 100 = 0
d x2 10x + 5 = 0 e x2 + 4x 7 = 0 f x2 8x 5 = 0

Answers
1 a (x + 6)2 36 b (x 3)2 9 c (x 10)2 100
2 a (x + 6)2 45 2
b (x 3) 6 c (x 10)
2

3 a x = 6 45 b x = 3 6 c x = 10 d x = 5 20 e x = 2 11 f x = 4 21

HarperCollinsPublishers Ltd 2003 185


9B3LP_16.qxd 18/09/2003 15:22 Page 186

LESSON Framework objectives Reinforcement of Number


Difference of two squares.
16.6
Oral and mental starter
Ask students to find the square root of 81.
They will obviously give the answer of 9, (answers can be written on mini white
boards) but may not give 9. Remind them that both answers are possible.
Now ask students for the square root of x2. They will, hopefully, give x as the
answer.
Now ask for the square root of 4x2. Some will give the answer 4x. Make sure
that they understand the answer is 2x.
Repeat with the square roots of 9y2, 16z2, 25x2, 81x2, 100y2 etc.

Main lesson activity


This is a lesson on factorising the quadratic the difference of two squares.
Start by asking students to expand expressions of the type (x + 2)(x 2) = (x2 4)
e.g. (x 4)(x + 4) = (x2 16), (2x 1)(2x + 1) = (4x2 1), (3x 6)(3x + 6) = (9x2 36) etc.
Eventually ask for the expansion of (a b)(a + b).
Discuss with students the similarities between the results.
Establish the result (a b)(a + b) = a2 b2.
Explain that this is known as the difference of two squares.
Now ask if students can reverse the result. i.e. can they factorise x2 36?
Explain that this can be written down as x2 62 = (x 6)(x + 6).
Emphasise that it is important to identify both squares since failure to do this can
lead to errors such as (x 36)(x + 36).
Repeat with x2 100 ((x 10)(x + 10)), 4x2 9 ((2x 3)(2x + 3)), 9x2 16y2
((3x 4y)(3x + 4y)).
Do more examples if necessary.
The class can now do Exercise 16F from Pupil Book 3 or Exercise 10J
(page 255) from the Higher Mathematics for GCSE textbook.

186 HarperCollinsPublishers Ltd 2003


9B3LP_16.qxd 18/09/2003 15:22 Page 187

Exercise 16F Answers

1 a x2 1 b x2 25 c x2 y2 d 4x2 1 e x2 4y2 f 4x2 9y2


2 a (x 10)(x + 10) b (x 2)(x + 2) c (x 6)(x + 6) d (x 9)(x + 9)
e (x 8)(x + 8) f (x 11)(x + 11) g (x z)(x + z) h (2x 5)(2x + 5)
i (x 3y)(x + 3y) j (4x 3)(4x + 3) k (2x 5y)(2x + 5y) l (5x 8)(5x + 8)
m (3 x)(3 + x) n (2x 6)(2x + 6) p (6x 1)(6x + 1)

Plenary Key Words


Ask the students to simplify (x + 2)2 (x 4)2.

difference of two
If possible, ask a student who wants to expand brackets to work through the
problem on the board, i.e. (x + 2)(x + 2) (x 4)(x 4) = (x2 + 4x + 4) squares
(x2 8x + 16) = x2 + 4x + 4 x2 + 8x 16 = 12x 12.
Now, ask if the original expression could be written in any other way. Some
students may recognise this as the difference of two squares.
Make sure that the students understand that the difference of two squares is an
identity, i.e. any values or expressions can be used for a and b in
a2 b2 = (a b)(a + b).
Using brackets (because of the minus signs), write the above as:
((x + 2) (x 4))((x + 2) + (x 4))
= (x + 2 x + 4)(x + 2 + x 4)
= (6)(2x 2)
= 12x 12.
Discuss the advantages of both methods.
Homework

1 Expand these brackets into quadratic expressions.


a (x + 11)(x 11) b (2x 3)(2x + 3) c (5x 2y)(5x + 2y)
2 Factorise the following quadratic expressions.
a x2 144 b x2 225 c 4x2 36
d 81x2 64 e x2 4y2 f 16x2 121
g x2 9z2 h 4x2 25y2 i 81x2 16y2

Answers
1 a x2 121 b 4x2 9 c 25x2 4y2
2 a (x 12)(x + 12) b (x 15)(x + 15) c (2x 6)(2x + 6) d (9x 8)(9x + 8) e (x 2y)(x + 2y)
f (4x 11)(4x + 11) g (x 3z)(x + 3z) h (2x 5y)(2x + 5y) i (9x 4y)(9x + 4y)

HarperCollinsPublishers Ltd 2003 187


9B3LP_16.qxd 18/09/2003 15:22 Page 188

LESSON Framework objectives Reinforcement of Number


An investigation into y = ax2 + bx + c.
16.7
Oral and mental starter
There is no starter as the investigation will need to be introduced via a class discussion.

Main lesson activity


The investigation in the Pupil Book is reproduced here. Students will need access to graphical calculators or
computers with a graph-drawing application package. The investigation will take more than one lesson.

The graph of a quadratic equation has a characteristic


shape called a parabola. y

Investigate the relationship between the values a, b and


c in the graph y = ax2 + bx + c and the points P, Q
(where the graph crosses the x-axis), the point R (where
the graph crosses the y-axis) and the point S (the vertex
or turning point of the graph).
y = ax2 + bx + c
You should use a graph plotting program or a graphical
calculator to help you.
You will not gain any credit for spending a lot of time
drawing graphs accurately.
P Q x
You will gain credit for a systematic investigation into
the effect of a, b and c on the graph.
R
S

Introduce the investigation and ask the students to suggest ideas for getting started.

For example, they could start by looking at graphs of y = x2, y = 2x2, y = 3x2 etc.
Then look at graphs of y = x2 + 1, y = x2 2, y = x2 + 3 etc.
Then look at graphs of y = x2 + 2x, x2 3x, x2 + 5x etc.
Once they have an idea of the effect of a, b and c individually they could then look at more general equations.
Students should be encouraged to look at the roots from the quadratic formula or the completing the square
method. The vertex also could be investigated via the completing the square method.

188 HarperCollinsPublishers Ltd 2003


9B3LP_16.qxd 18/09/2003 15:22 Page 189

GCSE Answers

1 0 and 5
2 a (x 4)(x 2) b x = 4, x = 2
3 a i 3( pq 2r) ii (c 4)(c 5)
b x = 7 or x = 2
p5
4 a (3p + 1)( p + 5) b
3p + 1
5 5.74, 1.74
6 a 9x2y6 b 3.14, 0.64
7 1.28, 0.78
8 a = 9 b = 2 c = 5
9 a = 5 b = 7
10 p = 2 q = 4

Plenary Key Words


There is no plenary. Students could be stopped working at various points and a
roots
general discussion on progress could be held. This can be helpful for students who
are having difficulty but avoid the bush fire effect of one student giving the rest of factors
the group the answer. vertex
intersect
coefficient
Homework

Students could be asked to continue the work at home if they have computer facilities or asked to look
on the Internet for information on the quadratic equation.

HarperCollinsPublishers Ltd 2003 189


9B3LP_16.qxd 18/09/2003 15:22 Page 190

Published by HarperCollinsPublishers Limited


7785 Fulham Palace Road
Hammersmith
London
W6 8JB

www.CollinsEducation.com
Online support for schools and colleges

HarperCollinsPublishers Ltd 2003

10 9 8 7 6 5 4 3 2 1

ISBN 0 00 713881 4

Keith Gordon, Kevin Evans, Trevor Senior and Brian Speed assert their moral rights to be identified as the authors
of this work.

Any educational institution that has purchased one copy of this publication may make duplicate copies for use
exclusively within that institution. Permission does not extend to reproduction, storage in a retrieval system, or
transmittal, in any form or means, electronic, mechanical, photocopying, recording or otherwise, of duplicate
copies for loaning, renting or selling to any other institution without the prior consent in writing of the Publisher.

British Library Cataloguing in Publication Data


A Catalogue record for this publication is available from the British Library

Edited by John Day


Typesetting and design by Gray Publishing
Project management by Nicola Tidman
Covers by Tim Byrne
Illustrations by Gray Publishing
Proofreading by Amanda Whyte and Jenny Wong
CD mastering by Alan Trewartha
Production by Sarah Robinson
Printed and bound by Martins the Printers, Berwick upon Tweed

The publishers would like to thank the many teachers and advisers whose
feedback helped to shape Maths Frameworking.

Every effort has been made to trace copyright holders and to obtain their permission for the use of copyright
material. The author and publishers will gladly receive any information enabling them to rectify any error or
omission in subsequent editions.

You might also like to visit:


www.harpercollins.co.uk
The book lovers website

190 HarperCollinsPublishers Ltd 2003


Maths Frameworking Year 9 Teachers Pack 3 Practice Paper Answers ISBN 0 00 713881 4

PRACTICE PAPER ANSWERS AND TUTORIAL

Question 1

Part Mark Answer Tutorial


(a) 3 8 grey cubes You can count cubes of each type that are showing.
12 white cubes You can still gain 1 mark if you get three of the four
6 dotted cubes numbers right, or get the right numbers in the wrong order.
1 striped cube
(b) 2 8 grey cubes You might need to make a drawing of a 4 4 4 cube.
24 white cubes You have to realise that there are 64 cubes altogether.
24 dotted cubes You can still gain 1 mark if you get three of the four
8 striped cubes numbers right, or get the right numbers in the wrong order.
Total 64 cubes

Total 5

Question 2

Part Mark Answer Tutorial


(a) 2 For full marks you have This question is testing whether you can understand a
to show something like: rule and carry it out, so your answer has to follow the same
10% of 180 = 18 method as the example.
20% of 180 = 36 You will be given some credit for using a different correct
5% of 180 = 9 method such as the method shown in part (b):
25% of 180 is 50% of 180 is 90
36 + 9 = 45 25% of 180 = 45
(b) 2 For full marks you have Again this question is testing whether you can
to show something like: understand a rule and carry it out, so your answer has to
50% of 460 = 230 follow the same method as the example.
25% of 460 = 115 You will be given some credit for using a different correct
method such as the method shown in part (a):
10% of 460 = 46
20% of 460 = 92
5% of 460 = 23
25% of 460 is 92 + 23 = 115

Total 4

HarperCollinsPublishers Ltd 2003 1


Maths Frameworking Year 9 Teachers Pack 3 Practice Paper Answers ISBN 0 00 713881 4

Question 3

Part Mark Answer Tutorial


(a) 2 21 packs This is long division. There are a couple of ways of doing
long division. The first is the traditional method:
2 0 r. 10
12) 2 5 0
2 4
1 0
0
1 0
As the remainder is fairly easy to work out you may have
done this as a short division.
2 0 r. 10
12) 2 5 1 0

Another method is the method of repeated subtraction:


2 5 0
10 12 = 1 2 0
1 3 0
10 12 = 1 2 0
1 0
The number of 12s in 250 is 20 remainder 10, which means
that the school will need 21 packs.
An answer of 21 gains 2 marks. If you make one mistake,
or say 20 packs, you will only gain 1 mark.
(b) 2 5500 p This is long multiplication.
1 55 There are several ways of doing this. One way is the grid
method.
or 250
200 50 22
20 4000 1000 5000 500
5000
2 400 100 500
5500
5500

An answer of 5500 gains 2 marks. If you make one


mistake you can still gain 1 mark.
To gain the last mark you need to convert your answer
from pence to pounds. This means dividing by 100. You can
still gain this mark for dividing by 100, even if your
calculation is wrong.

Total 5

2 HarperCollinsPublishers Ltd 2003


Maths Frameworking Year 9 Teachers Pack 3 Practice Paper Answers ISBN 0 00 713881 4

Question 4

Part Mark Answer Tutorial


(a) 1 n+7 There are lots of rules about what you can and cannot do
with algebra. Another acceptable answer is 7 + n, but x + 7
is wrong. You cannot change letters. Neither can you
simplify expressions wrongly. So n + 7 = 7n would be
wrong but c = n + 7 would be allowed as it could be read
as number of cubes = n + 7. It is safer to write down the
expression without using equals signs and not to try to
simplify the answer unless the question says write your
answer as simply as possible or simplify your answer.
(b) 1 4m You can write multiplications such as m 4 and 4 m
either way around and they would be acceptable but do not
write m4 as this is not correct.
There are four cubes in each layer and m layers so the
answer is 4 m or 4m.
You can check your answer using numbers. Suppose the
number of cubes is 20 then m would be 5 as 5 4 = 20.
(c) 1 You should have this Each tower has the same number of cubes, so because
box ticked: there are 4 cubes in each layer in the second tower it only
n=m4 needs to be 14 of the height.
(d) 1 2 You now know that n = 4m and n = m + 6.
You can combine these by writing an equation m + 6 = 4m.
Subtracting m from each side gives 3m = 6 and dividing by
3 gives m = 2.
You could also answer this question by trying different
tower sizes. If m = 2 there will be 8 cubes in tower M, so
the other tower is 8 cubes high, which is 2 + 6 as required.

Total 4

Question 5

Part Mark Answer Tutorial


(a) 2 59 1 goal was scored in 14 games = 14 goals
2 goals were scored in 8 games = 16 goals
3 goals were scored in 7 games = 21 goals
4 goals were scored in 2 games = 8 goals
14 + 16 + 21 + 8 = 59 goals altogether
(b) 2 Team B 20% of 50 games is 10 games.
You can work this out by saying 10% of 50 = 5 and
doubling it.
Team B scored 3 or more goals in 10 matches (5 + 2 + 3).
(c) 1 Any valid reason: Many different reasons could be acceptable such as;
It does not show how many games each team won;
It does not show how many goals were conceded;
Successful might mean different things to different people.

Total 5

HarperCollinsPublishers Ltd 2003 3


Maths Frameworking Year 9 Teachers Pack 3 Practice Paper Answers ISBN 0 00 713881 4

Question 6

Part Mark Answer Tutorial


3 A = Graph 4 Graph 1 shows a runner going at a steady speed as the
B = Graph 1 graph is a straight line.
C = Graph 2 Graph 2 covers distance very quickly at the beginning and
then less quickly at the end so this runner is getting slower.
Graph 3 covers distance very slowly at the beginning and
then more quickly at the end so this runner is getting faster.
Graph 4 covers distance very quickly at the beginning, then
less quickly in the middle and then more quickly at the end.
Graph 5 covers distance very slowly at the beginning, then
more quickly in the middle and then less quickly at the end.
You would gain 1 mark for each correct answer.
Total 3

Question 7

Part Mark Answer Tutorial


3 LINE B y = 5 Lines that are drawn horizontally, parallel to the x-axis,
LINE C x = 5 are of the form y = a, where a is the value where the line
LINE D x + y = 5 crosses the y-axis.
Lines that are drawn vertically, parallel to the y-axis, are
of the form x = b, where b is the value where the line
crosses the x-axis.
Line B passes through the point 5 on the y-axis and so
has the equation y = 5. You would gain no marks if you
wrote y is 5.
Line C passes through the point 5 on the x-axis and so
has the equation x = 5. You would gain no marks if you
wrote x is 5.
Line D passes through points where the coordinates add
up to 5 (x + y = 5) such as (0, 5), (1, 4), (2, 3), (5, 0) so the
equation is x + y = 5.
There is one mark for each correct answer.

Total 3

Question 8

Part Mark Answer Tutorial


(a) 1 4 Some formulae for the area of 2-D shapes that you need
to know will be given at the front of the test papers. Others
you need to learn. The formula for the area of a
parallelogram is A = bh.
If A = 20 and h = 5, then 20 = b 5 and so b = 4.
(b) 1 4 The formula for the area of a triangle is:
bh
A =
2
10 h
If A = 20 and b = 10, then 20 = and so h = 4.
2

4 HarperCollinsPublishers Ltd 2003


Maths Frameworking Year 9 Teachers Pack 3 Practice Paper Answers ISBN 0 00 713881 4

(c) 1 h = 10 The formula for the area of a trapezium is:


(a + b)h
A =
2
If A = 20, then (a + b) h = 40. You need to find a number
which when multiplied by 4 gives 40, so h = 10.
1 a+b=8 If A = 20, then (a + b) h = 40. So (a + b) 5 = 40,
so a + b = 8.
(d) 2 x=3 Find x first by noticing that the two expressions for the
Length = 10 cm length of the rectangle must be equal. This gives the
Width = 2 cm equation:
Perimeter = 24 cm 3x + 1 = 5x 5 (Take 3x from both sides)
1 = 2x 5 (Add 5 to both sides)
6 = 2x (Divide both sides by 2)
x=3
The length of the rectangle is 3x + 1 = 10 or 5x 5 = 10.
The formula for the area of a rectangle is A = lw. If A = 20
and l = 10, then 20 = 10w. This gives w = 2.
You would gain 1 mark if you showed that x = 3 or that
l = 10.
So the perimeter is 10 + 2 + 10 + 2 = 24 cm.

Total 6

Question 9

Part Mark Answer Tutorial


(a) 2 5 10 25 From the three patterns given, notice that the number of
black squares is always 2 times the pattern number,
12 24 144 and the number of white squares is always the pattern
number squared.
You would gain 1 mark if you gave two or three correct
values or if you got the colours the wrong way round.
(b) 2
n 2n n2 The question asks for expressions for the number of
black and white squares, so you must use n in your
answers, as shown in the table. Do not use equations such
as n = n 2, as you would lose marks. You could write
number of black squares = 2n, but try to avoid using
equations when questions ask you to write expressions.
For 2n you could also write 2 n or n + n, and for n2 you
could also write n n.
(c) 1 2n + n2 The total number of squares in pattern number n is 2n
for the black squares + n2 for the white squares which
is written 2n + n2.
(d) 2 3n +n2 The number of black squares in the pattern follows the
sequence 3, 6, 9 increasing by 3 each time.
The total number of squares in pattern number n is 3n
for the black squares + n2 for the white squares which
is written 3n + n2.

Total 7

HarperCollinsPublishers Ltd 2003 5


Maths Frameworking Year 9 Teachers Pack 3 Practice Paper Answers ISBN 0 00 713881 4

Question 10

Part Mark Answer Tutorial


(a) 1 300 If 2.5 30 = 75, then 0.25 300 = 75. Notice that to find
two multiplications that have the same answer, you can
divide one of the numbers by an amount (10 here), and
multiply the other number by the same amount.
You can, of course, do a division sum without a
calculator:
75 750 7500
= = = 300
0.25 2.5 25
1 0.1 or 1
10
If 75 1 = 75, then 7.5 0.1 = 75. Notice that to find
divisions that have the same answer, you can divide one of
the numbers by an amount (10 here), and divide the other
number by the same amount.
(b) 1 a = 18 4a + 3 = 75 (Subtract 3 from both sides)
4a = 72 (Divide both sides by 4)
a = 18
1 b = 50 2b 25 = 75 (Add 25 to both sides)
2b = 100 (Divide both sides by 2)
b = 50
1 c = 9 or c2 62 = 75 (Add 6 to both sides)
c = 9, or both c = 81 (Take the square root of both sides)
c=9

Since 81 is also 9, this answer would be accepted.

Total 5

Question 11

Part Mark Answer Tutorial


2 The completed diagram The line on the diagram that shows the position of the
should look like this: path must be drawn 1 square from the edge of the flower
bed on the sides.
The line at the corners is a quarter-circle, radius 2 cm,
centred at the corner, drawn using compasses.
Some allowance is made for diagrams that are slightly
inaccurate, but try to avoid drawing lines freehand. You are
expected to show that you can use a ruler and a pair of
compasses in the tests.
You would gain 1 mark for an otherwise correct diagram
with the circle arcs wrongly drawn or omitted.

Total 2

6 HarperCollinsPublishers Ltd 2003


Maths Frameworking Year 9 Teachers Pack 3 Practice Paper Answers ISBN 0 00 713881 4

Question 12

Part Mark Answer Tutorial


2 Any correct method to The easiest way to show that the statement is wrong is
show that the statement by substituting numbers for a and b into both sides of the
is wrong. equation and then showing that the answers obtained are
not the same.
For example, let a = 2 and b = 3.
2 3
Then the left-hand side becomes + = 1 + 1 = 2
2 3
(2+3) 5
and the right-hand side becomes = = 1.
5 5
2 is not equal to 1, so the values on both sides are not
equal, and we have found an example to prove the students
is wrong.

Total 2

Question 13

Part Mark Answer Tutorial


(a) 1 20 1
or or 0.2 The number of passengers waiting 15 minutes or longer
100 5 is found by adding the last three frequencies of the bar
chart: 8+ 7 + 5 = 20.
So P (passenger waits for 15 minutes or longer) =
20 1
= .
100 5
Probability is written as a fraction or a decimal. Always
cancel fractions if it is possible, although you would not lose
marks if you did not. You would get no marks, though, for
writing 20 out of 100 or 20 in 100.
(b) 1 42 21
or or 0.42
There were 40 passengers waiting from 5 to 10 minutes,
100 50 so approximately 20 passengers were waiting between 5
and 7.5 minutes. Therefore the number of passengers
waiting 7.5 minutes or less is approximately 22 + 20 = 42.
So P (passenger waits for 7.5 minutes or less) =
42 21
= or 0.42.
100 50
(c) 2 10.15 The last column of the table is given below:
fx
55
300
225
140
157.5
137.5
1015
total waiting time for all passengers
The mean =
total number of passengers
1015
=
100
= 10.15
You would gain 1 mark if your total in the fx column was
wrong but you correctly divided this by 100.

HarperCollinsPublishers Ltd 2003 7


Maths Frameworking Year 9 Teachers Pack 3 Practice Paper Answers ISBN 0 00 713881 4

Part Mark Answer Tutorial


(d) 1 Possible answers are: Make sure that you read the question carefully. You are
decrease the time already told one way of improving the survey, so you need
intervals for recording; to give a different way. Do not give answers that are
do it on different days; irrelevant, such as draw a pie chart or use the mode.
do it on different routes.

Total 5

Question 14

Part Mark Answer Tutorial


(a) 1 50 When p = 5,
2 53
a =
5
2555
= = 2 25 = 50
5
1 10 When p = 5,
556
b =
35
56
=
3
30
=
3
= 10
(b) 1 5c 15c
Notice how the d cancels, leaving = 5c.
3
(c) 1 7x + 6 This question involves multiplying out brackets.
You need to know that a(b + c) = ab + ac and a(b + c) =
ab ac and a(b c) = ab + ac.
Multiplying out the brackets gives 3x + 12 6 + 4x.
Notice the change in sign when multiplying out the second
bracket.
Collecting like terms together gives 7x + 6.
1 x2 + 8x + 15 The next three parts involve expanding two brackets,
so you need to know that:
(a + b) (c + d) = a(c + d) + b(c + d)
= ac + ad + bc + bd
(x + 3) (x + 5) = x2 + 5x + 3x + 15
= x2 + 8x + 15
1 x2 x 2 (x + 1) (x 2) = x22 2x + x 2
=x x2
1 x2 8x + 16 (x 4)2 = (x2 4)(x 4)
= x 4x 4x + 16
= x2 8x + 16

Total 7

8 HarperCollinsPublishers Ltd 2003


Maths Frameworking Year 9 Teachers Pack 3 Practice Paper Answers ISBN 0 00 713881 4

Question 15

Part Mark Answer Tutorial


(a) 1 Your graph should y = x33 + 2, so the y-value is 2 more than at each point
look like this: on y = x . This makes the curve translate 2 units in the
y positive y direction.

y = x3 + 2
x

(b) 1 y = x3 Curve A is a reflection of y = x3 in the x-axis, so all the y


values will now become negative.
(c) 1 Reflection in the y axis. All the points on the graph can also re-plotted with the
sign of the x values negative so the graph has been reflected
in the y-axis.
(d) 2 y < 2x and The shaded region is below the line y = 2x, so y < 2x is
y > x3 one inequality that describes the region. The shaded region
is also above the curve y = x3, so y > x3 is the other
inequality that describes the region.

You will gain 1 mark for each correct inequality. You


would get no marks if you ringed y < 2x and y > 2x or
y < x3 and y > x3, as these are conflicting inequalities.

Total 5

Question 16

Part Mark Answer Tutorial


(a) 1 3 104 is greater than This part4is about using numbers written in standard
4 103. form. 3 10 is a shorter way of writing 3 10 000 = 30 000
3 104 = 30 000 and 4 103 is a shorter way of writing 4 1000 = 4000.
4 104 = 4000 For example 4 000 0006 can be written as 4 1 000 000 or,
in standard form 4 10 .
(b) 2 500 and 0.5 103 5 102 = 5 100 = 500. 3
This is the same as 0.5 10 or 0.5 1000.
(c) 1 80000 or 8 104 This part uses the fact
10a 10b = 10a+b
(4 105) (2 101) = 8 1051 = 8 104
= 8 10 000 = 80 000
1 200 000 000 or 2 108 10a
This part uses the fact
10b
= 10ab
8 1010
= 2 108 or 200 000 000
4 102

Total 5

HarperCollinsPublishers Ltd 2003 9


Maths Frameworking Year 9 Teachers Pack 3 Practice Paper Answers ISBN 0 00 713881 4

Question 17

Part Mark Answer Tutorial


(a) 1 Number Number Number There are 4 shapes with only straight edges.
of of of
shapes shapes shapes
with with with
straight curved both
edges edges straight There are 3 shapes with only curved edges.
only only edges
and
curved
edges
The are 3 shapes with both straight and curved edges.
4 3 3

(b) 1 7

10 There are 7 shapes which do not have both straight and
curved edges (4 + 3) or (10 3) as there are 3 shapes with
both straight and curved edges.
Therefore P (a shape does not have both straight and
curved edges) = 107.
(c) 1 12 There are 6 shapes altogether which have curved edges
and 3 of them also have straight edges.
Therefore P(shape has straight edges given that it has
curved edges) = 36 = 12.

10 9 For this question you need to know that for two events A
6
(d) 1 5
and B:
P(A and B) = P(A) P(B)
provided A and B are independent events (i.e. an outcome
from one event does not determine an outcome of the other
event).
P(first shape has a curved edge) = 106
If another shape is chosen this leaves 9 shapes to choose
from, of which 5 have curved edges.
So, P(second shape has curved edges) = 59.
P(both shapes have curved edges) = 106 59.

Total 3

10 HarperCollinsPublishers Ltd 2003


Maths Frameworking Year 9 Teachers Pack 3 Practice Paper Answers ISBN 0 00 713881 4

Question 18

Part Mark Answer Tutorial


(a) 3 9a2 4a2 = 5a2 This is an example of an unstructured question. This
means that you have to do more than one step to get the
answer. It usually involves showing a fair amount of
working. The formula for the area of a circle is A = r 2.
Now use this formula to calculate the area of each separate
circle.
For the large circle, r = 3a.
So, A = r 2 = (3a)2 = 3a 3a = 9a2
For the small circle r = 2a.
So A = r 2 = (2a)2 = 2a 2a = 4a2
The shaded area is given by 9a2 4a2 = 5a2
Remember to take away the area of the small circle.
You would gain 2 marks if you found the area of both
circles but did not obtain the final answer.
You would gain one mark if you only found the area of
one circle.
(b) 2 2
a =

5a2 = 20 (Divide both sides by 5)
20 4
a2 = = (take the square root of both sides)
5


4 2
a = =


You would only gain 1 mark if you tried to substitute
3.142 for , since the question asks for the answer in terms
of .

Total 5

HarperCollinsPublishers Ltd 2003 11


CHAPTER
16 GCSE Preparation
LESSON Framework objectives Reinforcement of Number
Solving quadratic equations.
16.1

Oral and mental starter


Ask students to mentally solve x 1 = 0. They should find that x = 1 quickly
(answers can be written on mini white boards).
Repeat with x + 4 = 0 (4), x 3 = 0 (3), x + 3 = 0 (3).
Now ask students to give two values for a and b that solve a b = 0. Ask
students to write their answers down on mini white boards as a = ? and b = ?.
Check answers. Some could be written on the board. Discuss the common
characteristics, which should be that either a or b should be zero in each case.
Now ask students to write down a value for x that will make (x 2)(x + 4) = 0.
Once again check answers. Hopefully they will be either x = 2 or x = 4.

Main lesson activity


This is a lesson on solving quadratics that factorise.
Ask students if they can find a value for x that solves the quadratic equation
x2 + 6x 7 = 0.
They may spot the answer x = 1, but are unlikely to spot the answer of x = 7.
Outline the method. First, factorise and then solve each bracket equal to zero.
i.e. x2 + 6x 7 = 0 (x 1)(x + 7) = 0.
So, either x 1 = 0 x = 1, or x + 7 = 0 x = 7.
Repeat with other examples such as:
x2 x 6 = 0 (x = 3 or 2), x2 + 4x + 3 = 0 (x = 1 or 3),
x2 8x + 15 = 0 (x = 3 or 5).
Do more examples if necessary.
The class can now do Exercise 16A from Pupil Book 3 or Exercise 10K
(page 256) from the Higher Mathematics for GCSE textbook.

176 HarperCollinsPublishers Ltd 2003


Exercise 16A Answers

1 a x = 1 or 1 b x = 2 or 5 c x = 3 or 6 d x = 4 or 3 e x = 2 or 7
f x = 3 or 8 g x = 8 or 1 h x = 3 i x = 4
2 a x = 1 or 2 b x = 5 or 6 c x = 2 or 4 d x = 3 or 2 e x = 5 or 2
f x = 1 or 4 g x = 5 h x = 4 i x = 5 or 3 j x = 3 or 5 k x = 6 or 4
l x = 3 or 2 m x = 9 or 1 n x = 6 or 3 p x = 1

Plenary Key Words


Ask the students if they can solve 2x2 3x + 1 = 0.

quadratic
They may spot x = 1 as a solution. Give a clue that the other answer is a fraction
between 0 and 1. equations
They should find 12 fairly quickly. unitary
Discuss ways that this could be solved mathematically. coefficient
The students may establish that the factorisation is (x 1)(2x 1) = 0 and that
these brackets solve to 1 and 12.
Do more examples with a non-unit coefficient of x2, if time allows, such as
2x2 5x + 3 = 0.
Homework

1 Solve these equations.


a (x + 3)(x 4) = 0 b (x 1)(x + 6) = 0 c (x 7)(x + 6) = 0
d (x + 5)(x + 2) = 0 e (x 3)(x + 6) = 0 f (x 9)(x 3) = 0
2 First factorise, then solve these equations.
a x2 + 8x + 15 = 0 b x2 + 13x + 30 = 0 c x2 + 4x 5 = 0
d x2 9x + 14 = 0 e x2 + 4x 21 = 0 f x2 4x + 4 = 0

Answers
1 a x = 3 or 4 b x = 1 or 6 c x = 7 or 6 d x = 5 or 2 e x = 3 or 6 f x = 9 or 3
2 a x = 5 or 3 b x = 3 or 10 c x = 1 or 5 d x = 2 or 7 e x = 3 or 7 f x = 2

HarperCollinsPublishers Ltd 2003 177


LESSON Framework objectives Reinforcement of Number
Factorisation of quadratics of the form ax2 + bx + c, where a > 1.
16.2
Oral and mental starter
Give students the factors of 12 in pairs and the factors of 15 on each side of a
vertical line, i.e.
1 2 3 1 3
12 6 4 15 5
Ask the students to find a combination of products using one pair of factors from
each side that give a total of 28. This first exemplar may need to be
demonstrated i.e.
2 3 2 5 + 6 3 = 28.
6 5
Repeat with totals of 29(3 3 + 4 5), 27(3 5 + 4 3 or 1 15 + 12 1),
41(1 5 + 12 3).
Now ask for the total 8. Suggest to the students that negative values could be
used. The answer is
2 3 2 5 + 6 3 = 8.
6 5
Repeat with totals of 24(2 15 + 6 1), 3(3 5 + 4 3).
Do more examples if time available.

Main lesson activity


This is a lesson on factorising quadratics with a non-unit coefficient for x2.
Students have met the idea in previous plenaries, but a method for solution has
not been outlined.
Ask the students to factorise 2x2 + 9x + 4.
They may have an intuitive idea of the answer but they also need to have a
method outlined.
There are several methods, two of which are outlined in the Pupil Book.
Example: Find factors of the x2 coefficient and of the constant term.
For 2x2 + 9x + 4, the factors of 2 are 1 2 and the factors of 4 are 1 4.
Now find a combination that gives the coefficient of x, e.g.
1 1 11+24=9
2 4
The brackets are then the opposite to the pairs i.e. (x + 4)(2x + 1).
Example: Factorise 6x2 17x + 12.
The brackets must start (3x )(2x ) or (6x )(x ) and the constant term has
factors 3 4, 2 6, 1 12.
By trial and improvement we can find that the combination (3x 4)(2x 3) works.
Do more examples if necessary.
The class can now do Exercise 16B from Pupil Book 3 or Exercise 10L
(page 257) from the Higher Mathematics for GCSE textbook.

178 HarperCollinsPublishers Ltd 2003


Exercise 16B Answers

1 a 2x2 + 11x + 5 b 3x2 + 9x 12 c 4x2 18x + 20 d 6x2 15x 21


e 4x2 + 24x + 36 f 9x2 24x + 16 g 6x2 22x 8 h 8x2 + 10x 3 i 4x2 1
2 a (2x + 1)(x + 3) b (x + 2)(2x + 5) c (x + 4)(3x + 1) d (x 1)(2x + 1)
e (2x + 1)(3x + 2) f (x 2)(2x + 3) g (x + 3)(2x 3) h (2x + 1)2
i (4x 1)(x + 2) j (5x + 1)(x + 2) k (3x 1)(x + 1) l (4x + 1)(2x + 1)
m (x 2)(3x + 1) n (2x + 1)(3x 1) p (4x + 1)(x 3) q (2x 3)(2x + 5)
r (x 7)(2x + 5) s (x 5)(2x + 5) t (3x 1)(x + 5) u (3x + 1)2
v (2x + 3)(5x 1)

Plenary Key Words


Ask the students if they can solve the quadratic equation 2x2 3x 2 = 0.

quadratic
They should be able to put all previous ideas together to explain the process of
factorisation and solving each bracket. equations
e.g. (2x + 1)(x 2) = 0 giving x = 12 or x = 2 non-unitary
Repeat with other examples, if time allows, such as 3x2 5x 2 = 0. coefficients
Homework

1 Expand these brackets into quadratic expressions.


a (3x + 1)(x 4) b (3x 1)(x + 5) c (2x 1)(2x + 3)
d (3x 2)(3x + 2) e (3x 1)2 f (2x + 5)2
2 Factorise the following quadratic expressions.
a 2x2 7x 4 b 2x2 + 13x + 15 c 3x2 + 5x 2
d 4x2 + 23x 6 e 6x2 5x + 1 f 6x2 + 11x + 3
g 5x2 26x + 5 h 6x2 5x 6 i 4x2 16x + 15

Answers
1 a 3x2 11x 4 b 3x2 + 14x 5 c 4x2 + 4x 3 d 9x2 4 e 9x2 6x + 1 f 4x2 + 20x + 25
2 a (2x + 1)(x 4) b (2x + 3)(x + 5) c (3x 1)(x + 2) d (4x 1)(x + 6) e (3x 1)(2x 1)
f (3x + 1)(2x + 3) g (x 5)(5x 1) h (3x + 2)(2x 3) i (2x 3)(2x 5)

HarperCollinsPublishers Ltd 2003 179


LESSON Framework objectives Reinforcement of Number
Solving quadratic equations of the form ax2 + bx + c = 0, where a > 1.
16.3
Oral and mental starter
Ask students to mentally solve 2x 1 = 0. They should find that x = 12. Answers
can be written on mini white boards.
Repeat with 3x + 4 = 0 (113), 4x 3 = 0 ( 34 ), 2x + 3 = 0 (112 ), 5x 1 = 0 ( 15 ),
3x 2 = 0 ( 23 ).
Do more examples if necessary.
Now ask students to write down a value for x that will make (3x 2)(4x + 1) = 0.
Once again check answers. They should be x = 23 or x = 14 .

Main lesson activity


This is a lesson on solving quadratic equations with non-unitary coefficients.
Ask students if they can find a value for x that solves the quadratic equation
2x2 x 1 = 0.
They may spot the answer x = 1, but are unlikely to spot the answer of 12 .
Outline the method. First factorise and then solve each bracket equal to zero.
i.e. 2x2 x 1 = 0 (2x + 1)(x 1) = 0, so either x 1 = 0 x = 1 or
2x + 1 = 0 x = 12 .
Repeat with other examples such as 3x2 + 2x 1 = 0 (x = 13 or 1),
12x2 + 5x 2 = 0 ( x = 23 or 14 ), 8x2 14x 15 = 0 (x = 34 or 212 ).
Do more examples if necessary.
Ask students to solve the quadratic equation x2 + x = 12.
This is not in the correct form to factorise and solve. Encourage students to
always rearrange quadratic equations into the form ax2 + bx + c = 0.
The equation above then becomes x2 + x 12 = 0, which can be factorised and
solved to give x =3 or 4.
Repeat with 2x(x 7) = 6 3x. This must be expanded and then collected into
the correct form.
2x2 14x = 6 3x 2x2 11x 6 = 0 (2x + 1)(x 6) = 0 x = 12 or 6.
Do more examples if necessary.
The class can now do Exercise 16C from Pupil Book 3 or Exercise 10M
(page 258) from the Higher Mathematics for GCSE textbook.

180 HarperCollinsPublishers Ltd 2003


Exercise 16C Answers

1 a x = 12 or 3 b x = 2 or 13 c x = 4 or 12 d x = 112 or 12 e x = 6 or 12
f x = 2 or 14 g x = 212 or 13 h x = 13 or 12 i x = 112 j x = 14 or 15
k x = 212 or 23 l x = 13 m x = 12 n x = 56 or 1 p x = 123 or 2
2 a x = 2 or 1 b x = 1 or 1 c x = 12 or 1 d x = 23 or 1 e x = 14 or 2
f x = 12 g x = 5 or 5 h x = 12 or 113 i x = 14 or 15

Plenary Key Words


Write the following equation on the board x4 5x2 + 4 = 0.

quadratic
Explain that the equation has 4 solutions since the highest power of x is 4.
Ask students if they can spot any of the solutions. They may spot 1 and 2 but equations
may not spot 1 and 2. non-unitary
Outline the method of solution ( x2 1)( x2 4) = 0. coefficients
So, x2 1 = 0 x2 = 1 x = 1 or x2 4 = 0 x2 = 4 x = 2. roots
Repeat with 4x4 37x2 + 9 = 0.
(4x2 1)(x2 9) = 0 x = 12 or 3.
Homework

1 Solve these equations.


a 2x2 15x + 7 = 0 b 3x2 5x + 2 = 0 c 2x2 9x 5 = 0
d 24x2 + 14x 5 = 0 e 6x2 + 23x + 20 = 0 f 6x2 23x + 7 = 0
2 Solve these equations
a x2 + x = 6 b 2x(x + 4) = 3(x 1) c 8x2 3x + 4 = 2x2 + 2x + 3

Answers
1 a 7 or 12 b 23 or 1 c 12 or 5 d 14 or 56 e 212 or 113 f 312 or 13
2 a 2 or 3 b 3 or 12 c 13 or 12

HarperCollinsPublishers Ltd 2003 181


LESSON Framework objectives Reinforcement of Number
The quadratic formula.
16.4
Oral and mental starter

4 + 41
(
4 + 6.5 2.5
Ask students to estimate the value of = = 1.25 .
2 2 2 )

3 52 5 + 31 1 13
Repeat with ( 2.5), ( 0.25), ( 1.1),
4 2 4

4 + 21
( 0.25).
2

5 60
Repeat with (two answers required) ( 1.25 or 6.25),
2

1 17
( 0.75 or 1.25).
4

Main lesson activity


This is a lesson that introduces the quadratic formula.
Introduce the formula:

b b2 4ac
x =
2a
and explain that a, b and c are the coefficients of x2, x and the constant term in
the general quadratic equation ax2 + bx + c = 0.
Work through some examples but encourage students to follow some basic
principles:
Substitute values in brackets before attempting to work anything out.
Take particular care with: The change of sign of b.
Squaring b inside the root, particularly if it is negative
Working out 4 a c correctly particularly if c is negative.
Dividing the whole top line by 2a, not just the square root.
Example: Solve 2x2 + 3x 4 = 0.
First, identify a, b and c (a = 2, b = 3, c = 4.

(3) (3)2 4(2)(4)
Substitute into the formula x =
2(2)

(3) 41 3 6.403
Now evaluate the square root x = =
2(2) 4
3 + 6.403 +3.403
So, the solutions are x = = = 0.8508 or
4 4
3 6.403 9.403
= = 2.3508
4 4
Answers are normally given to 2 decimal places, so x = 0.85 or 2.35.
Repeat with 3x2 4x 2 = 0 (1.72, 0.39).
Repeat with x2 4x 1 = 0, but this time leave the answer in surd form.

4 20
The answers are or 2 5 .
2
The class can now do Exercise 16D from Pupil Book 3 or Exercise 10N
(page 259) from the Higher Mathematics for GCSE textbook.

182 HarperCollinsPublishers Ltd 2003


Exercise 16D Answers

1 a x = 4 or 6 b x = 1 or 7 c x = 2 or 12 d x = 212 or 4 e x = 0 or 5
f x = 6 or 6
2 a x = 1.14 or 2.64 b x = 1.59 or 1.26 c x = 1.32 or 5.32
d x = 2.22 or 0.22 e x = 0.16 or 6.16 f x = 0.19 or 2.69
g x = 1.82 or 0.18 h x = 2.19 or 0.69 i x = 4.56 or 0.44

(3 5 ) 4 20 4 12
3 a b or 2 5 c or 2 3
2 2 2

6 32 10 92 2 8
d or 3 8 e or 5 23 f or 1 2
2 2 2

Plenary Key Words


Ask students to solve 2x2 + 3x + 5 = 0 using the quadratic formula.
quadratic
This example could be worked through on the board by a student.
The formula gives formula
coefficients
3 31
x = which cannot be solved due to the square root.
4
Explain that when a quadratic equation is solved, the roots are where the graph
crosses the x-axis.
This gives three situations

2 roots 1 (repeated) root No roots

This will be useful for the investigation that ends this chapter.
Homework

1 Solve these equations using the quadratic formula. All answers are whole numbers or fractions.
a x2 + 4x 5 = 0 b 2x2 + 5x 3 = 0 c 6x2 19x + 10 = 0
2 Solve these equations, giving your answers to 2 decimal places.
a x2 + 7x 10 = 0 b 2x2 x 4 = 0 c 4x2 + x 7 = 0
3 Solve these equations, giving your answer in surd form.
a x2 4x 2 = 0 b x2 + 6x 1 = 0 c x2 + 5x 2 = 0

Answers
1 a 1 or 5 b 12 or 3 c 23 or 212
2 a 1.22 or 8.22 b 1.69 or 1.19 c 1.20 or 1.45

4 24 6 40 5 33
3 a or 2 6 b or 3 10 c
2 2 2

HarperCollinsPublishers Ltd 2003 183


LESSON Framework objectives Reinforcement of Number
Solving quadratics by completing the square.
16.5
Oral and mental starter
Ask students to fill in the missing number in problems such as 22 9 = ?
(Answer 13).
Repeat with other examples such as 32 + 5 (answer 4), 42 1 (answer 17).
Make sure that students understand that 32 = (3 squared) and not (3) squared.
Repeat with more examples if necessary.

Main lesson activity


This is a lesson on completing the square.
Ask students to expand (x a)2 and (x + a)2.
Hopefully they will get the answer x2 2ax + a2 and x2 + 2ax + a2.
Now ask if they can fill in the missing numbers in x2 + 4x 3 = (x + )2 .
Eventually they will probably come up with the answer (x + 2)2 7.
Discuss how this relates to the identities established at the start of the lesson.
Work through the following examples on completing the square.
Example: Write x2 + 6x in the form (x + a)2 b.
The value, a, inside the bracket is half the coefficient of x. The value b is the
square of this, So, x2 + 6x = (x + 3)2 9.
Repeat with x2 + 12x ((x + 6)2 36), x2 4x ((x 2)2 4) and
x2 + 30x ((x + 15)2 225).
Example: Write x2 + 6x 7 in the form (x + a)2 b.
The value of a is half the coefficient of x. The value b is the sum of the existing
constant term and the square as before.
So, x2 + 6x 7 = (x + 3)2 9 7 = (x + 3)2 16.
Repeat with x2 + 12x + 15 ((x + 6)2 21), x2 4x 3 ((x 2)2 7) and
x2 + 30x + 100 ((x + 15)2 125).
Demonstrate how to solve equations using completing the square.
Example: Solve x2 + 6x 4 = 0.

x2 + 6x 4 = 0 (x + 3)2 13 = 0 (x + 3)2 = 13 x + 3 = 13

x = 3 13.

Repeat with x2 6x 2 = 0 (x = 3 11 ) and x2 + 10x + 15 = 0 (x = 5 10 ).
Complete more examples if necessary.
The class can now do Exercise 16E from Pupil Book 3 or Exercise 10P
(page 260) from the Higher Mathematics for GCSE textbook.

184 HarperCollinsPublishers Ltd 2003


Exercise 16E Answers

1 a (x + 4)2 16 b (x 1)2 1 c (x 6)2 36 d (x 7)2 49 e (x + 2)2 4


f (x + 1)2 1
2 a (x + 4)2 17 b (x 1)2 + 2 c (x 6)2 31 d (x 7)2 42 e (x + 2)2 7
f (x + 1)2 6 g (x + 3)2 11 h (x + 5)2 34 i (x 3)2 6

3 a x = 4 17 b x = 3 or 1 c x = 6 31 d x = 7 42 e x = 5 or 1

f x = 3 11 g 5 24 h 3 5 i 4 11 j x = 1 2 k x = 1 6

l x = 6 43

Plenary Key Words


Ask students to solve x2 + 3x 2 = 0 using the completing the square method.

completing the
A student could be asked to come to the board to give the solution.
The odd coefficient of x will cause problems, particularly when taking a square square
root coefficient

i.e. x2 + 3x 2 = 0 (x + 112 )2 414 = 0 (x + 112 )2 = 414 x + 112 = 414

x = 112 414 .
Now ask students to solve 2x2 + 4x 3 = 0 using the completing the square
method.
2x2 + 4x 3 = 0 2(x2 + 2x) 3 = 0 2[(x + 1)2 1] 3 = 0
2(x + 1)2 2 3 = 0

2(x + 1)2 = 5, (x + 1)2 = 212 x + 1 = 212 x = 1 212 .
Homework

1 Complete the square for the following.


a x2 + 12x b x2 6x c x2 20x
2 Rewrite the following quadratic expressions by completing the square.
a x2 + 12x 9 b x2 6x + 3 c x2 20x + 100
3 Solve the following quadratic equations using the completing the square method.
a x2 + 12x 9 = 0 b x2 6x + 3 = 0 c x2 20x + 100 = 0
d x2 10x + 5 = 0 e x2 + 4x 7 = 0 f x2 8x 5 = 0

Answers
1 a (x + 6)2 36 b (x 3)2 9 c (x 10)2 100
2 a (x + 6)2 45 2
b (x 3) 6 c (x 10)
2

3 a x = 6 45 b x = 3 6 c x = 10 d x = 5 20 e x = 2 11 f x = 4 21

HarperCollinsPublishers Ltd 2003 185


LESSON Framework objectives Reinforcement of Number
Difference of two squares.
16.6
Oral and mental starter
Ask students to find the square root of 81.
They will obviously give the answer of 9, (answers can be written on mini white
boards) but may not give 9. Remind them that both answers are possible.
Now ask students for the square root of x2. They will, hopefully, give x as the
answer.
Now ask for the square root of 4x2. Some will give the answer 4x. Make sure
that they understand the answer is 2x.
Repeat with the square roots of 9y2, 16z2, 25x2, 81x2, 100y2 etc.

Main lesson activity


This is a lesson on factorising the quadratic the difference of two squares.
Start by asking students to expand expressions of the type (x + 2)(x 2) = (x2 4)
e.g. (x 4)(x + 4) = (x2 16), (2x 1)(2x + 1) = (4x2 1), (3x 6)(3x + 6) = (9x2 36) etc.
Eventually ask for the expansion of (a b)(a + b).
Discuss with students the similarities between the results.
Establish the result (a b)(a + b) = a2 b2.
Explain that this is known as the difference of two squares.
Now ask if students can reverse the result. i.e. can they factorise x2 36?
Explain that this can be written down as x2 62 = (x 6)(x + 6).
Emphasise that it is important to identify both squares since failure to do this can
lead to errors such as (x 36)(x + 36).
Repeat with x2 100 ((x 10)(x + 10)), 4x2 9 ((2x 3)(2x + 3)), 9x2 16y2
((3x 4y)(3x + 4y)).
Do more examples if necessary.
The class can now do Exercise 16F from Pupil Book 3 or Exercise 10J
(page 255) from the Higher Mathematics for GCSE textbook.

186 HarperCollinsPublishers Ltd 2003


Exercise 16F Answers

1 a x2 1 b x2 25 c x2 y2 d 4x2 1 e x2 4y2 f 4x2 9y2


2 a (x 10)(x + 10) b (x 2)(x + 2) c (x 6)(x + 6) d (x 9)(x + 9)
e (x 8)(x + 8) f (x 11)(x + 11) g (x z)(x + z) h (2x 5)(2x + 5)
i (x 3y)(x + 3y) j (4x 3)(4x + 3) k (2x 5y)(2x + 5y) l (5x 8)(5x + 8)
m (3 x)(3 + x) n (2x 6)(2x + 6) p (6x 1)(6x + 1)

Plenary Key Words


Ask the students to simplify (x + 2)2 (x 4)2.

difference of two
If possible, ask a student who wants to expand brackets to work through the
problem on the board, i.e. (x + 2)(x + 2) (x 4)(x 4) = (x2 + 4x + 4) squares
(x2 8x + 16) = x2 + 4x + 4 x2 + 8x 16 = 12x 12.
Now, ask if the original expression could be written in any other way. Some
students may recognise this as the difference of two squares.
Make sure that the students understand that the difference of two squares is an
identity, i.e. any values or expressions can be used for a and b in
a2 b2 = (a b)(a + b).
Using brackets (because of the minus signs), write the above as:
((x + 2) (x 4))((x + 2) + (x 4))
= (x + 2 x + 4)(x + 2 + x 4)
= (6)(2x 2)
= 12x 12.
Discuss the advantages of both methods.
Homework

1 Expand these brackets into quadratic expressions.


a (x + 11)(x 11) b (2x 3)(2x + 3) c (5x 2y)(5x + 2y)
2 Factorise the following quadratic expressions.
a x2 144 b x2 225 c 4x2 36
d 81x2 64 e x2 4y2 f 16x2 121
g x2 9z2 h 4x2 25y2 i 81x2 16y2

Answers
1 a x2 121 b 4x2 9 c 25x2 4y2
2 a (x 12)(x + 12) b (x 15)(x + 15) c (2x 6)(2x + 6) d (9x 8)(9x + 8) e (x 2y)(x + 2y)
f (4x 11)(4x + 11) g (x 3z)(x + 3z) h (2x 5y)(2x + 5y) i (9x 4y)(9x + 4y)

HarperCollinsPublishers Ltd 2003 187


LESSON Framework objectives Reinforcement of Number
An investigation into y = ax2 + bx + c.
16.7
Oral and mental starter
There is no starter as the investigation will need to be introduced via a class discussion.

Main lesson activity


The investigation in the Pupil Book is reproduced here. Students will need access to graphical calculators or
computers with a graph-drawing application package. The investigation will take more than one lesson.

The graph of a quadratic equation has a characteristic


shape called a parabola. y

Investigate the relationship between the values a, b and


c in the graph y = ax2 + bx + c and the points P, Q
(where the graph crosses the x-axis), the point R (where
the graph crosses the y-axis) and the point S (the vertex
or turning point of the graph).
y = ax2 + bx + c
You should use a graph plotting program or a graphical
calculator to help you.
You will not gain any credit for spending a lot of time
drawing graphs accurately.
P Q x
You will gain credit for a systematic investigation into
the effect of a, b and c on the graph.
R
S

Introduce the investigation and ask the students to suggest ideas for getting started.

For example, they could start by looking at graphs of y = x2, y = 2x2, y = 3x2 etc.
Then look at graphs of y = x2 + 1, y = x2 2, y = x2 + 3 etc.
Then look at graphs of y = x2 + 2x, x2 3x, x2 + 5x etc.
Once they have an idea of the effect of a, b and c individually they could then look at more general equations.
Students should be encouraged to look at the roots from the quadratic formula or the completing the square
method. The vertex also could be investigated via the completing the square method.

188 HarperCollinsPublishers Ltd 2003


GCSE Answers

1 0 and 5
2 a (x 4)(x 2) b x = 4, x = 2
3 a i 3( pq 2r) ii (c 4)(c 5)
b x = 7 or x = 2
p5
4 a (3p + 1)( p + 5) b
3p + 1
5 5.74, 1.74
6 a 9x2y6 b 3.14, 0.64
7 1.28, 0.78
8 a = 9 b = 2 c = 5
9 a = 5 b = 7
10 p = 2 q = 4

Plenary Key Words


There is no plenary. Students could be stopped working at various points and a
roots
general discussion on progress could be held. This can be helpful for students who
are having difficulty but avoid the bush fire effect of one student giving the rest of factors
the group the answer. vertex
intersect
coefficient
Homework

Students could be asked to continue the work at home if they have computer facilities or asked to look
on the Internet for information on the quadratic equation.

HarperCollinsPublishers Ltd 2003 189


Published by HarperCollinsPublishers Limited
7785 Fulham Palace Road
Hammersmith
London
W6 8JB

www.CollinsEducation.com
Online support for schools and colleges

HarperCollinsPublishers Ltd 2003

10 9 8 7 6 5 4 3 2 1

ISBN 0 00 713881 4

Keith Gordon, Kevin Evans, Trevor Senior and Brian Speed assert their moral rights to be identified as the authors
of this work.

All rights reserved. This CD-ROM must not be sold, rented, leased, sub-licensed, lent, assigned or transferred, in
whole or in part, to third parties. No part of this CD-ROM may be reformatted, adapted, varied or modified by the
user other than specifically for teaching purposes where enlargements and/or minor adaptations may be necessary.

This CD-ROM may not be reproduced or transmitted in any form or by any means, without the permission of the
publishers other than the form of printed copies for single use only.

British Library Cataloguing in Publication Data


A Catalogue record for this publication is available from the British Library

Edited by John Day


Typesetting and design by Gray Publishing
Project management by Nicola Tidman
Covers by Tim Byrne
Illustrations by Gray Publishing
Proofreading by Amanda Whyte and Jenny Wong
CD mastering by Alan Trewartha
Production by Sarah Robinson

The publishers would like to thank the many teachers and advisers whose
feedback helped to shape Maths Frameworking.

Every effort has been made to trace copyright holders and to obtain their permission for the use of copyright
material. The author and publishers will gladly receive any information enabling them to rectify any error or
omission in subsequent editions.

You might also like to visit:


www.harpercollins.co.uk
The book lovers website

190 HarperCollinsPublishers Ltd 2003

You might also like